You are on page 1of 196

‫أوليسِيُجزىِ ُمحسنٌ ِاالِباحسانِ‬

‫ن‬
‫طمئ قلوب الناس‬
‫أن‬
‫الناس‪ .‬للناس‬
ِ‫أوليسِيُجزىِ ُمحسنٌ ِاالِباحسان‬

Rule of the B’s.. If the pH & the bicarb are both in the same direction =
metabolic If they are in different directions = respiratory

RESPIRATORY OR METABOLIC !

PH ↑ HCO3 ↑ or PH ↓ HCO3 ↓ it is Metabolic

PH ↑ HCO3 ↓ or PH ↓ HCO3 ↑ it is Respiratory


Respiratoryِ‫…………………ِعكسِاالتجاهِتكون‬.Metabolic ِِِِ‫ِِِِِِ نفسِاالتجاهِتكون‬

pH = 7.35-7.45 acidosis/alkalosis HCO3


(bicarb) = 22-26 (2+2+2 = 6)
CO2 = 45-35
ACIDOSIS OR ALKALOSIS !

PHِ< 7.45 Alkalosis


PHِ> 7.35 Acidosis
Compensated or uncompensated !

Un compensated if PCO2 or HCO3 are Normal


Partially compensated if Nothing is Normal
Compensated if PH is Normal
----------------------------------------------------------------------------------------------------------------
ex:
PH: 7.30 ↓
bicarb: 20 = ↓ = metabolic acidosis
ex:
pH: 7.58 =↑
bicarb: 32 = ↑ = metabolic alkalosis
ex:
pH: 7.22 =↓
bicarb: 30 =↑ = respiratory acidosis
ِ‫أوليسِيُجزىِ ُمحسنٌ ِاالِباحسان‬
Vomiting Metabolic Alkalosis

Nasogastric tube suction Metabolic Alkalosis

Diarrhoea Metabolic Acidosis

Ileostomy Metabolic Acidosis

Dehydration Metabolic Acidosis

CPR Respiratory Acidosis

Drowning Respiratory Acidosis

Cushing syndrome Metabolic Alkalosis

1. A 12- year- old boy was brought to the Emergency respiratory arrest due to drowning. Cardiac resuscitation
what is the major complication that might happen if treated after drowning quickly?
A. Sepsis
B. Alkalosis
C. Acidosis
D. Hypothermia

2. An ICU nurse reviews the chart of a 47-year-old man patient mechanical ventilator for a long time. Arterial
blood gas result see lab results).
Test Result Normal Values
ABG HCO3 24 22-28 mmd/L
ABG PCO3 10.66 4.7-6.0 KPa
PH 7.16 7.36-7.45
ABG PO2 6.13 10.6-14.2 KPa
SA O2 81 95-100 %
What condition the patient is experiencing presently?
A. Metabolic acidosis
B. Metabolic alkalosis
C. Respiratory acidosis
D. Respiratory alkalosis

3. Gastric suction can cause :


A. Metabolic acidosis
B. Respiratory acidosis
C. Metabolic alkalosis
D. Respiratory alkalosis.

4. ABG reading was low PH ,HIGH PCO2 , NORMAL Hco3 what the interpretation ?
A. Compensated respiratory acidosis
B. Uncompensated respiratory acidosis
C. Metabolic acidosis
D. Metabolic alkalosis
5. Patient with ABG PH 7.33 ‫ و‬HCO3 30 , PCO2 50
A. Compensate respiratory
B. Compensate metabolic
ِ‫أوليسِيُجزىِ ُمحسنٌ ِاالِباحسان‬
C. Uncompensated respiratory
D. Uncompensated metabolic

6. The nurse assesses a client with an ileostomy for possible development of which of the following acid-base
imbalances?

A. Respiratory acidosis
B. Metabolic acidosis
C. Metabolic alkalosis
D. Respiratory alkalosis

7. The nurse is assigned to care for the a patient with Cushing syndrome on adrenal corticoid hormones syndrome
on adrenal corticoid hormones Ph 7.2 Which of the following condition should nurse expect to patient
A. Respiratory alkalosis
B. Respiratory acidosis
C. Metabolic alkalosis
D. Metabolic acidosis

8. A nurse reviewed a chart of a 42 year-old man whose ABG analysis report is shown PH 7.20 PCO2 35 HCO3
20. Which of the following I the most likely interpretation of the findings in the report?
A. Respiratory alkalosis
B. Metabolic acidosis
C. Respiratory acidosis
D. Metabolic acidosis

9. The nurse is assigned to care for the a patient with Cushing syndrome on adrenal corticoid hormones syndrome
on adrenal corticoid hormones Ph 7.2 Which of the following condition should nurse expect to patient
A. Respiratory alkalosis
B. Respiratory acidosis
C. Metabolic alkalosis
D. Metabolic acidosis

10. A 20 year old woman, a case of panic attacks, comes to the emergency department. An arterial blood gas
analysis is done PH 7.53 7.35-7.45 HCO3 22 22-26 PCO2 27 35-45 what is most likely?
A. Metabolic acidosis
B. Metabolic alkalosis
C. Respiratory acidosis
D. Respiratory alkalosis
ِ‫أوليسِيُجزىِ ُمحسنٌ ِاالِباحسان‬
Intravenous solution

11. A client was admitted in


the Emergency Room due to mild metabolic acidosis associated with dehydration and potassium the doctor
administered Hartmann’s (lactated Ringer’s) intravenous fluid and electrolyte replacement. Which of the
following elements of the lactated Ringer’s solution highest value?
A. Calcium
B. Sodium
C. Potassium
D. Magnesium

12. Ringer Lactate consider as which type of IV solution?


A. Hypotonic
B. Hypertonic
C. Isotonic
D. Hyper alimentation

13. A nurse reviewed a chart of a 42 year-old man whose ABG analysis report is shown PH 7.20 PCO2 35 HCO3
20. Which of the following I the most likely interpretation of the findings in the report?
A. Respiratory alkalosis
B. Metabolic acidosis
C. Respiratory acidosis
D. Metabolic acidosis

14. physician orders an intravenous fluid of D5NS at 100cc/hr. This is an example of which of the solution?
A. hyper alimentation
B. hypertonic
C. hypotonic
D. isotonic

15. 13-year-old patient is admitted for diarrhoea and vomiting. He looks pale and lethargic. A nurse is preparing to
give IV hypotonic solution.
Blood pressure 110/70 mmHg
Heart rate 76 /min
Respiratory rate 18 /min
Temperature 36.1°C
Which IV solution is most appropriate?
A. 0.9% saline
ِ‫أوليسِيُجزىِ ُمحسنٌ ِاالِباحسان‬
B. Lactated ringers
C. 10% dextrose in water
D. 0.45% sodium chloride

CPR Questions
16. CPR DEPTH ONE THIRD OF CHEST = 1.5 INCH =4CM IN INFANTS , IN ADULT 2 INCH = 5CM

17. CPR site

18. CPR Technique IS :


A. 30:2 120
B. 15:2 100

19. You are performing CPR on an infant when a second rescuer appears. What is the next step in management?
A. Immediately transport the patient
B. Wait until exhausted, then switch
C. Have the second rescuer help with CPR, to minimize fatigue
D. Have the second rescuer begin ventilations; ratio 30:2

20. What should be your first concern at the scene where a person has been seriously burned?
A. Checking the scene for safety.
B. Checking the victims breathing and pulse.
C. Calling your local emergency phone number.
D. Cooling the burned area.

21. While performing cardiopulmonary resuscitation (CPR) on a 5-year-old child, the nurse palpates for a pulse.
Which of the following sites is best for checking the pulse during CPR in a 5-year-old child?
A. Femoral artery
B. Carotid artery.
C. Radial artery
D. Brachial artery

22. After activating the emergency call system, what should be the next immediate action?
A. Initiate ventricular pacing
B. Administer a bolus of lidocaine as prescribed
C. Defibrillate the patient
D. Open the patient's airway

23. A nursing instructor teaches a group of students about basic life support. The instructor asks a student to
identify the most appropriate location to assess the pulse of an infant under 1 year of age. Which of the
following if stated by the student, would indicate that the student understands the appropriate procedure:
A. Carotid
B. Popliteal
C. Radial
D. Brachial

.
Chest Tube questions
ِ‫أوليسِيُجزىِ ُمحسنٌ ِاالِباحسان‬
24. A nurse is transferring a patient with chest tube the X-ray department for chest X-ray. Which location should the
nurse place chest tube
A. Directly on the stretcher in an upright
B. On the side of stretcher next to patient
C. Hanged on IV pole that is attached to the
D. Attached to the stretcher and hanged

25. 69 year-old man is admitted to the intensive care unit following cardiac surgery. Two hours after admission, the
nurse performs a routine assessment and notes the patient's chest tube drainage is 200 milliliters and a dark red
color. He has had 60 milliliters output from the indwelling urinary catheter
Blood pressure 138/68 mmhg
Heart rate 76/min
Respiratory rate 16/min
Body temperature 37.0C oral
Oxygen saturation 94% 6L/min nasal cannula
Which finding should be reported to the doctor?
A. Dark red chest tube drainage
B. Urinary output
C. Oxygen saturation
D. Chest tube output volume

26. 40 year-old male patient has a chest tube and properly fixed following cardiac surgery. The chest tube is patent
and functioning. When the nurse removes the dressing the tube falls out of the chest. Which is the most
appropriate initial intervention the nurse should take?
A. Reinsert the chest tube and notify the surgeon
B. Apply an occlusive dressing to the insertion site
C. Place the open end of the tube in 20 cm of water
D. Administer O2 at 10L/min via non-rebreather mask

27. While attempting to get of the bed a patient accidentally disconnects the chest tube from the pleura evac
drainage system which of the following actions should the nurse take first
A. Insert the end of the chest tube in a container of sterile solution
B. Raise the end chest tube above the level of insertion of the chest tube
C. Clamp the chest tube near pleura evac drainage system
D. Apply pressure dressing to chest tube insertion site

28. The nurse is caring for a client who has just had a chest tube attached to a water seal drainage system. To ensure
that the system is functioning effectively the nurse should:
A. Observe for intermittent bubbling in the water seal chamber
B. Flush the chest tubes with 30-60 ml of NSS every 4-6 hours
C. Maintain the client in an extreme lateral position
D. Strip the chest tubes in the direction of the client

29. A client chest tube is connected to a chest tube drainage system with a water seal . The nurse noted that the
water seal c is fluctuating with each breath that client takes . The fluctuation mwans that
A. There is an obstruction in the chest tube
B. The client is developing emphysema
C. The chest tube system is functioning properly
D. There is leak in the chest tube system

30. 20-Your patient has a chest tube. Your assessing the water seal chamber and you note ).11 that the water moves
up as the patient inhales and then moves down when the patient ?exhales. What may be causing this to happen
A. This is normal and expected
B. The chest tube has a leak
C. The left chest tube is occluded
ِ‫أوليسِيُجزىِ ُمحسنٌ ِاالِباحسان‬
D. The water seal suction should be increased 2-5 mmHg

31. 21-You are assisting a MD with the removal of a chest tube. What activity may the MD have ?the patient
perform while the chest tube is being removed
A. Valsalva maneuver.
B. Leopold Maneuver
C. Chest Physiotherapy
D. Huff Cough Technique

32. A 67 year-old man is admitted to the Post-anesthesia Recovery unit following chest surgery. The patient has a
right chest tube that is attached to low suction. Three hours after admission to the unit, the nurse observes the
drainage output from the chest tube is 300 milliliters. What is the most appropriate initial intervention?
A. Notify the doctor
B. Reduce IV infusion rate
C. Strip tube with roller device
D. Re-position in left lateral decubitus

33. The nurse is caring for a client who has had a chest tube inserted and connected to water seal drainage. The
nurse determines the drainage system is functioning correctly when which of the following is observed:
A. Continuous bubbling in the water seal chamber
B. Fluctuation in the water seal chamber
C. Suction tubing attached to a wall unit
D. Vesicular breath sounds throughout the lung fields
ِ‫أوليسِيُجزىِ ُمحسنٌ ِاالِباحسان‬

34. Which of the following is an example of epidemic point source?


A. Tuberculosis
B. Public health agency
C. Contaminated water source
D. Communicable disease pavilion
Airborne Droplet Contact
TB epidemic disease Small pox RSV
Corona virus Meningitis MRSA
measles Diphtheria Staphylococcus
chicken pox Rubella Ebola virus
influenza
Streptococcus
COVID 19

Tuberculosis
 TB epidemic disease
 TB. Wear N95 mask
 TB. Test Mantoux test / PPD TEST
 TB TEST WITHIN 48-72 Hours
 Isoniazid for TB side effect numbness , hepatitis symptoms , stomach upset ,rashes
 Tb accurate test sputum culture and to be done early morning

Meningitis. ...
 Streptococcal pneumonia
 meningococcal vaccine. Hib
 Neck rigidity
 Droplet
 Cloudy CSF
 Lumber puncture
 Surgical mask. No N95 mask

35. A 32 years old man develops chronic productive cough. He has not been feeling hungry and has lost three
kilograms body weight in the past three weeks. On examination of his lung
fields, there was diminished breath sound and widespread crackles. An early
morning sputum culture was sent to the lab.
A. Droplet
B. Contact
C. Airborne
Rationale TB chest XRAY

36. 8 year-old man is diagnosed with tuberculosis (TB) and negative pressure room. Which of the following should
wear a facemask?
A. Patient's health care providers
B. All people who enter the patient's room
C. Person has close contact with the patient
D. Family members who are at risk for infection
ِ‫أوليسِيُجزىِ ُمحسنٌ ِاالِباحسان‬

37. A nurse working in medical unit is preparing to with droplet precaution measures in place. The following
personal protective equipment; eyewear. What is the correct sequence foe putting the equipment on?
A. Face Mask, Gown, Eyewear, and Gloves
B. Gown, Face Mask, Eyewear, and Gloves
C. Eyewear, Cloves, Face Mask, and Gown
D. Gloves, Gown, Face Mask, and Eyewear

38. A nurse working in medical unit is going out (removing) with droplet precaution measures in place. The following
personal protective equipment; eyewear. What is the correct sequence foe putting the equipment off?
A. Face Mask, Gown, Eyewear, and Gloves
B. Gown, Face Mask, Eyewear, and Gloves
C. Eyewear, Cloves, Face Mask, and Gown
D. Gloves, Eyewear , Gown, , and Face Mask

39. Mr X attended in outpatient clinic with symptoms of shortness of breath, diarrhea and severe respiratory
distress Which of the following is the best diagnosis of Mrs. A case?
A. Corona virus
B. Swine Flue
C. Zika virus
D. Hepatitis

40. A nurse receives a telephone call from the admission office of the hospital and is told that a patient with
streptococcal meningitis will be admitted to the Medical Unit. The nurse is planning to apply infection control
measures for the patient. Which type of isolation precaution the nurse must observe?
A. Droplet precautions
B. Contact precautions
C. Airborne precautions
D. Standard precautions

41. A nurse receives a telephone call from the admission office of the hospital and is told that a patient with
streptococcal meningitis will be admitted to the Medical Unit. The nurse is planning to apply infection control
measures for the patient. Which type of isolation precaution the nurse must observe?
A. Droplet precautions
B. Contact precautions
C. Airborne precautions
D. Standard precautions

42. A 35 year old patient was admitted to a medical ward with confirmed agnosies f meningococcal infection.
Which of the following infection control preventions the nurse should implement?
A. Droplet precaution
B. Contact precaution
C. Airborne precaution
D. Standard precaution
43. A 16 month old child is hospital in the intensive care unit with multi resistant sepsis. On the 3 rd day. She had
explosive diarrhea. A stool sample was sent to the laboratory for C. difficle investigation. Which of the following
transmission based precaution is most appropriate?
A. Combination airborne and droplet
ِ‫أوليسِيُجزىِ ُمحسنٌ ِاالِباحسان‬
B. Contact
C. Droplet
D. Airborne

44. Which of the following is the most effective infection control precaution to prevent nosocomial infection?
A. Hand washing before and after patient contact
B. Wearing gloves and mask for direct patient care
C. Isolation precaution
D. Broad spectrum antibiotic

45. Mantoux test done for a patient result was 6.5cm it consider?
A. Latent
B. Suspected
C. Active

46. When planning discharge teaching for a patient hospitalized for treatment of the 3rd burns over 30% of the
body, a nurse knows it is most important to include instructions regarding the loss of large amounts of serum
occurring with burns and the resulting loss of immune function. Which of the following instructions should be
include?
A. Wash hands frequently each day
B. Wear supplemental oxygen at night
C. Wear masks while in public spaces
D. Take a multiple vitamin tablet night

47. An infection control nurse notices purulent exudates , redness and tenderness on the surgical wound site for
few post –operative patients in a surgical unit , She discussed with the ward nurse and emphasized that wound
infection after the surgery can be prevented. Which of the following is the best possible action to minimize the
incidence of wound infection ?
A. Perform assessment of pain on the wound site
B. Wash hand before and after each patient activity
C. Encourage adequate intake and early ambulation

48. A patient in surgical was transferred to isolation room after the wound swab confirmed to have methicillin
Resistant staphylococcus Atreus MRSA. Which of the following measures should the nurse take to prevent
infection in the ward?
A. Clean the would three times a day
B. Discard all soiled dressing into waste bag
C. Instruct the patient to wash hands regularly
D. Wear gloves and gown on every entry into the room

49. Medical asepsis requires which of the following hand washing techniques?
A. Use hot water to ensure that pathogens are killed
B. Use circular motion washing from clean to dirty areas
C. Rinse soap off keeping hands and forearms lower than downs
D. Hands shall be held higher than elbows and scrub
50. A 45-year-old patient admitted with pulmonary tuberculosis. The unit nurse placed the patient in an isolation
room with negative air pressure and prepared all the personal protective equipment at the entrance of the
room. What type of precaution measure has the nurse activated?
A. Contact
B. Droplet
ِ‫أوليسِيُجزىِ ُمحسنٌ ِاالِباحسان‬
C. Airborne
D. Standard

51. To reduce the risk of transmitting methicillin-resistant staphylococcus auras (MRSA) from an infectious wound,
which of the following precautions should be implemented?
A. Airborne
B. Contact
C. Droplet
D. Reverse isolation

52. A 45 year old patient is admitted with pulmonary tuberculosis. The unit nurse placed the patient in isolation
room with negative air pressure and prepared all PPE at the entrance of the room. Which of the following PPE is
the most important for the nurse when caring for this patient?
A. Hair cover
B. Sterile gloves
C. N95 respirator
D. Protective goggle

53. 8 month-old child diagnosed with bronchiolitis due to respiratory ncytial virus (RSV) is admitted to the pediatric
hospital. The nurse should initiate which of the following isolation precaution?
A. Contact
B. Airborne
C. Standard
D. Droplet

54. A 5-year-old child is seen in the primary care clinic with mild fever, headache, and malaise for about 2 days and
today he has a rash filled with fluids. which of the following is the best suggested diagnosis?
A. Chicken pox
B. German measles
C. Measles
D. Scarlet fever

55. A eight year-old boy present to the pediatric clinic with his mother who complains that the child has developed a
skin rash. There is a clear drainage from the nose and he appears tired. The rash first appeared on the face and
then spread over the trunk and is now beginning to appear over the hands. Some lesions show open ulcerations
and other have crusted over. There are scratch marks over the skin. Which finding would indicate the child is
ready to return to school
A. Normal body temperature
B. Dry and scabbed lesions
C. Nares of free of drainage
D. Absence of rash over trunk

56. A patient with measles (rubella) is on airborne precautions. Which of the following precaution techniques would
be essential to implement for non-immune persons entering the room?
A. Gloves
B. Gowns
C. Masks
D. Face shields
ِ‫أوليسِيُجزىِ ُمحسنٌ ِاالِباحسان‬
57. A home health nurse has entered a home to complete an admission assessment on a patient who has a
methicillin-resistant Staphylococcus aureus (MRSA) urinary tract infection. The patient will receive intravenous
anti-infective via a peripherally inserted central catheter (PICC) for 3 weeks. Which of the following actions
should the nurse take FIRST?
A. Shake the patient’s hand
B. Place the nursing supply began a clean, dry surface.
C. Obtain the patient’s written consent for home health care
D. Perform hand hygiene per the agency protocol

58. You have just admitted a patient with bacterial meningitis to the medical-surgical unit. The patient complains of
a severe headache with photophobia and has a temperature of 102.60 F orally. Which type of isolation should be
suitable for the patient?
A. Air borne
B. Contact
C. Droplet

59. In order to reduce the risk of disease transmission from a patient with diphtheria, which of the following
standard precautions would the nurse implement?
A. Airborne
B. Contact
C. Droplet
D. Ventilatory

60. Patient with a colostomy complains of itching of the peris tornal skin. On assessment, the skin is covered in a red
rash with white patches visible. What is the most likely cause of this condition?
A. Not changing the pouch regularly
B. Candidiasis
C. Consuming acid-producing foods
D. Dehydration

61. A 33-year-old woman has come to the outpatient clinic for treatment of a vaginal infection. Physical assessment
reveals yellowish excessive, thin offensive and frothy discharge. Which of the following is the most likely
diagnosis?
A. Candidiasis
B. Trichomoniasis
C. Bacterial vaginosis
D. Chlamydia

62. The nurse is assessing a 65-year-old patient, who reports the fatigue, weight loss, night sweats, and a productive
cough with thick sputum The nurse should immediately initiate isolation precautions for which of the following?
A. Influenza
B. Pertussis
C. Bacterial pneumonia
D. Pulmonary tuberculosis

63. Which of the following diagnostic tests is definitive for TB?


A. Chest x-ray
B. Mantoux test
ِ‫أوليسِيُجزىِ ُمحسنٌ ِاالِباحسان‬
C. Sputum culture
D. Tuberculin test

64. A patient presented with high fever, headache, vomiting and neck stiffness for the past 3 days, which of the
following is the first diagnostic intervention for this patient:
A. Urine and stool analysis
B. lumber puncture with CSF aspiration
C. Complete blood count
D. Chest and abdomen x-ray

65. When a patient is admitted with acute influenza, what type of isolation is MOST appropriate?
A. Reverse isolation
B. Contact isolation
C. strict isolation
D. Respiratory isolation

66. What precautions are necessary when caring for a patient with Hepatitis A?
A. Gowning before entering the room
B. Wearing gloves for direct care
C. Wearing a mask at all times
D. Placing the patient in a private room

67. The nurse is assigned to care for the a patient with Ebola virus disease. Which of the following is the most
common mode of transmission for Ebola virus?
A. Vector
B. Airborne
C. Direct contact
D. Common vehicle

68. A nurse is caring a patient diagnosed with pulmonary tuberculosis and she has to wear N95 mask when she
enters to the parent room. Based on the cycle of infection, which of the following is the reason to use the mask?
A. Break the transmission chain at the portal of entry.
B. Break the transmission chain at the portal exit
C. Kill the agent exits in the patients room.
D. Eliminate the reservoir of the bacteria

69. After receiving the initial treatment of TB , a 27 years old woman in the chest disease ward was given discharge
instruction on dietary management , medication, regimen, hygiene care and follow up visits. What is the
instructions need to be emphasized the most?
A. Increase protein diet to promote healing
B. More rest and relaxation to restore therapy
C. Maintain daily exercise schedule to improve health
D. Dispose sputum as guided to avoid spread of disease

70. A patient is admitted to the emergency room with an unknown infection disorder. Which of the following is the
most important measure a nurse implement to reduce the risks of transmitting these microorganisms other?
A. Hand hygiene
B. Wearing gloves
C. Wearing a mask
D. Wearing a gown
ِ‫أوليسِيُجزىِ ُمحسنٌ ِاالِباحسان‬
71. A nurse is assigned to care for a patient with small that needs to be placed on airborne precaution. Which of the
following item should be used before entering the patient room?
A. Gloves and gown
B. Gloves and goggles
C. Gloves and mask
D. Gloves , gown N 95

72. Which of the following is the primary level of prevention?


A. Detect and treat existing disease
B. Prevent illness or injury occurring
C. Reduce the extent and severity of health problem
D. Minimize disability and restore to prevent function

73. A Nurse is surgical unit prepares for a wound dressing. While she is preparing. She sneeze over the dressing
trolley without wearing a facemask. Which of the following is the most appropriate action should the nurse
take?
A. set up a new sterile field
B. put on a mask and continue working
C. continue working the most sterile field
D. Replace the equipment she thinks is contaminated

74. Which of the following is the most effective infection control precaution to prevent nosocomial infection?
A. Hand washing before and after patient contact
B. Wearing gloves and mask for direct patient care
C. Isolation precaution
D. Broad spectrum antibiotic

75. A 16 month old child is hospital in the intensive care unit with multi resistant sepsis. On the 3 rd day. She had
explosive diarrhea. A stool sample was sent to the laboratory for C. difficle investigation. Which of the following
transmission based precaution is most appropriate?
A. Combination airborne and droplet
B. Contact
C. Droplet
D. Airborne

76. When planning discharge teaching for a patient hospitalized for treatment of the 3rd burns over 30% of the body,
a nurse knows it is most important to include instructions regarding the loss of large amounts of serum occurring
with burns and the resulting loss of immune function. Which of the following instructions should be include?
A. Wash hands frequently each day
B. Wear supplemental oxygen at night
C. Wear masks while in public spaces
D. Take a multiple vitamin tablet night

77. A 20 year old woman is hospitalized with a strong and uncontrollable cough and has difficulty breathing while
coughing. A doctor writes an order for the patient to be transported from the medical surgical department to
the radiology department for an xray examination. The nurse prepares to transfer the patient and considers
standard precautions and additional transmission precautions. Who of the following would be required to wear
a mask?
A. Radiology staff
B. Nurse and patient
ِ‫أوليسِيُجزىِ ُمحسنٌ ِاالِباحسان‬
C. Patient
D. Nurse

Thyroidectomy
78. A patient returned to the Surgical Unit from the thyroidectomy. The nurse observed that the arousable. Blood
pressure 90/60 mmHg Heart rate 108 /min What immediate action should the nurse take?
A. Recheck pulse and blood pressure
B. Administer intravenous fluids as ordered
C. Place client in modified Trendelenburg's
D. Assess the back of neck surgical dressing for bleeding

79. Complication of thyroidectomy


A. Distension
B. Bleeding
C. Vocal cord injury

80. progressive enlargement of a multi-nodular go tracheal compression including pain at the site of
the ear and jaw, difficulty swallowing, change of pf breath by compressing the oesophagus. There is in otherwise the
patient is at risk. What is the preferred treatment?
A. Iodine treatment
B. Thyroid hormone treatment
C. Radioactive iodine treatment
D. Surgical resection of abnormal thyroid

81. A Post thyroidectomy patient transferred to ward and developed tetany the nurse suspected that patient has ?
A. Hypokalemia
B. Hypocalcemia
C. Hyponatremia

82. A nurse Wayne is aware that a positive Chvostek’s sign indicate


A. Hypocalcemia
B. Hyponatremia
C. Hypokalemia
D. Hypermagnesemia

83. Trousseau’s sign A sign of hypocalcemia. Carpal spasm can be elicited by compressing the brachial artery with a
blood pressure cuff for 3 minutes.

84. Chvostek’s sign A sign of hypocalcemia. A spasm of the facial muscles elicited by
tapping the facial nerve just anterior to the ear

85. Patient is being admitted to the Recovery Room following a thyroidectomy. The back of the neck wound is
covered with dressing. During the first 15 minutes, the patient started working and having diarrhea . A general
ِ‫أوليسِيُجزىِ ُمحسنٌ ِاالِباحسان‬
assessment is performed with special attention given for the high risk for haemorrhage . Where would bleeding
most likely occur?
A. Stool
B. Vomitus
C. Dressing
D. Back of neck

86. A 29 year-old man is in the Surgical Ward on his first post-operative thyroidectomy. He appears drowsy but the
he is able to respiration by nodding head. He is developing mild restlessness. What is the initial recommended
goal of care?
A. Monitor vital signs of thyroid storm
B. Assess for bilateral vocal fold mobility
C. Monitor for swelling on the neck
D. Monitor for vocal cord paralysis

87. A nuclear plant experienced a leakage, and all involved workers were brought to the Emergency Department for
treatment . the medical team prescribed potassium iodide for the workers to block radioactive iodine to be
absorbed by an organ in the body. Which of the following organs is the most sensitive to radioactive iodine?
A. Brain
B. Lungs
C. Kidney
D. Thyroid

88. The nurse care for a 60 year old woman who history hypertension, hypothyroidism and elevated cholesterol
levels. She takes tablets daily for each of the health problem. The doctor orders a routine dual- x-ray
absorptiometry test that shows decrease bone density. Which medication most likely contributed the test
result?
A. Statins
B. Anti-hypertensive
C. Synthetic thyroid hormones
D. Cholesterol absorption inhibitors

89. A 36 years old man ha undergone a subtotal thyroidectomy ago. he Is suspected to develop tetany after the
surgery. Which of the following symptoms best indicates tetanus?
A. Tingling in the fingers
B. Pain in hands and feet
C. Tension on the suture lines
D. Bleeding on the back of the dressing

90. The nurse is assessing 50 year old woman whose thyroid enlarged. A blood sample was collected and an analysis
confirm diagnosis.
TSH 0.12 normal 0.4-6.5
Free 210 normal 50-140.
Thyroxin T4 normal 4.5-11.2
A. Thyroidectomy
B. Incision and drainage
C. polythyroidectomy
D. adrenalectomy

Tonsillectomy
ِ‫أوليسِيُجزىِ ُمحسنٌ ِاالِباحسان‬
91. 3-year-old child with an elevated body temperature is administered oral aspirin. The nurse records the body
temperature of the child two hours an The American Academy of pediatrics
suggests that removal of the tonsils under certain conditions. Which of the
following meets these conditions?
A. times viral tonsillitis per year
B. Infrequent snoring and nasal quality
C. Three times bacterial tonsillitis per year
D. Tonsillitis accompanied by adenoid inflammation

92. 5year-old child is postoperative after tonsillectomy. The nurse should ask the parents to give the child which of
food after discharge from the hospital?
A. Meat and rice
B. Hot dog and potato chips
C. Mashed potatoes and soup
D. Cucumbers and tomato salad

93. The nurse is receiving a child postoperative tonsillectomy. Which of the following nursing assessment is suitable
for the postop care?
A. Encourage the child to cough spontaneously
B. Observe for subtle signs of haemorrhage
C. Place the child in the prone position
D. Suction the mouth to clear the airway Because of violent behaviour

94. The nurse is receiving a child postoperative tonsillectomy. Which of the following nursing action is suitable for
the postoperative care?
A. Encourage the child to cough spontaneously
B. Observe for subtle signs of hemorrhage
C. Place the child in the prone position
D. Suction the mouth to clear the airway Because of violence

95. After tonsillectomy, a child begins to vomit bright red blood .the Initial nursing action is to?

A. Notify the physician


B. Turn the child to the side
C. Maintain an NPO status
D. Administer the prescribed antiemetic

96. A client presents with Hypocalcemia, hyperphosphatemia, muscle cramps, and positive Trousseau's sign. What
diagnosis does this support?
A. Diabetes insipidus
B. Conn's syndrome
C. Hypoparathyroidism
D. Acromegaly

97. The nurse cares for a client who has undergone a tonsillectomy. The nurse is most concerned about which post-
operative finding?
A. Lack of appetite
B. Throat pain
C. Frequent swallowing
D. Nausea
Appendicitis
ِ‫أوليسِيُجزىِ ُمحسنٌ ِاالِباحسان‬
98. 18-year-old man college student was rushed fainted at the school. He complained of severe quadrant. Upon
palpation, he jerks even with sample was obtained. What is the most likely diagnosis of this patient?
A. Appendicitis
B. Liver Cirrhosis
C. Kidney stones
D. Duodenal ulcer

99. - year-old child was admitted with suspected appendicitis. A nurse was evaluating the child’s condition and the
mother stated that the child did have his bowel movement for the past two days and requested for natives.
What is the risk of giving laxative to patient with appendicitis?
A. Pain
B. Fever
C. Rupture
D. Diarrhea

100. A woman patient is admitted for abdominal pain. She complain generalized plan nausea vomiting and
constipation rebound tenderness and abdominal rigidity the past hour her localized on right side
BP 130/68 HR 92 RR 18 TEM 38.6 What should the nurse suspect
A. Intestinal obstruction
B. Influenza
C. Appendicitis
D. Pyloric stenosis

101. A man is to be discharged from the General appendectomy. The precautionary measures, plans are
discussed with him. What is the most important desired outcome after discharge
A. Remain free of post-surgical complications
B. Report fever, redness or drainage from the wound site
C. Use pain management techniques appropriate
D. Resume gradual activities and avoid weight

102. A 17 year-old arrived to the Emergency Room complaining abdominal pain on right lower quadrant. Pain was
rated as 9 numeric scale with positive rebound tenderness over the pain
Blood pressure Heart rate Respiratory rate Température
120/70 mmHg 95 /min
20 /min 39.2
Which of the following interventions has the highest priority?
A. Keep NPO
B. Secure an IV access
C. Prepare for ultrasound
D. Prepare for abdominal surgery

103. The nurse would increase the comfort of the patient with appendicitis by:
A. Having the patient lie prone
B. Flexing the patient's right knee
C. Sitting the patient upright in a chair
D. Turning the patient onto his or her left side

104. A 10-year-old girl presents to the Emergency Room (ER) with pain. On assessment, you noticed that when
you palpate the right lower quadrant of the child’s abdomen, the child feels pain in the rig quadrant.
Which of the following is the name of this sign?
A. Rebound tenderness
B. McBurney sign
ِ‫أوليسِيُجزىِ ُمحسنٌ ِاالِباحسان‬
C. Roving’s sign
D. Obdurate sign

105. A 10-year-old girl presents to the Emergency Room (ER) with pain. On assessment, you noticed that when
you palpate the left lower quadrant of the child’s abdomen, the child feels pain in the rig quadrant.
Which of the following is the name of this sign?
A. Rebound tenderness
B. McBurney sign
C. Roving’s sign
D. Obdurate sign

106. Treatment of nephritic syndrome is corticosteroids

107. Isoniazid for TB side effect numbness , hepatitis symptoms , stomach upset ,rashes

108. A 66 year old woman with a history of unstable angina and hypertension present to the emergency
department with a dull chess pain that she describes as similar to heartburn. The pain radiates down the left
arm. She had taken sublingual nitro-glycerine tablets with any relief any electrocardiograph is perform and
shows elevated S T segments. Which medication is most likely to given to dissolve the thrombus?
A. Heparin
B. Warfarin
C. Streptokinase
D. Aspirin

109. A 62 year man with a history of intracerebral bleeding three months ago, was referred primary health care
following acute starch symptoms. Brain CT scan is normal and the patient is receiving oxygen by nasal cannula at
4LPM
BP 185/105 HR 82 RR 18 TEM 36.6 SO2 93%
Which of the following medication would be ordered first?
A. RtPA
B. Aspirin
C. Dopamine
D. Nicardipine

110. A nurse checks the medication chart of a patient, which has the following order, nitroglycerin Tab sublingual.
Which of the following medication administration routes should the nurse use?
A. In the nose
B. In the eyelid
C. Under the skin
D. Under the tongue

111. A 28 year man admitted to orthopedic ward complaining of throbbing pain in casted leg. Which of the
following nursing intervention should be taken first?
A. Remove the cast
ِ‫أوليسِيُجزىِ ُمحسنٌ ِاالِباحسان‬
B. Notify a doctor
C. Assess pedal pulse
D. Administer PRN medication

112. A 62 year old woman presents to the clinic with a primary compliant of fatigue. An assess shows a smooth
and reddened tongue with a loss of papillae and pallid mucous membranes. She complaints of a sense of
constant fullness of the stomach, with a decreased appetite and two three loose bowel movement per day. A
neurological assessment shows numbness in the feet and lower legs. The patient type of test would the most
likely?
A. Schilling
B. Erythropoietin
C. Folic acid levels
D. Vitamin B12 levels

113. A mother of a patient who is on antipsychotic drug asked a doctor about some more detail about the drug.
The doctor told the mother that her son is taking an atypical antipsychotic drug. Which of the following is an
examples of this type of drug?
A. Thioridazine
B. Clozapine
C. Chlorpromazine
D. Haloperidol

114. A nurse is preparing scheduled medications due at 6 pm. If a doctor orders paracetamol tab 1g QID, and it
was supplied from the pharmacy in 250 mg tablets. Which of the following is the most appropriate nursing
actions?
A. Ask the pharmacy to provide 1g tablets
B. Call the doctor to recheck the dosage
C. Give the patient four 250 mg tablets
D. Hold the medication and document in nursing notes

115. A patient with a history of atrial fibrillation has an order for 25 milligrams of drug (X), the available
supply/dose is 0.25 milligrams.
Which of the following doses in correct?
A. Dispense two tablets to the patient
B. Administer one tablet twice daily
C. Dispense one half tablet to the patient
D. Return tablets to the pharmacy and re-order

116. 13-year-old patient is admitted for diarrhea and vomiting. He looks pale and lethargic. A nurse is preparing
to give IV hypotonic solution.
Blood pressure 110/70 mmHg
Heart rate 76 /min
Respiratory rate 18 /min
Temperature 36.1°C
Which IV solution is most appropriate?
A. 0.9% saline
B. Lactated ringers
C. 10% dextrose in water
ِ‫أوليسِيُجزىِ ُمحسنٌ ِاالِباحسان‬
D. 0.45% sodium chloride

117. A nurse is assigned to care for a group of patient in the medion expected to review the medical records of
these patient. What patient is at risk for excess fluid volume?
A. Patient with ileostomy
B. Patient taking a loop diuretic
C. Patient with chronic renal failure
D. Patient hooked to gastrointestinal

118. physician orders an intravenous fluid of D5NS at 100cc/hr. This is an example of which of the solution?
A. hyper alimentation
B. hypertonic
C. hypotonic
D. isotonic

119. When a patient was first diagnosed with schizophrenia, one of his family members asked the nurse about the
possible causes. The nurse said that one reason is that he may have had an excess secretion of a
neurotransmitter. Which of the following neurotransmitters?
A. serotonin
B. dopamine
C. glutamate

120. The nurse administered a dose of morphine sulfate as prescribed to a patient who is in the post anesthesia
care unit (PACU). The patient appears to be resting comfortably, the respiratory rate is 8 and the O2saturation is
21 oxygen via cannula is 86%. The nurse should IMMEDIATELY administer:
A. Flumazenil(Romazicon)
B. Medazolum(versed)
C. Naloxone (Narcan)
D. Ondansetron (Zofran)
121. Digoxin not given to pàtient who has which of the following ?
A. tachycardià
B. hypertension
C. tachypnea
D. bradycardia

122. Atropine is indicated for increase


A. Heart rate
B. Respiratory rate

123. Morphine side effect ?


A. tachycardià
B. hypertension
C. tachypnea
D. bradypnea

124. patient with ongoing magnesium sulfate the nurse should be alert to ?
A. temperature
B. respiratory rate
C. heart rate
ِ‫أوليسِيُجزىِ ُمحسنٌ ِاالِباحسان‬
125. Soldier was brought to triage area after being exposed to chemical weapons. Signs and symptoms of nerve
gas exposure were noticed. A nurse prepares for medical management. Which medication should the nurse
prepare for the patient?
A. Atropine
B. Adrenaline
C. Sodium nitrate
D. Sodium thiosulphate

126. Nitroglycine administration route :


A. Intramuscular
B. Sublingual
C. Subcutaneous
D. Oral

127. A nurse checks the medication chart of a part, which has the following order; Nitroglycerin tab Buccal at stat.
Which of the following medication administration routes should the nurse use?
A. In the ear
B. Intravenous
C. Under the skin
D. Between cheek and gum

128. A nurse is caring for a patient who had Coronary Artery bypass Graft Surgery (CABG) four hours ago. The
nurse notices that the patient has increased confusion and is restless. The patient reports nausea, weakness and
paresthesia in the extremities (see lab results)
Normal Values
Result Test
134-146 mmol/L
145 Sodium
3.5-5.2 mmol/L 6.8 Potassium
2.15-2.62 mmol/L 2.50 Calcium
Which of the following is the best medication?
A. Naloxone (Narcan)
B. Hydralazine (Apresoline)
C. Potassium chloride (KCI)
D. Sodium polystyrene sulfonate (Kayexalate

129. Which of the following statement by the nurse about the clomid as an ovulation inducing drug?
A. Given for the first 15 days in each cycle
B. Maximum dose is 50 mg daily for a month
C. It increases the risk of birth defects
D. It increase the risk of multiple pregnancies

130. Clomiphene citrate (Clomid) is prescribed for a 32-year-old infertility treatment. The nurse should
understand that this medication is used for following actions?
A. induce ovulation
B. Decrease prolactin level
C. Reduce endometriosis
D. Stimulate the release of Follicle-Stimulating Hormone

131. A nurse is caring for a client with bipolar disorder, who is receiving Lithium carbonate. Before Administration
of the next dose, the client complains of nausea and vomiting and the nurse finds that the client’s lithium blood
level 1.5mEq/lL. Which of the following actions is considered Apriority?
ِ‫أوليسِيُجزىِ ُمحسنٌ ِاالِباحسان‬
A. call the client’s physician immediately
B. withhold the next dose.
C. Administer IV fluids
D. Repeat the blood lithium level testing

132. A patient was on a regular dose of lithium carbonate. The nurse noticed he has hand tremor, polyuria,
diarrhea and vomiting. What immediate action should be taken by the nurse?
A. Diuretics
B. Withholding lithium
C. Calling the psychiatrist
D. Monitoring serum lithium level

133. A patient was on a course of lithium carbonate drug. During the nurse found that he complained from
nystagmus visual hallucination, and oliguria Which of the following drug related complications best symptoms?
A. Overdose
B. Mild toxicity
C. Severe toxicity
D. Moderate toxicity

134. 49-year-old women presented to the Emergency Department complaint of severe chest pain. The ECG
showed that the patient myocardial infarction. The doctor ordered the nurse to give the 800 mg of aspirin. What
is the primary indication of aspirin in this case?
A. Breaks down the thrombus
B. Decreases the formation of platelet plugs
C. Inhibits the conversion of prothrombine to
D. Interferes with vitamin k to maintain

135. which of the following condition is a Contraindication for a woman oral Contraceptives?
A. Dysmenorrhea
B. Menorrhagia
C. Thrombophlebitis
D. Toxic shock syndrome

136. A patient was on a course of lithium carbonate drug. During the nurse found that he complained from
nystagmus visual hallucination, and oliguria Which of the following drug related complications best symptoms?
A. Overdose
B. Mild toxicity
C. Severe toxicity
D. Moderate toxicity

137. A 45-year-old distressed and restless patient in the Psychiatric Ward was unable to sleep during the nights
for the last two days. He was ordered sleep medication which was to be administered at 10 pm. At the time
medicine administration, the patient was found asleep in bed Which of the following action should the nurse
take regarding the dictation?
A. Discard medicine and cancel the order
B. Leave it at the bed side for the patient
C. Wake him up and administer
D. Hold, record and report
ِ‫أوليسِيُجزىِ ُمحسنٌ ِاالِباحسان‬
138. 13- nurse is given health education to the parent how to reduce febrile avulsion in the child at home. Which
of the following medication in the safest intervention?
A. Analgesic
B. Antipyretic
C. Antibiotic
D. Antiemetic

139. Doctor order to give Ofloxacin Otic route the nurse understand that medication will be given :
A. Nasal
B. Eye
C. Ear
D. Sublingual.

140. A home care patient with chronic Obstructive reports an upset stomach. The patient is taking and
triamcinolone acetonide (azmacort). Which of the following counselling should be given
A. Theo-dur on an empty stomach
B. Theo-dur and azmacort at the same time
C. Theo-dur and azmacort 12 hours apart
D. Theo-dur with milk or crackers

141. Which of the following vitamins is necessary for wound healing process?
A. Vitamin A
B. Vitamin C
C. Vitamin D
D. Vitamin K

142. Which of the following vitamin supplements can decrease the incidence of Neural tube defects such as
anencephaly and spina bifida new-borns ?
A. Vitamin A
B. Riboflavin
C. Folic Acid
D. Vitamin K

143. A 78-year-old woman who lives in a long-term care facility has been ving repeated episodes of urinary tract
infections. She is prescribed then amine mandelate one gram to be taken by mouth four times per before meals
and at bedtime. The nurse advises the patient that she need to remove milk from the diet while taking the
medication. What is the primary purpose for this dietary advice?
A. To prevent mal absorption of medication
B. Decrease risk of gastrointestinal upset
C. Reduces effectiveness of medication
D. To make the urine acidic

144. A nurse received the serum digoxin level result for the patient the day and notes that the result is
2.6 ng/mL (see lab result)

Test result normal value

Digoxin (men) 2.6 0.8-2ng/mL Which of following


nursing actions is the most important?
ِ‫أوليسِيُجزىِ ُمحسنٌ ِاالِباحسان‬
A. Notify the physician
B. Check previous vital signs of patient
C. Record normal value on nursing note
D. Administer scheduled dose of medication

145. A patient is being admitted in medical unit and has orders for fluid restriction of 1400ml / 24 hours. He is
receiving IV medications in 50ml solutions QID. How much fluid should the caring nurse allocate for cra intake in
24 hours?
A. 400 ml
B. 800 ml
C. 1000 ml
D. 1200 ml

146. A nurse is preparing an order of Amydramine 30 mg PO BD. The available check is 15 mg /5 ml. What is the
correct dosage of the medication to be administered?
A. 5 ML
B. 10 ML
C. 15 ML
D. 20 ML

147. A nurse is preparing a medication order of 100 mg IV stat. The available hydrocortisone is 200mg per 5 ml
distilled water for injection. Which of the following is the correct administered?
A. 4.5 ML
B. 3.5 ML
C. 2.5 ML
D. 1.5 ML

148. A nurse is preparing to administer an intravenous infusion of 2000 ml of Ringer's lactate over 12-
hours. The administration set has a chamber that delivers 15 drops per ml. How many drops per
minute should be administered?
A. 14
B. 24
C. 42
D. 56

149. Doctor order to give medication x 300mg and available is 900mg/6ml how many ml the nurse will give
A. 1.2
B. 2
C. 4
D. 6

150. A 56-year-old man was admitted with complaint working for three days. The nurse is preparing to administer
infusion of saline 1000 ml over six hours. What is the hourly infusion rate that the nurse needs to infusion
therapy in mL/hour?
A. 155
B. 167
C. 190
ِ‫أوليسِيُجزىِ ُمحسنٌ ِاالِباحسان‬
D. 217

151. Doctor Order: Heparin 10000 units OD S/C Standard Solution 40000 units/ml How many ml would you
administer?
A. 0,25.ml

152. A Physician has ordered clindamycin phosphate 300 mg to be administered intravenously the available stock
was labelled . clindamycin phosphate 900mg mg in 6 ml what is the correct dose to be administered by the nurse
?
A. 2 ml
B. 4 ml
C. 6 ml
D. 8 ml

153. A nurse check the medication chart of a patient, which has the following order; Paracetamol 500mg P.O.
QID. How many times a day should the nurse administer the medication?
A. Once
B. Twice
C. Three times
D. Four times

154. The following syringe contains 2 gm of morphine sulphate, how many grams are there in 1 ml:
A. 0.5 gm
B. 1 gm
C. 2 gm
D. 4 gm

155. The following syringe contains morphine sulphate in a concentration of 2 mg/cc, how much total morphine
this syringe contains:
A. 0.5 mg
B. 1 mg
C. 2mg
D. 4 mg
156. A nurse is assigned to care for a patient in medical ward
who has a physician order of 1500ml of normal saline 0.9% to be administered over 12 hour period. Which of the
following is the infusion rate?
A. 75 ml/hr
B. 100ml/hr
C. 125ml/hr
D. 150ml/hr

157. Doctor order to give 75ml per hour over 12hours of normal saline how many ml total should be given?
A. 450ml
B. 600ml
C. 900 ml
D. 1000 ml

158. A 37year old female prevented to the Emergency room with complain of symptomatic bradycardia
appropriate nursing intervention include establish intervention access what is the possible drug to be given in
the IV solution ?
A. Digoxin
B. Atropine
ِ‫أوليسِيُجزىِ ُمحسنٌ ِاالِباحسان‬
C. Anticoagulant
D. Calcium channel blocker

159. A 25 year old woman present to the emergency room with decrease reflexes, hypoventilation, hypotension,
and fuced pupils, a family member who is accompanying the patient has bottle of diazepam which the label
states was recently the family member also indicates that the patient has a depression. What intervention
should the nurse expect to administer?
A. Naloxone
B. Active charcoal
C. Tap water enema
D. Magnesium sulfate to reduce the risk seizure

160. Nurse gives health education to patient In outpatient department regarding a medication . she told him
"avoid working outdoors in a hot weather, excessive sweating ad during l water daily Which of the following is
most likely medication?
A. Diazepam
B. Haloperidol
C. Lithium carbonate
D. Monoamine oxidase inhibitor

161. During CPR of an admitted patient following the cardiac arrest. Two Dc shocks and one dose of epinephrine
was given. A third dose was delivered followed by anti arrhythmic medication . what medication is the most
likely to be given next?
A. Adenosine
B. Lidocaine 2 mg
C. Epinephrine 1 mg
D. Sodium bicarbonate

162. A nurse is giving discharge planning instructions to the parents of a one year old child acute otitis media
which of the following discharge instruction takes priority?
A. Administrate antibiotics as prescribed
B. Administer influenza vaccination
C. Breast feeding as long as possible
D. Continue using of pacifier

163. A patient comes to the emergency unit with cough and severe dyspnea. The patient’s medical history
revealed a diagnosis of chronic heart failure and chronic obstructive pulmonary disease.
Blood pressure 110/70 mmhg
Heart rate 87/min
Respiratory rate 23/min
Temperature 37.3 C
Which of the following diagnostic tests will be most beneficial to a nurse to figure out if there is an
exacerbation of heart failure?
A. B-type natriuretic peptide (BNP)
B. arterial blood gas (ABG)
C. cardiac enzymes (CK-MB)
D. chest x-ray

164. A 4 years old girl, was playing outside, she came to her mom crying and holding her right upper arm, she went
to the hospital with swelling over the upper arm, pain and itching, the appropriate management is:
A. Maintain patent airway
B. Administer s/c Epinephrine
C. Prepare for intubation
ِ‫أوليسِيُجزىِ ُمحسنٌ ِاالِباحسان‬

165. An older adult client with renal failure comes to the emergency department with a report of nausea and
vomiting. The client's heart rate is 45 beats/min. The nurse is most concerned about which medication that the
client takes
A. Nitroglycerin
B. Digoxin
C. Doxorubicin
D. Furosemide

166. The nurse is caring for a 60-year-old in the Medical-Surgical Unit. On entering the room the nurse finds the
patient alert, oriented and. The skin color appears healthy and pink. The nurse observes electrocardiogram and
notices absent P-waves. Which initial action is the most important?
A. Defibrillate
B. Cardio version
C. Pacemaker preparation
D. Intravenous amiodarone

167. During health education for a patient on Monooamine oxidase inhibitor (MAOI) the nurse instruct him to
avoid certain foods that can interact with the MAOI. Food containing with of the following contents should be
avoided?
A. Alcohol
B. Caffeine
C. Tyra mine
D. Folic acid

168. After teaching a 54 year old patient with angina on how to take nitroglycerin sublingual PRN. Which of the
following statements reflect the patient understanding?
A. I have to take this medication once i need it only
B. It is ok to take one tablet daily to prevent the heart attack
C. I can take two tablets together at once if the attack is
D. This medication will regulate my heart beats and I will be

169. A 45 year old client who diagnosed with brain was schedule for craniotomy. It is important to preventing the
developing of cerebral edema after surgery. What medication would the nurse expect to prescribed the client?
A. Steroids
B. Diuretics
C. Ant convulsions
D. Antihypertensive
170. A 71 year-old woman who resides in a long-term nursing home fell while walking down stairs. The attending
nurse arrives to find the patient sitting motionless on the stairs. She is alert and oriented but wishes to rest.
While she rests, the nurse reviews the chart and notes that her medication regimen includes metformin,
loratadine, warfarin and diclofenac. Which medication is most likely to increase the patient's risk of injury?
A. Metformin
B. Loratadine
C. Warfarin
D. Diclofenac

171. ECG showed S-T elevation to confirm diagnosis of MI the nurse expect to do?
A. Troponin level
B. Cardiac catheterization
C. ECO
ِ‫أوليسِيُجزىِ ُمحسنٌ ِاالِباحسان‬

172. On orientation day, a nurse educator said to a new nurse" our hospital provides the highest level of
specialized healthcare in an integrated educational and research setting" which of the following aspects of the
hospital's strategic plan is indicated by this statement?
A. Vision
B. Policy
C. Values
D. Mission

173. A head nurse of an intensive care unit wrote a report about the needs of her department and gave it to the
nursing supervisor. Which of the following types of communication is this?
A. Upward
B. Horizontal
C. Diagonal
D. Downward

174. A head nurse of a coronary care unit delegated staff scheduling to a senior nurse in that unit. Which of the
following steps must the head nurse implement before delegating tasks?
A. Negotiate with the senior
B. Take signature of senior
C. Explain task to senior
D. Make sure hospital policies for delegating tasks

175. A 45 year old patient has a colostomy bag attached as a result of benign mass removal, while conducting the
discharge teaching session, the nurse assessed the patient's wife understanding of the teaching. Which of the
following steps of nursing process is exercised here?
A. Implementation
B. Planning
C. Assessment
D. Evaluation

176. Which of the following is correct nursing process order?


A. Diagnosis, assess, plan, evaluate, implement
B. Assess, diagnosis, plan, implement, evaluate
C. Evaluate, assess, plan, implement, diagnosis.
D. Plan, assess, diagnosis, evaluate, implement

177. A nurse calls a physician regarding a change in a patient's condition. A physician gives orders over the
telephone for stat arterial blood gases. Which of the following is the best intervention to be implemented?
A. Give order stat to health unit coordinator to input in computer
B. Write down order and read it back to physician
C. Write down order for ABGs immediately
D. Call respiratory therapist stat to draw the ABGs
ِ‫أوليسِيُجزىِ ُمحسنٌ ِاالِباحسان‬
178. During Cardiopulmonary Resuscitation (CPR) for a 75-year-old man in the Emergency Department, the
doctor introduced himself as a leader for the CPR. What is the most appropriate leadership style for this
situation?
A. Autocratic
B. Democratic
C. Bureaucratic
D. Laissez-faire

179. A nurse manager is open minded, listens to the team, understands others, makes changes to improve unit
operations and procedures. What type of leadership style is this?
A. Situational
B. Democratic
C. Compassionate
D. Transformational

180. A nurse manager includes staff in decision-making process what unit structure is used?
A. Divisional
B. Centralized
C. Functional
D. Decentralized

181. What is the main limitation of democratic style of leadership?


A. it ignores the incompetent personnel
B. cannot be used in emergency situations
C. decreases collaboration among the team
D. may end up in inappropriate decision making

182. A nurse manger is preparing and writing a plan for dealing disasters (code red). Which procedure is the top
priority for the nurse manager the plan?
A. Infection control
B. Staff orientation
C. Patient education
D. Patient relocation

183. nurse manager was not happy about low results of patient’s survey. Which of the following is the first step
for nurse manager?
A. Schedule meeting with staff
B. Start changes to improve
C. Review evaluation details
D. Report to director

184. A 14- nurse manager assigns tasks according to clinical competencies of the nurses. Which of the following is
the management function?
A. Delegating
B. Evaluating
C. Planning
D. Controlling

185. post-operative patient who underwent an abdominal procedure requests a pain medication from the nurse
and rates the pain at a level nine. There a standing order for narcotic administration. When the nurse opera the
narcotic box and performs a count, the number of pills remaining in the box is different than the number of pills
recorded on the sheet. What is the most appropriate initial nursing action?
ِ‫أوليسِيُجزىِ ُمحسنٌ ِاالِباحسان‬
A. Notify the nursing supervisor
B. Write the finding on the narcotic sheet
C. Administer the patient's requested medication
D. Identify the last nurse who used the narcotic box

186. An 81-year-old bed ridden patient in the Geriatric Ward was pyretic through his nasogastric tube. After an
hour, the patient deep sleep, his breathing pattern changed and he did not rep calling his name. Which of the
following should be the immediate nursing intervention?
A. Call the physician to examine the patient
B. Check for the correct dosage given
C. Check vitals and inform physician
D. Try to wake up the patient

187. The nurses in the Male Medical Unit took a signature on the from a patient who needs an abdominal CT
with contrast. What is the best nursing practice?
A. Consent should be taken after the procedure
B. Verbal consent is acceptable for this procedure
C. Consent should be taken from the patient's relatives
D. Consent should be taken according to the organization's

188. In the hospital digital dashboard, what types of data displayed?


A. Staffing
B. Financial
C. Performance
D. Knowledge-based

189. A newly nursing director assigned to a hospital. That is the first should he do?
A. Evaluates the staff
B. Change the head nurses
C. Change the roles
D. Nothing to do

190. surgeon instructs a nurse to serve as a witness to an elderly patient’s informed consent for surgery. During
the explanations to the patient, it becomes clear that the patient is confused and does not understand the
procedure, but reluctantly sign the consent form. The nurse should:
A. Sign the form as a witness, making a nation that the patient did not appear to understand
B. Not sign the form as a witness and notify the nurse supervisor
C. Not sign the form and answer the patient’s questions after the surgeon leaves he room

191. A community nurse assigned to work in a Makkah, one of the most multicultural cities in Saudi Arab has to
interact with multicultural population every day What is the most important step the nurse should do before
with the clients?
A. Asking the client about his/her cultural background
B. Conducting an appropriate culturological assessment
C. Ensuring that the client has his/her a privacy
D. Looking at the client's file to take history
ِ‫أوليسِيُجزىِ ُمحسنٌ ِاالِباحسان‬
192. The head nurse meets with staff nurses to discuss ways to improve communication among shifts Which of
the following statement best exemplifies the final stage of conflict management?
A. "We need to clearly define the nature of the conflict”
B. " I will evaluate the outcomes of the strategies used monthly
C. " Let us create a time line for the implementation of our strategies”
D. " I have to force you to follow the rules to resolve the issue”

193. The head nurse of a Coronary Care Unit delegated the staff a senior nurse in that unit What initial step must
the head nurse implement before?
A. Check the hospital policies for delegating tasks
B. Explain the task to the senior nurse
C. Negotiate with the senior nurse
D. Take the signature of the senior nurse
194. What must be known on legal points of delegation when process to a new nurse?
A. Evaluation of performance of delegate by clients
B. Actual time it takes to complete the task by delegat
C. Institution definition of the job description of
D. Number of times that the delegate has previously task

195. The nursing director wants to evaluate the quality of nursing care at the in-patient areas. The management
team will evaluate on quarterly basis the documentation and the relationship between the patient's length of
stay and the quality of care which of the following is the most important data source to identify the quality of
care?
A. Patient's complaints and time taken to resolve them
B. Patient's satisfaction level at the time of discharge
C. Details of nursing notes for patient's progress

196. The nurse manager received complaints from some nursing to improper work distribution and
discrimination How should the nurse manager handle the situation?
A. Investigate the complaints
B. Individual counselling
C. Identify list of problems
D. Continue to observe

197. which of the following is the most appropriate action for a head nurse starting at a new hospital?
A. Make immediate change at the unit
B. Plan and coordinate new strategies
C. Assess unit activities for at least three months
D. Ask about the previous head nurse managerial style

198. After teaching the deep breathing and coughing who is undergoing a surgery, the nurse asked
Demonstration and then helped him in correcting what part of therapeutic communication is used
A. Evaluation
B. Intervention
C. Identification
D. Demonstration
ِ‫أوليسِيُجزىِ ُمحسنٌ ِاالِباحسان‬
199. the intensive care unit nurse manager plans to delegates a nurse. What is indicated for a successful
delegation?
A. Nurse Manager supervises nurse.
B. Nurse has authority to change task.
C. Nurse Manager checks task sometimes.
D. Nurse Manager asks another nurse to check task.

200. The head nurse of a Coronary Care Unit delegated the staff a senior nurse in that unit What initial step must
the head nurse implement before?
A. Check the hospital policies for delegating tasks
B. Explain the task to the senior nurse
C. Negotiate with the senior nurse
D. Take the signature of the senior nurse

201. nurse accidently dropped a medication ampoule, informed the charge nurse, and completed an incident
report form. The charge nurse arranges medication replacement.
Which of the following is the immediate nursing action required?
A. Revise protocol for medication related incidences
B. Allocate senior nurses to medication assignments
C. Provide missed medication dosage to patient first
D. Incident reporting must be given the priority

202. The nursing assistant with 20 years of experience approaches a recently graduated nurse who recently passed
the licensing examination. The nursing assistant states “the only difference between you and me is the size of
our pay checks”. Which of the following is the most appropriate
response for the newly graduated nurse?
A. assert a hierarchical position
B. emphasize the additional education received
C. explain the legal difference in the scope of practice
D. focus on the need to work together for quality client care

203. Nurse manager first days in new hospital will..?


A. introduce plan and strategy to top manager in hospital
B. introduce himself to staff
C. observation for 3 month before begin work

204. After accessing patients' medical records, which behavior nurse shows that patients confidentiality has been
breached?
A. Reviews patients medical record
B. Read patients care plan
C. Disclosing patients information

205. During the assessment phase of a preoperative interview, the patient reports feeling nervous. The patient
conveys to the nurse that a parent died in surgery due to malignant hyperthermia .to whom would this
information be most important ?
A. PACU NURSE
B. SCRUB NURSE
C. ANAESTHESIA TEAM
D. CHARGE NURSE

206. a charge nurse in the nurse was noted poor staffing schedule in one of the shift. The schedule caused
problems among staff members in the unit. The women the charge nurse should bring the problem to?
ِ‫أوليسِيُجزىِ ُمحسنٌ ِاالِباحسان‬
A. Supervisor
B. Chief nurse
C. Follow charge nurse
D. Grievance committee

207. nurse responsible for narcotic medication loses on ampule morphine. She reports the problem to nurse
manager. Which of the following is the first thing she should say according to the SBAR approach?
A. I have lost one ampule of morphine
B. We should ask the nurse about the morphine
C. I think I left the lid of narcotics box open
D. We should open an investigation into problem

208. A nurse intervention a patient recently admitted to long term care facility to obtain information on the patient
health perception. The nurse encourage the patient to elaborate this change. Which of the following type of
question would be the most effective in this situation?
A. Analytical
B. Focused
C. Closed
D. Open ended

209. A nurse enters the room of a patient named Ahmed Saeed to administered medication but the patient not
wear his identification bracelet. Which of the following is the most appropriate nursing action?
A. Ask the patient are you Ahmed Saeed?
B. Ask the patient what is your name
C. Ask the patient hat mediation do you take

210. Research data to be collected from children on nursing unit. What type of consent is the most appropriate ?
A. Informed written consent
B. Consent signed by parents
C. Consent from physician
D. Hospital administration consent

211. Nurse manager prepares unit clinical operational plan What is top priority in the plan?
A. Infection control
B. Staff orientation
C. Quality projects
D. Safe patient care

212. .What is a characteristic of democratic leadership style?


A. More changes
B. Structured work
C. Staff participation
D. No leader decisions

213. Which of the following vaccines in highly recommended to be taken by pilgrims before Haji season?
A. Meningococcal meningitis
B. Tuberculosis
C. Hepatitis A
D. Polio myelitis
ِ‫أوليسِيُجزىِ ُمحسنٌ ِاالِباحسان‬
214. Which of the following is the main reason the makes nurses concerned about adolescents health status?
A. Take risky behaviors
B. Consider themselves as adult
C. Have more health issues
D. Transitional period to adulthood

215. A patient fell in the bathroom and his left leg was fractured, in order to communicate information about the
patient to the next shift. Which of the following documentation should be used by the nurse at the nurse at the
end of the shift?
A. Kardex record
B. Assignment record
C. Shift report
D. Incident report

216. A nurse is showing stress and anxiety due to long duty hours. Which of the following should the nurse
manager do ?
A. give day off
B. give light tasks
C. offer vacation
D. talk about coping

217. Ovulation occurs 14 days before the onset of the next menstrual cycle

218. A woman informed a nurse that she was never vaccinated against rubella. Which of the following is the best
nursing advice?
A. No need for her to be distress, rubella is not harmful to the fetus
B. The vaccine can be administered any time during her pregnancy
C. She can get pregnancy any time after receiving the vaccine
D. She should be vaccinated after delivery of the baby she get discharge

219. A 29 year old woman had been diagnosed with a 3 cm ovarian cyst. Which of the following is the appropriate
step in management?
A. Cyst aspiration
B. Hormonal therapy
C. Cyst removal by laparoscopy
D. Examination after next menstruation

220. A diabetic mother delivered a full term neonate by Caesarean section infant is admitted to the neonatal
intensive care unit for observation. This infant is at risk of which of the following complication?
A. Pneumothorax atelectasis
B. Hyperglycemia
C. Atelectasis
D. Hypoglycemia
ِ‫أوليسِيُجزىِ ُمحسنٌ ِاالِباحسان‬
221. The midwife was assessing a 36 year old gravis 4 para 2mother. The patient was in labour for 10 hour and
had extraction. Two saturated pads were fully soaked with blood with hours been admitted in the post natal
ward. Which of the following is the appropriate nursing diagnosis?
A. Anxiety related to blood loss
B. Fatigue related to lack of oral intake
C. Activity intolerance due to discomfort
D. Fluid volume deficit due to uterine atony

222. Which dietary intake should be initiated in pregnant woman?


A. Yogurt
B. Soft cheese
C. Processed cheese
D. Pasteurized milk

223. A 33 old woman presented to the ER with general weakness. The laboratory investigation indicated VIT D
deficiency. Which of the following nutrient should be recommended as a good source of vitamin?
A. Rice
B. Green tea
C. Orange juice
D. Fish liver oils

224. The midwife was caring for a 30 year old gravida 3 para 3 postpartum normal delivery mother. After three
hours, the patient was restless, her skin was cool, clammy and feeling thirsty. What will be midwife’s initial
action?
A. Notify the doctor
B. Check the vital sign
C. Give patient a drink
D. Start fundal massage

225. Which of the following statement indicate nursing action during the first hour after delivery of the placenta?
A. Monitor of mothers hemoglobin
B. Assess maternal vital signs every 15 minutes
C. Ensure that the mother mobilize and empty her bladder
D. Administer 10 units of oxytocin via IV line to ensure uterus is well contracted.

226. Which of the following nursing responsibilities should be done immediately following administration of
lumbar epidural anesthesia to a woman in labor?
A. Reposition from side to side
B. Administer oxygen
C. Assess for maternal hypotension

227. A nurse is assessing the uterus of a G5P4 patient immediately after delivery. The nurse notes the fundus is
not contracted. Which of the following is the most appropriate immediate action should be taken?
A. Massage the fundus
B. Assess the bladder
C. Elevated the mother's legs
D. Encourage the mother to void
ِ‫أوليسِيُجزىِ ُمحسنٌ ِاالِباحسان‬
228. A nurse is assisting during a normal vaginal delivery on a 22 year diabetic patient. The head was delivered
without any complication head suddenly retracts against the perineum prompting the physician to immediately
ask for the nurse assistance with this dystocia. Which of the following will be the nurse appropriate action to
impacted shoulders of the infant?
A. Fracture the infants clavicle.
B. Prepare patient for immediate cesarean section
C. Apply fundal pressure to displace anterior shoulder
D. Perform supra public pressure to release anterior shoulder

229. At labor room, a nurse assessed the condition of the patient and gathered the following data. Cervical
dilatation 2-5minutes lasting 40-60seconds increasing bloody show leg discomfort with heaviness what is the
significance of the data?
A. Patient on the first stage of labor
B. Patient on the third stage of labor
C. Patient on the second stage of labor
D. Patient is experience a prolonged labor

230. A primigravida mother is having her baby through normal vaginal delivery. The baby is completely delivered
but the mother is still experiencing the uterine contractions.
A. Beginning of the third stage of labour
B. Indication of increase in vaginal bleeding
C. Need for reducing the rate of intravenous oxytocin
D. Uterine contraction will gradually reduce then stop.

231. A mother is the outpatient Clinic for her first post natal visit on the 15th day her normal vaginal delivery. Her
physical examination reveals a stable condition, breasts are soft and her sanitary napkin has bright coloured
rubra. Which of the following needs further evaluation?
A. Amount and frequency of breast feeding
B. Hydration level and bleeding breast feeding
C. Activity, exercise and resting periods
D. Uterine size and position

232. A midwife is discussing the birth spacing measures with a mother whose first baby boy is a 1 year old and
was born with spina bifida. The midwife has explained the possible causes of condition of the baby and the
measures is necessary if the mother is planned for a next pregnancy?
A. Increase iron and calcium supplements
B. Multivitamin and folic acid intake
C. Genetic screening
D. Immunization before and during pregnancy

233. A 32 weeks pregnant patient present for her follow up appointment in the antenatal clinic. She complains of
experiencing frequent heartburn. Which of the following is the most appropriate advice to the patient?
A. Drink plain water between meals
B. Raise head of the bed
C. Eat favorite foods
D. Lie down for one hour after taking food
ِ‫أوليسِيُجزىِ ُمحسنٌ ِاالِباحسان‬
234. A midwife visits a mother four weeks after delivery. The mother is breastfeeding her baby but she requests
the midwife to suggest alternative formula milk as she has to return back to her job and her baby will stay in a
day care center. Which of the following teaching plans is suitable for the mother?
A. Supplementary medications with bottle feeding
B. Combined schedule of the breastfeed and top feed
C. Hygiene practices with bottle feeding
D. How to express and save milk

235. A women in labor is progressing well. She has been diagnosed with a large fibroid in the fundus. What should
the nurse observe her for after delivery?
A. Thrombophlebitis
B. Postpartum depression
C. Postpartum hemorrhage
D. Loss of bladder tone during puerperium

236. Which of the following assessment is contraindicated while providing intrapartum nursing care for a patient
with HELLP syndrome?
A. Heart sounds
B. Blood studies
C. Leopold's maneuvers
D. Deep tendon reflexes

237. For which of the following issues should the nurse observe the movie closely during the 4TH stage of labor?
A. Uterine irritability
B. Signs of infection
C. Signs of bleeding
D. Unwillingness to breastfeed

238. A 32year old woman has just been told that she is pregnant she states ‘ As mush as I love my children , I had
had hoped we would not have any more “ this statement reflect of which of the following related to pregnancy
A. Anger
B. Denial
C. Guilt
D. Ambivalence

239. After nurse assessed a newborn she reported that the baby has syndactyly the student nurse ask the nurse in
charge what is the syndactyly ?
Which of the following is the best nursing response
A. Fistula.
B. Abnormal big head
C. Extra finger or toes
D. Finger or toes wholly of partly united

240. A newborn is diagnosed with Ventricular Septal Defect (VSD) Which of the following information should the
nurse give to the newborn's mother?
A. Cyanosis will occur most of the time during sleeping
B. Breast feeding is not recommended for your child
C. The defect might close spontaneously after 6 months
D. Blood pressure is different on the child's arm and the leg

241. A couple asked the nurse which of the first investigation they should do for infertility?
ِ‫أوليسِيُجزىِ ُمحسنٌ ِاالِباحسان‬
Which of the following should be the proper a nurse answer?
A. Hysterosalpingogram
B. Serum progesterone
C. Semen analysis
D. Endometrial biopsy

242. A nurse is assessing a 5 month old infant who has admitted to the pediatric ward with coarcotation of aorta
Which of the following is the most common assessment findings ?
A. Cyanosis and clubbing hands
B. Bounding pulse and hypotonicity
C. Cyanosis occur frequently during and after feeding
D. Blood pressure is different on the arm the leg

243. A 36 years old woman is in her 18 weeks pregnancy came to antenatal clinic and assessed by the nurse. She
has three previous abortion during the first trimesters prior to pregnancy. How the nurse will document the
status of the mother?
A. Parity
B. Nullipara
C. Multipara
D. Primipar

244. During vaginal examination the nurse palpated the posterior fontanel to be at the right side and upper
quadrant of the maternal pelvis ?
A. ROP
B. LOP.
C. ROA
D. LOA

245. A 38 years old gravida 5 para 3 post natal mother delivered via caesarean section. On the fourth post
operative day the mid wife noted human's sign was positive . human's sign is result by pain which of the
following leg site?
A. Calf
B. Foot
C. Heel
D. Thigh

246. Which of the following vaccines are safe to administer to pregnant women?
A. Measles
B. Tetanus
C. Rubella
D. Varicella

247. Which of the following is independent function of primary health care nurse?
A. Complete history and nursing assessment
B. Referral for diagnostic evolution
C. Medical examination
D. Psychotherapy referral

248. Which of the following statement by the nurse describes the effect of clomid an ovulation inducing drug?
A. Given for the first 15 days in each cycle
ِ‫أوليسِيُجزىِ ُمحسنٌ ِاالِباحسان‬
B. Maximum dose is 50mg daily for a month
C. Increase the risk of birth defects
D. It increases the risk of multiple pregnancy

249. A 50 year old woman is attending the outpatient clinic for annual check up. Which of following should be
reported if inspected during breast examination?
A. Everted nipples
B. Symmetrical visible
C. Dimple in the left breast
D. Right breast is slightly larger than left breast

250. A pregnant woman is 36 weeks pregnant was admitted to antenatal ward for observation after being
involved in a car accident the refuses when the nurse tried to touch her abdomen to perform examination
saying it is painful. She also present mild vaginal bleeding . Which of the following is the most appropriate
diagnosis
A. Placenta previa
B. Tubal pregnancy
C. Abruptio placenta
D. Inevitable abortion

251. Mother of nine children, three of them with congenital anomalies and one down syndrome, she is a primary
school graduate, with low financial status, she is not using any method of family planning . according, the
primary health are nurse has referred her for counseling.
Which of the following is a model concerned with disability as from of social injustice due to stigma, or
discrimination
A. Health belief
B. Biomedical
C. Sociopolitical
D. Economic

252. A primiparous mother is calling the clinic to report that her baby often regurgitates small amount of breast
milk which of the following is appropriate nursing advice the mother?
A. Elevate the head of the bed
B. Feed the baby when he is too hungry
C. Attend to the clinic with the baby
D. Put the baby in prone position after feeding

253. A 10 month old infant is admitted to the surgical ward with hydrocephalus. Which of the following .indicate
increased intracranial pressure?
A. Bulging
B. Decrease blood pressure
C. Rapid, shallow breathing
D. Increase body temperature

254. A multigravida and 37 weeks pregnant mother presents t the antenatal Clinical. Her condition of the stable
and her baby position and heart rate are normal. Which of the following discussion is more appropriate with thr
mother?
A. Observer until the 40weeks complete
ِ‫أوليسِيُجزىِ ُمحسنٌ ِاالِباحسان‬
B. Family planning and birth spacing
C. Arrange for caesarean section
D. Possible of induced labor.

255. A 26 years old patient in the gynecological ward complained of pain and swelling in her episiotomy stitches
on her second post-natal day. On examination the localized swelling redness foul smell and pussy discharge were
identified at the stitches site after examination the nurse checked vital sings BP 116/28 HR 132 RR 28 TEM
39.8. which of the following should the nurse do prior to the patient examination by the gynecologist?
A. Arrange dressing instrument
B. Maintain patient privacy
C. Arrange stitch removal kit
D. Collect detailed history

256. A 28-year-old primigravida who is pregnant at 16 weeks of attended at the emergency department because
of dark brow discharge. investigation showed a decline in pregnancy test for which of the following is the most
likely type of the spontaneous abortion?
A. Missed
B. Threatened
C. Incomplete
D. Inevitable

257. A nurse is educating a primigravida woman who is pregnant at 30weeks on breast feeding. Which of the
following statement by the woman indicates that she needs additional teaching?
A. Breast milk can stored at room temperature
B. Breast feeding should be based on baby demand
C. Baby can beheld in different ways during feeding
D. Breast feeding helps the uterus to return to pregnancy size

258. Which of the following vaccines should be given to a 9 months old ?


A. Hepatitis
B. Varicella & measles
C. Oral polio and bacillus cellmate Guerin
D. Measles and meningo conjugate quadrivalent ( MCV4)

259. nurse is instructing a female client how to do breast self-exam. Which of the followings is the best time to perform this
exam?
A. After ovulation
B.After the period
C. Two weeks after period
D. Three days before period

260. A nurse is explaining pudendal block anesthesia to primigravida women who is inactive labor. Which of the
following relief areas identified by the woman would indicate that teaching was effective?
A. Back
B. Perineum
C. Fundus
D. Abdomen
ِ‫أوليسِيُجزىِ ُمحسنٌ ِاالِباحسان‬
261. Which of the following classifications of placenta previa is applicable when the placental edge is 5 cm away from the
internal cervical OS?
A. Total
B. Partial
C. Marginal
D. Complete

262. A postdate pregnant woman is admitted for the induction of labour. Her fetal heart rate and vital signs are
within normal range her intravenous line is maintained and she is to be started on low doses of labour inducing
medication. Which of the following medication the mother is likely to receive intravenously ?
A. Oxytocin
B. Cervidil
C. Cytotec
D. Cytoxin

263. Which of the following patients are at risk for cord prolapse?
A. Fetus that remains at high station
B. Mother with oligohydramnios
C. Presenting part at station +
D. Intact membranes

264. While taking care of the a patient with a spinal cord injury, the patient suddenly complains of pounding
headache upon assessment the patient was found to have diaphoresis , drop in heart and elevated blood
pressure , autonomic dysreflexia is suspected and the head on the bed is elevate . which of the of the following is
the most appropriate to important immediately?
A. Notify the physician
B. Assess bladder for distension
C. Continue to monitor for next hour

265. vital statistics and control of communicable disease serve many function. Which of the following is the basic
function for health?
A. Public
B. Mental
C. Education
D. Maternal & child

266. A 6 months old infant mother decided to wean her child. Which of the following is the best principle of
weaning process?
A. Start the weaning process by 8 month of life
B. Gradually replace one breast session at a time
C. Discontinues the nighttime feeding first
D. Allow the child to take a bottle of milk or juice bed

267. A midwife is conducting a health a health education session to the primigravid mothers. The session is about
antepartum care to ensure the health of the mother their babies. Which of the following statement made by
mother suggest their appropriate understandings of anti-partum periods ?
A. Beginning of labor till the baby birth
B. Diagnosis of pregnancy till the baby birth
C. Last three months of pregnancy
D. Forty days after the child is born

268. A 41 weeks pregnant was admitted to labor and delivery unit for induction with oxytocin infusion 3 hour
later her contraction are 5 to 6 in 10 minutes and strong which of the following is he best nursing action?
ِ‫أوليسِيُجزىِ ُمحسنٌ ِاالِباحسان‬
A. Gives an analgesic
B. Stop the oxytocin & inform the doctor
C. Gives psychological support
D. Change position to left lateral

269. A 26 year old patient in the gynecology ward complained of pain and swelling in episiotomy stitches on her
second post natal day. On examination, the localized swelling, redness, foul smell and pussy discharge were
identified at the stitches site after examination, the nurse checked her vital signs
BP 116/58 RR 28 HR 132 TEM 39.8
Which of the following should the nurse do prior patient examination by the gynecologist?
A. Arrange dressing instruments
B. Maintain patient privacy
C. Arrange stitch removal kit
D. Collect detailed history

270. A woman delivered her baby boy 3 hours ago with caput succedaneum. Which of the following the nurse
expects to find when she examines the newborn baby caput?
A. Soft, fluctuant mass filed with fluid
B. Bilateral mass on both biparietal boons
C. A mass with clear edges that end at the suture lines
D. A hard mass that is filled with blood

271. A 25 year woman was admitted to medical ward for anorexia chemotherapy the nursing diagnosis was
imbalanced nutrition less than body requirements related to dysfunctional eating pattern which of the following
is the best way to evaluate the outcome of nursing care of this patient?
A. Record daily weight
B. Monitor vital signs accurately
C. Schedule meals with family members
D. Offer small portions of favorites

272. Total ante natal visits ?


10 -12 visit ( every 4 weeks in the first 28 weeks , every 2 weeks in 32-36 weeks , every week from 36weeks -
40weeks )

273. What is the antenatal assessment schedule for women the period of conception and 28 weeks of gestation?
A. Once a week
B. Every 2 week
C. Every 3 weeks
D. Every 4 weeks

274. A nurse is instructing a 20years old nulligravida woman about side effect of oral contraceptive. Which of the
following if stated by the woman indicates for further education?
A. Nausea
B. Headache
C. Weight gain
D. Ovarian cancer

275. A 33 week pregnant mother is on her routine medications that include iron supplement , folic acid ,
multivitamins and calcium supplement , despite all medication her hemoglobin has not been increase since last
two month, she is experiencing more fatigue and lethargy since past few weeks
HB 80 ….. HCT 0.22
Which of the following intervention is the most desired?
ِ‫أوليسِيُجزىِ ُمحسنٌ ِاالِباحسان‬
A. Add vitamin c to increase drug absorption
B. Start injectable multivitamins as per regimen
C. Advise increase intake of organ meat and fortified food
D. Advise increased food intake by frequent small meals and snacks

276. A primigravida attend the antenatal clinic for her routine visit the nurse performed an abdominal palpitation
and found the fundus to be midway between the symphysis pubis and the umbilicus what are the weeks of
gestation according to this findings ?
A. 8 Weeks
B. 16 weeks
C. 24 weeks
D. 28 weeks

277. What is the meaning of the acronym A of REEDA which is used to assess the perineum after birth?
A. The status of edges of the perineal wound it should be closed
B. Presence of discharge from the perineal wound
C. The presence of bruising of the perineal area
D. Excessive swelling of the perineal area

278. A pregnant woman has visited the antenatal clinic her last menstrual date was on 10th August this year. Her
menstrual cycle was regular at 28 day. What is the expected date of delivery?
A. 17th may next year
B. 20st may next year
C. 23rd may next year
D. 25th may next year

279. A 30 years old woman , gravid 3 para 2 in her20 weeks pregnancy was complaining of increase leucorrheoa
during past weeks the nurse reassured the mother and explained the reason of this secretions ?
A. Vaginal infection
B. Expansion of uterus
C. Stimulation of cervix
D. Production of estrogen

280. Lochia red sometimes happens during postpartum to bleed heavily with Foul smell for the first three to ten
postpartum days, the nurse should expect that may to
A. Lochia pink
B. Sign of abnormal hemoglobin
C. Infection
D. Bleeding

281. Which of the following assessment findings indicates laceration of the canal in the fourth stage of labor?
A. Red-brown lochia
B. Firm contracted uterus
C. Fundus is palpated at the level of the umbilicus
D. More than 1 saturated perineal pad per hour

282. Which of the following must be checked during the third day of post-partum period by the nurse?
A. The flow of milk
B. Uterine prolapse
C. Exclusive breast feeding
D. Complete involution of uterus

283. A child with sever bronchial asthma is started on prednisone. Which of the following is a side effect of
prednisone that might be exhibited by the child?
ِ‫أوليسِيُجزىِ ُمحسنٌ ِاالِباحسان‬
A. Anorexia
B. Weight loss
C. Anemia and fever
D. Neurologic symptoms

284. A woman was rushed to the emergency room because if vaginal bleeding mild cramps, tenderness over the
uterus, and closed cervix. After the assessment, it was suspected that the patient may be lead to inevitable
abortion. What is the possible management of this patient?
A. bed rest
B. surgical management
C. induction of oxytocin
D. dilatation and curettage

285. A 28 weeks pregnant mother is in the antenatal clinic with the complaint of vaginal irritation itching and thick
white secretion . she is diagnosed as having vaginal candidiasis she had been having the same infection before
the pregnancy for which she was prescribed the vaginal cream she is using the same cream for the past few days
but her symptoms are not relieved what need to be emphasized to mother
A. change clothes daily use sanitary napkins
B. Avoid self treatment and seek doctor advise
C. Use medicated bubble baths and reduce activates
D. Was with warm water every two hours

286. Which of the following nursing responsibilities should be done immediately following administration of lumbar
epidural anesthesia to a woman in labour?
A. reposition from side to side
B. administer oxygen
C. increase IV fluid as indicated
D. assess for maternal hypotension

287. the antenatal clinic, a 9-month pregnant woman notifies the nurse that e is concerned that she gained 15Kg
during this pregnancy and she is and needs to start a diet program. which of the following is the best nursing
response?
A. This weight gain is normal during pregnancy
B. You can reduce the carbohydrates in your diet
C. Wait till you have the baby and start a diet program
D. This weight is lower than you should gain during pregnancy

288. Intrauterine growth curves were used to classify a 32-week-old preterm new-born. Birth weight and
gestational age shows the infant's growth rate falls below the 10th percentile. What is the priority nursing
diagnosis for a new-born with small for gestational age?
A. Risk for injury related to impaired gluconeogenesis
B. Risk for impaired gas exchange related to meconium aspiration
C. Risk for ineffective thermoregulation related to lack of subcutaneous fat
D. Risk for altered nutrition less than body requirement related to increased metabolic
needs

289. A nurse is leading an educational session on the correct use of oral contraceptives. One of the attendees ask
the nurse what to do if she missed taking an oral contraceptive for one day? Which of the
following should be the nurse advice to her?
A. Continue as usual with no back up contraception
B. Take an active pill immediately and take the next pill at the usual time
C. Take two pills as soon as possible and then one pill daily at the usual time
D. Use back up contraception such as a condom for the next 7 days
ِ‫أوليسِيُجزىِ ُمحسنٌ ِاالِباحسان‬
290. Which of the following represents secondary infertility?
A. Male infertility
B. A couple that does not conceive
C. Infertility that occurs after previous pregnancy
D. Infertility lasts for more than 3 years

291. A women patient with a body mass index of 35 is admitted to ward. Which of the following nursing
diagnosis, related to Imbalance the most appropriate :
A. Activity intolerance
B. Less than body requirement
C. More than body requirement
D. Deficient knowledge on normal nutrition

292. A nurse is providing health education for a primigravida patient who has gestational diabetes mellitus. Which
of the following statements made by the patient indicates the need for additional education?
A. “I should not do exercises”
B. “I should follow the prescribed diet”
C. “+I should monitor my blood glucose”
D. “I should report any sign of infection”

293. Gestational diabetes is high blood sugar (diabetes) that starts or is found during pregnancy. The best time for
glucose screening tests during pregnancy is:
A. 7 – 12 weeks
B. 17 – 18 weeks
C. 24 – 28 weeks
D. 20 – 35 weeks

294. laboring women desires to participate in her cesarean and have pain control. Which of the following
methods would satisfy the women's needs
A. Epidural block
B. Pudendal block
C. Meperidine injection
D. General anesthesia

295. which of the following statement described the latent phase?


A. First 3 cm of cervical dilatation
B. From onset of labour to full dilatation
C. When the cervix is 100% effaced
D. Time of progress from 4 cm to 7 cm

296. A nurse is performing an assessment of a women who is delivery Which assessment finding would indicates
a need to physician?
A. Hemoglobin of 11.0 g/dl
B. White blood cell count of 12,000
C. Fetal heart rate of 180 beats per minute
D. Maternal pulse rate of 85 beats per minute

297. Amniotic fluid importance . surrounds ,cushion ,protect baby . allow the fetus to move freely ,
maintain body temperature to the fetus , and contain urine from the fetus cushion
ِ‫أوليسِيُجزىِ ُمحسنٌ ِاالِباحسان‬
298. A 34-weeks-pregnant mother experiences a sudden gush o from her vagina and mild uterine contractions. She
informs about her condition and requests if she could wait until the delivery. Which of the following is the best
desired response for report to the hospital?
A. Intravenous fluids and medicines need to be administer
B. Observation is necessary to identify premature labor
C. Pain and fluid flow both need to be controlled
D. Fetal heart sound monitoring is necessary

299. During A Vaginal Delivery of Woman 38 Years Old the Nurse Should Consider the Risk of Which of The
Following
A. Acute bleeding and coma
B. Low potassium
C. Brain Injury
D. Fetal heart rate

300. Pregnant woman 34 weeks has hypertension this problem is identified as?
A. Hypertension
B. pregnancy induced DM
C. PIH (Pregnancy induced hypertension)
D. placenta Previa

301. A nurse is explaining to the nursing students working on the antepartum unit how to assess edema. Which
edema assessment score indicates edema of the lower extremities, face, hands, and sacral area?
A. +1
B. +2
C. +3
D. +4

302. 38weeks-pregnant woman complains that she has been craving to eat non food substance What is the term
that describes this pregnant mother’s condition?
A. Pica
B. Bulimia
C. Anorexia
D. Binge eating

303. pregnant client is making her first Antepartum visit. She has a two year old son born at 40 weeks. a 5 year
old daughter born at 38 weeks and 7 year old twin daughters born at 35 weeks. She had a spontaneous abortion
3 years ago at 10 weeks. Using the GTPAL format the nurse should identify that the client is:
A. G4 T3 P2 A1 L4
B. G5 T2 P2 A1 L4
C. G5 T2 P1 A1 L4
D. G4 T3 P1 A1 L4

304. multiparous woman is admitted to postpartum ward after vaginal delivery. Assessment showed, lochia:
steady trickle of bright red blood and ndus: firm.
Blood pressure 110/70 mmHg Heart rate 80
/min Respiratory rate 20 /min Temperature
37.5
Which of the following is the most likely diagnosis?
A. Endometritis
B. Uterine atony
C. Vulvar hematoma
ِ‫أوليسِيُجزىِ ُمحسنٌ ِاالِباحسان‬
D. laceration of the genital tract

305. A 7-year-old insulin dependent diabetic mother has delivered normally in 38 gestational weeks. The nurse
was assessing the insulin requirement this mother after delivery. What is the insulin requirement for this
patient?
A. Higher than before pregnancy
B. No changes in insulin requirement
C. Lower than when she was pregnant
D. Slightly increased than before deliver
306. Mother of nine children, three of them with congenital anomalies and one down syndrome; she is a primary
school graduate, with low financial status. She is not using any method of family planning. Accordingly, the
primary health care nurse has referred her for counseling.
Which of the following is a barrier facing the team responsible for providing health services to handicapped
individuals?
A. Sensory limitations
B. Rigid rules
C. Developmental disabilities
D. Deafness & hearing limitations

307. Mother of nine children, three of them with congenital anomalies and one down syndrome; she is primary
school graduate, with low financial status. She is not using any method of family planning. So, the primary health
care nurse has referred her for counseling. Which of the following is the best health education method that can
be used?
A. community organization
B. individual counseling
C. group discussion
D. health class

308. multiparous patient on day 1 postpartum is asking the nurse to send her baby to the nursery so she can
sleep. What is the most likely phase of psychological adaptation?
A. Taking-in
B. Letting-go
C. Taking-go
D. Letting-in

309. A postdate pregnant woman is admitted for the induction of labour. Her fetal heart rate and vital signs are
within normal range her intravenous line is maintained and she is to be started on low doses of labour inducing
medication. Which of the following medication the mother is likely to receive intravenously ?
A. Oxytocin
B. Cervidil
C. Cytotec
D. Cytoxin

310. A 65-year-old women visited the gynecological outpatient history reveals that she had 3 pregnancies, one
abortion gestational age, had 2 normal deliveries. She smokes 20 Her complaint is that she wets herself when
she cough embarrassing for her? Which of the following can be considered as risk factors pelvic floor muscles?
A. Chronic coughing
B. Diabetes mellitus
C. Excessive spot
D. Sedentary life style
ِ‫أوليسِيُجزىِ ُمحسنٌ ِاالِباحسان‬

311. At is the recommended weight gain during pregnancy of a woman with MI of < 18.5?
A. 12.5 -18 Kg
B. 11.5 -16 Kg
C. 7 -11.5 Kg
D. 5- 9 Kg

312. A 25-year-old mother gravid 2 para q came for a routine check Antenatal Clinic. The nurse assessed fetal
heart rate for the 38 pregnant mother. What is the expected normal fetal heart rate per minute?
A. 90
B. 100
C. 140
D. 170

313. A woman has polycystic ovary syndrome which is 3cm in size what should the nurse do?
A. A repeat the ultrasound after menstruation
B. Remove it by laparoscopy.
C. Give the medication
D. None

314. Nancy, a primipara who is due to deliver next week, calls and tells the nurse she has been having contractions
every four minutes for an hour. Before asking any further questions, the nurse confirms that Nancy knows how
to correctly time contractions, which is
A. from the end of one contraction to the beginning of the next.
B. from the beginning of one contraction to the end of the next.
C. from the end of one contraction to the end of the next.
D. from the beginning of one contraction to the beginning of the next

315. A 22-year-old gravida 2 para 1 with gestational age 38 week admitted to the hospital. The chief complaint is
decreased the fetal non-stress test revealed decreased variability and fetal movement. The next morning as part
of the antenatal the nurse checks the fetal heart rate by Doppler Sonicaid decreased the fetal heart rate to less
than 100 /min. which of the following action the nurse should do first?
A. Reassure the mother that the FHR is Ok
B. Notify immediately the physician or midwife
C. Reposition the patient to left lateral position
D. Ask the mother about the pattern of fetal movement

316. Pregnant patient admission to triage with abdominal pain on assessment the FHR found 70 p/m. what is the
first action for the nurse?
A. Call the doctor
B. Report the finding
C. Put the patient in left lateral position

317. 25 weeks-pregnant, primary gravid woman is on her first antenatal visit, completing physical examination and
history, the midwife found out her husband has sickle cell anemia minor.
What should be the most appropriate plan of care?
A. Genetic counselling
B. Identify severity of disease
C. Discuss the chances of transferring
D. Amniocentesis to identify genetic abnormality
ِ‫أوليسِيُجزىِ ُمحسنٌ ِاالِباحسان‬

318. Which of the following statements describe puerperal infection ?


A. presence of fever of 38 or higher from 2 days to 10 days
B. presence of fever of 40

319. When would the nurse schedule a woman for a glucose tolerance test?
A. 6th -10th week of pregnancy
B. 12th -16th week of pregnancy
C. 24th -28th week of pregnancy
D. 32th -36th week of pregnancy

320. A new-born has a diagnosed Developmental Dysplasia of the Hip (DDH) and is using a Pavlik Harness as
treatment. Which of the following mechanical factors is associated with DDH?
A. Intrauterine breech position
B. Caesarean section
C. Small infant size
D. Single fetus

321. The relative of a 25-year-old post-partum patient stopped her from taking bath until the 40 days provided
with the food by the family that was of saturated fat and a liter of reduced full asking, she said the diet was good
for the milk Which of the following should be the prioritized patient's understanding about?

A. A healthy and balanced diet is beneficial


B. Breast feed is not affected by mother's diet
C. Maintaining personal hygiene is most
D. Rest and relaxation is necessary to regain

322. When is the ideal time to administer analgesia to a women in labour?


A. A soon as she requests analgesia
B. When labour is well established
C. When the women enters into transition phase
D. when the women progress from latent to active phase

323. A patient height is 183 cm and weight is 63.5 kg BMI 19. What is the patient weight classified is ?
A. Underweight
B. Normal
C. Overweight
D. Obese

324. Which of the following would the nurse included in her discharge education for a postpartum patient
regarding signs and symptoms that should be reported immediately?
A. Lochia rubra persists
B. Nipples become red
C. Lochia decreases in amount
D. After pains increase with breast feeding

325. Every how many minutes to check fetal pulse with second stage of labor Every ?
A. 5min
B. 10min
C. 15min
D. 20min
ِ‫أوليسِيُجزىِ ُمحسنٌ ِاالِباحسان‬
326. When the nurse assessed the fundus of a multiparous mother who delivered 2 hours ago, she found the
following Level: 2 cm above the umbilicus. Position: deviated to the right. Consistency: Not well contracted
what is the next nursing action after massaging the fundus until it becomes firm?
A. Assess vital signs
B. Increase V fluids
C. Evacuate the bladder
D. Ask the mother to res

327. A 32 years old multigravida woman presents to the outpatient clinic complaining of dysmenorrhea and
menorrhagia. She had been diagnosed with uterine fibroids and blood studies has been ordered for her. Which
of the following results should the nurse report?
A. hematocrit, 37%
B. hemoglobin, 9 g/dl
C. white blood cell count, 10,000 cells/mm3
D. platelets count, 300,000 platelets/ microliter

Mastitis question
328. Mastitis caused by ...... E.colia Streptococcus, & V aureus

329. A nurse is providing instructions to a mother who has been diagnosed with mastitis. Which of the following
statements if made by the mother indicates a need for further teaching?
A. I need to take antibiotics. And I should begin to feel better in 24-48 hours.
B. I can use analgesics to assist in alleviating some of the discomfort.
C. I need to wear a supportive bra to relieve the discomfort.
D. “I need to stop breastfeeding until this condition resolves.”

330. A primiparous mother who has mastitis is asking the nurse Feeding Which of the following would be the
proper nursing responthe organism of mastitis?
A. It will not affect the newborn
B. It will be deactivated by salivation
C. It will not be expressed in breast milk
D. It will be killed by immunoglobulins in breast milk

331. 41- A nurse is giving health education for a mother who has mastitis. Which of the following if stated by the
mother about what she needs to do, indicate that additional education is needed?
A. Take antibiotics
B. Use analgesics
C. Wear a supportive bra
D. Stop breast-feeding

332. A 23-year-old vaginal delivery primigravida mother was discomfort due to breast engorgement on the
second post The mother complained of pain on the breast site and the able to suck the milk.
Which of the following will relief the mother's discomfort?
A. Breast binder
B. Well-fitting brassiere
C. Encourage breast feeding
D. Lactation suppressing medication

333. The nurse was planning care for a 25-year-old primigravida post-partum mother who had engorgement due
to poor feeding technique. the left breast appeared red and swollen and was diagnosed as Which of the
following is the best education for the mother
A. Avoid wearing brassiere
B. Begin suckling on the right breast
ِ‫أوليسِيُجزىِ ُمحسنٌ ِاالِباحسان‬
C. Stop pumping milk from the left breast
D. Take antibiotics till the soreness subside

334. After 3 days of breast feeding a post partial patient reports nipple soreness. To relieve her discomfort the
nurse should suggest that she:
A. lubricate her nipples with expressed milk before feeding.
B. dry her nipples with soft towel after feeding.
C. Apply warm compresses to her nipples just before feeding.
D. Apply soap directly to her nipples, and then rinse.

335. The nurse is teaching a newly delivered mother about breast feeding which of the following statements
indicates that the mother need further teaching?
A. I will empty my breast and use an alternate breast at
B. I will take a daily shower and clean the nipples with
C. I will let my nipples dry after feeding to prevent nipple tenderness
D. I will always wear a supportive wireless bra

336. Immediately after delivery of the placenta the mother starts bleeding profusely from ?the birth canal. Which
of the following is the nurse's immediate action
A. Administers oxygen by face mask
B. Monitors vital signs continuously
C. Increase the rate of intravenous infusion
D. Rubs the fundus to stimulate contraction

337. postpartum psychological phases sequences?.


A. Taking in , taking hold ,letting go
B. Taking in , taking hold , letting hold
C. Taking hold, taking in , letting hold
D. Letting go, taking hold , taking in

338. Which of the following statement indicate nursing action during the first hour after delivery of the placenta?
A. Monitor of mothers hemoglobin
B. Assess maternal vital signs every 15 minutes
C. Ensure that the mother mobilize and empty her bladder
D. Administer 10 units of oxytocin via IV line to ensure uterus is well contracted.

339. Mastitis is an infection of the breast that occurs most often 2-4 days after childbirth. Which of the following
is considered first line treatment of mastitis?
A. Drainage of breast abscess
B. Antibiotic therapy and cessation of breast feeding
C. Antibiotic therapy and continuation of breast feeding
D. Advice mother to stop breastfeeding until infection is clean

340. A 30-year-oldarimigravida postnatal mother come to the clinic on the She complained of feeling shivering,
redness, swelling and pain in her right Blood pressure 90/62 mmHg Hea 84/min Respiratory rate Temperature
38.8 °C Which is the most common organism that causes mastitis?
A. Pseudomonas
B. Escherichia coli
C. Group B streptococcus
D. Staphylococcus aurous
ِ‫أوليسِيُجزىِ ُمحسنٌ ِاالِباحسان‬
341. The nurse is teaching a 32-week pregnant women how to distinguish between pre-labor (false) contractions
and true labor contractions. Which statement about pre-labor contraction is accurate?
A. They are regular and increase gradually
B. They are felt in the abdomen
C. They start at the back and radiate to the abdomen
D. They become more intense during walking

342. Azithromycin is prescribed for an adolescent female who has pneumonia and recurrent chlamydia . What
information is most important for the nurse to provide the
A. Use two forms of contraception while taking this drug
B. Have partners screened for human immunodeficiency
C. Decrease intake of high-fat foods, caffeine, and alcohol
D. Report a sudden onset arthralgia to the healthcare provider

343. A mother, who is planned for the labour induction, is started on intravenous medication. She is in the first
stage of her labour and is having regular and increasingly stronger uterine contractions. Her cervix is 1 cm dilated
for the past few hours; both the mother and the baby are being monitored.
Which of the following signs should alert the midwife?
A. Baby's head not engaged
B. Decreasing heart rate of the baby
C. Mother's blood pressure 110/60 mmHg
D. Mother's perspiration and increased thirst

344. A nurse is teaching a 26-year-old primigravida who are 33 weeks pregnant on the how to use a kick chart.
Which of the following statements will indicate that she understand the nurse's teaching?
A. Fetal movements must be counted three times per day
B. Fetal movements are felt best when the women is on her right side
C. Fetal movements is a reassuring sign which indicates that the fetushealthy
D. The kick chart is used to record fetal movement for the first time during pregnancy

345. Which test should be performed to screen for cervical neoplasia during antenatal assessment?
A. Papanicolau (PAP)
B. Vaginal rectal culture
C. Rapid plasma regain test (PPR)
D. Venereal disease research laboratory test (VDRL)

346. A woman was diagnosed with gestational trophoblastic disease. What is the lab investigation was done to
diagnose the disease condition?
A. cervical pap smear
B. serum HCG levels
C. serum estrogen levels
D. plasma thyroxine level

347. A postpartum women who was admitted for 24 hours, community nurse visited her after 2 days of
discharge. Which of these are abnormal findings?
ِ‫أوليسِيُجزىِ ُمحسنٌ ِاالِباحسان‬
A. Frequent urination
B. Lochia serosa
C. Uterus below symphysis pupis
D. Breast full of milk .

348. A23-year-old gravid 1 ,para 0 mother is presented to the Antenatal clinic.


The health educator educates group of mothers in the waiting area. Which
information regarding fetal circulation the educator should stress on?
A. One umbilical artery and one umbilical vein
B. One umbilical artery and two umbilical veins
C. Two umbilical arteries and one umbilical vein
D. Two umbilical arteries and two umbilical veins

349. Second day post delivery which blood color will be the lochia
A. Locia rubra
B. Lochia serosa
C. Lochia alpa

350. A 30 year old married woman has dilatation and curettage as a therapeutic abortion. A nurse was preparing
for discharge instruction. Which of the following should the nurse include in discharge instruction?
A. Taking high protein diet
B. Use tampons during swimming
C. Avoid sexual intercourse for two weeks
D. Continue on bedrest for two weeks at home

351. While the nurse is performing postpartum assessment for a primiparous mother delivered 2 hours ago, she
found the following:
Fundus
level: Midway between symphysis pubis and umbilicus
Position: At the mid line
Consistency: Contracted
Lochia: Constantly
Which of the following is the most likely diagnosis?
A. full bladder
B. Perineal hematoma
C. Birth canal laceration
D. Retained placental part

352. What the purpose of a late vaginal_rectal culture during ante natal screening ?
A. To screen for group B streptococci
B. To detect a Chlamydia infection
C. To screen for gonorrhea
D. To screen for syphilis

353. A woman at 24 weeks gestational age has fever body ache and has coughing last 5 days she is sent to hospital
with admission prescriptions for H1N1influenza . which prescription has the highest priority ?
A. Assign private room
B. Vital signs every 4 hours
C. Obtain specimens for culture
D. Ringer lactate IV
ِ‫أوليسِيُجزىِ ُمحسنٌ ِاالِباحسان‬
354. A woman was diagnosed gestational trophoblastics disease what is the lab investigation was done to diagnose
condition ?
A. Cervical pap smear
B. Serum HCG levels
C. Serum estrogen level
D. Plasma thyroxin level

355. A community Nursing nurse is planning to conduct prenatal teaching and community assembly for pregnant
adolescents. Which teaching strategy would be most effective?
A. Offering open sessions for pregnant adolescents and anyone else who wants to attend
B. Designing posters that girls can view individually in community Nursing centre
C. Preparing group class sessions for teaching pregnant adolescents together
D. Conducting one to one teaching sessions for both mothers and daughters

356. A multiparous 28 year old delivered a full term infant weight 3500 grams fetal heart monitoring during the
active phase of the first stage of labor showed variable decelerations. The midwife is inspecting the placenta
immediately following expulsion which type of placenta is most likely ?
A. Normal
B. Circumvallate
C. Succenturiate
D. Velamentous

357. Uterus size after delivery is ?


A. 900 grams
B. 1000 grams
C. 1400grams
D. 1600 grams

358. A pregnant women G1 P0 vaginal delivery observed in the second postpartum day that the perineal pad
saturated with bright red lochia rubra what is the priority nursing intervention?
A. Massage fundus
B. Obtain vital signs
C. Inform physician
D. Inquire about time of pervious saturated perineal pads

359. A 32 weeks pregnant patient present for her follow up appointment in the antenatal clinic. She complains of
experiencing frequent heartburn. Which of the following is the most appropriate advice to the patient?
A. Drink plain water between meals
B. Raise head of the bed
C. Eat favorite foods
D. Lie down for one hour after taking food

360. A midwife visits a mother four weeks after delivery. The mother is breastfeeding her baby but she requests
the midwife to suggest alternative formula milk as she has to return back to her job and her baby will stay in a
day care center. Which of the following teaching plans is suitable for the mother?
A. Supplementary medications with bottle feeding
B. Combined schedule of the breastfeed and top feed
C. Hygiene practices with bottle feeding
D. How to express and save milk
ِ‫أوليسِيُجزىِ ُمحسنٌ ِاالِباحسان‬
361. A multipara mother complained of small vulva with swelling following vaginal delivery of a baby weight 3.8
Kg What is the initial nursing action should the nurse advise the mother to perform?
A. Apply ice pack
B. Maintain bed rest
C. Administer analgesics
D. Encourage fluid intake

362. intrauterine growth curves were used to classify a 32-week-old preterm new-born. Birth weight and
gestational age shows the infant's growth rate falls below the 10th percentile. What is the priority nursing
diagnosis for a new-born with small for gestational age?
A. Risk for injury related to impaired gluconeogenesis
B. Risk for impaired gas exchange related to meconium aspiration
C. Risk for ineffective thermoregulation related to lack of subcutaneous fat
D. Risk for altered nutrition less than body requirement related to increased metabolic
needs

363. A primigravida mother is assisted out of the bed a few normal vaginal delivery. She is taken to the bathroom
cleaning herself and to pass urine. She has difficulty in commode seat and is having no urge to urinate. Which of
the following intervention is the most desired?
A. Teach kegel's exercises
B. Give warm sits bath first
C. Pour warm water over vulva
D. Identify possible perineal tears

364. The nurse is assessing a young-adult pregnant client with no allergies who has tested positive for gonorrhea.
Which of the following medications should the nurse expect to be part of the treatment plan?
A. Tetracycline
B. Ciprofloxacin
C. Azithromycin
D. Ceftriaxone

365. A pregnant mother at early pregnancy was admitted in Emergency Room with leakage of amniotic fluid,
vaginal bleeding and lower dominal cramping pain. What is the possible ?diagnosis should the nurse suspect
A. Missed
B. Inevitable
C. Incomplete
D. Threatened

366. A 20-year-old primigravida, who is pregnant at 40 weeks admitted to labor and delivery unit in active labor.
Vaginal revealed that, the fetal occiput is close to the maternal Which of the following ould the nurse expect to
be increase this fetal position?
A. Leg cramps
B. Back discomfort
C. Vaginal bleeding
D. Nausea and vomiting

367. A mother, who is planned for the labor induction, is started on intravenous medication. She is in the first
stage of her labor and is having regular and increasingly stronger uterine contractions Her cervix is1cm dilated
for the past few hours; both the mother and the baby are being monitored. Which of the following signs should
alert the midwife?
A. Baby's head not engaged
ِ‫أوليسِيُجزىِ ُمحسنٌ ِاالِباحسان‬
B. Decreasing heart rate of the baby
C. Mother's blood pressure 110/60 mmHg
D. Mother's perspiration and increased thirst

368. A nurse in the postnatal ward is assigned for a multiparous patient has just delivered a healthy newborn.
When should the nurse plan to take the patient vital signs?
A. Every hour for the first 2 hours
B. Every 30 minutes during the first hour
C. Every 15 minutes during the first hour
D. Every 5 minutes for the first 30 minutes

369. which valuable information can be obtained from perform abdominal palpation ( lepoids maneuvers ) during
the later pregnancy?
A. Weight of the fetus
B. Gestational age of the fetus
C. Number of fetus in current pregnancy
D. Location and presentation of the fetus

370. A nurse who is caring for a woman in labor, prepares to auscultate fetal heart rate by using Doppler
ultrasound device. How does the nurse determine that the fatel heart sounds are correctly?
A. Notify if the heart rate is greater than 140 BPM
B. Placing the diaphragm of the Doppler on the woman abdomen
C. Palpating the maternal radial pulse while listening to the rate
D. Performing Leopold's manoeuvres first to determine the fetal heart

371. A client who is gravida 1, para 0 is admitted in labor. Her cervix is 100% effaced, and she is
dilated to 3 cm. Her fetus is at +1 station. The nurse is aware that the fetus’ head is:
A. Not yet engaged
B. Entering the pelvic inlet
C. Below the ischial spines
D. Visible at the vaginal opening

372. Less than 24 hours after postpartum calls a nurse in the Mother/Infant Unit. She reports that she has very
heavy bright-red bleeding. being discharged, a mother who is newly What would be the best advice to give her?
A. "Don't worry about it; it is normal."
B. "Call your doctor and ask what to do."
C. "Lie down, massage lower abdomen. If it does not work then come to the hospital
immediately."
D. "Lie down for about an hour. Then check your bleeding, if it is still as heavy, call me
back."

373. A nurse was educating a group of woman on prevention of infection. The nurse asked each woman to state
one preventive of vaginal infection. Which woman needs more education?
A. First woman "keep vaginal area clean & dry"
B. Second woman "wear cotton under wear"
C. Third woman "wipe from front to back after urination or
D. Fourth woman "Do vaginal douche twice a day "

374. 30 years old women absent her menstruation for 5 months and her menstrual cycle come every
28 days, she controlled her diet and do heavy exercise this women condition?
A. Pregnancy
B. Primary amenorrhea
C. Secondary amenorrhea
ِ
‫ِاالِباحسان‬ ٌ‫أوليسِيُجزىِ ُمحسن‬
375. Molar pregnancy: is an abnormal form of pregnancy in which a non-viable fertilized egg implants in the
uterus and will fail to come to term.

376. Molar pregnancy sings ………. Rapid increase of uterine growth

377. A 28-year-old pregnant woman at 9 weeks presents to the o with vaginal bleeding. During assessment, the
nurse found height is 12cm. Which of the following is the most likely diagnosis?
A. Placenta previa
B. Abruptio placenta
C. Ectopic pregnancy
D. Hydatidiform mole

378. During a prenatal examination. The nurse draws blood from a young Rh negative client and explain that an
indirect Coombs test will be performed to predict whether the fetus is at risk for:
A. acute hemolytic disease
B. respiratory distress syndrome
C. Protein metabolic deficiency.
D. pathologic hyperbilirubinemia

379. Which of the following statements will indicate that she understand the nurse's teaching?
A nurse is teaching a 26-year-old primigravida who are 33 weeks pregnant on the how to use a kick
chart.
A. Fetal movements must be counted three times per day
B. Fetal movements are felt best when the women is on her right side
C. Fetal movements is a reassuring sign which indicates that the fetus healthy
D. The kick chart is used to record fetal movement for the first time during pregnancy

380. The antenatal clinic nurse was assessing a 32yaers old gravida 2, para1 pregnant mother on the fundal height
at 36 weeks. What is the expected fundus position?
A. Umbilicus
B. Xiphoid process
C. Symphysis pubis

381. A woman is at 30 weeks gestational age admitted to antenatal with premature rupture of membrane . the
nurse administered Dexamethasone to her according doctor prescription. She ask what is the Drug for. Which of
the following the best answer ?
A. To promote fetal lung maturation
B. Prevention of chorioamnionitis
C. To increase uteroplacental exchange
D. Treatment of fetal respiratory distress

382. A 25 years old patient with history of amenorrhea for two month was admitted for hydatiform mole
investigation. Which signs and symptoms would the nurse observe ?
A. Hypotension
B. Hyperglycemia
C. Rapid uterine growth
D. Painful uterine contraction

383. A women was discharged from gynecological ward after gestational trophoblastic disease (molar pregnancy)
Which of the following is the best advice to give her?
A. Never to fall pregnant again
ِ‫أوليسِيُجزىِ ُمحسنٌ ِاالِباحسان‬
B. To request the doctor to sterilize her
C. To consider having her uterus removed
D. to avoid falling pregnant for at least one year

384. During a vaginal delivery of woman 38 years old the nurse should consider the risk of which of the following?
A. Acute bleeding and coma
B. Low potassium
C. Brain Injury
D. fetal heart rate

385. Pregnant woman signs and symptoms of hypoglycemia


A. Blurred vision
B. Dry mouth
C. Poly urea
D. Flushing

386. postpartum mother is to be discharged on the second day of her forceps delivery. She had sutures on her
vaginal and perineum tears. She is breastfeeding her baby and eating the special food provided by her family.
what discharge teaching needs more emphasis?
A. Diet management and exercise plan
B. Newborn care and vaccination records
C. Hygiene practices and alert signs to report
D. Family planning and child growth monitoring

387. When performing a postpartum assessment on a women the presence of clots in the lochia. The nurse
examines the that they are larger than 1 cm, Which of the following nursing actions is most appropriate?
A. Document the findings
B. Notify the physician
C. Reassess the client in 2 hours
D. Encourage increased intake of fluids

388. Multiparous mother is attending at the outpatient clinic 2 weeks after er delivery for follow up. While the
nurse is assessing the mother, she would not palpate the fundus. Which of the following is the most appropriate
nursing action?
A. Document normal finding
B. Massage the fundus to be firm
C. Assess lochia amount and color
D. Admit the mother to the hospital

389. A pregnant woman of 15 week gestation age and Rh –ve has an abortion admitted in the ward what should
the nurse do for this woman ?
A. Administer Rheum within 72hour
B. Do not give Rheum since it is not used with abortion
C. Do not give Rheum since the pregnancy is more than 12
D. Make certain she received Rheum on her first clinic visit

390. Lochia red fleshy odor mean :


A. Normal
B. Infection
C. Bleeding
ِ‫أوليسِيُجزىِ ُمحسنٌ ِاالِباحسان‬
391. The nurse was educating a postpartum woman during discharge about importance of breast feeding. Which
of the following if said by the women, indicates the need for further education?
A. Breast milk is nutritionally balanced
B. Breast milk reduces the risk of infection
C. Breast feeding promotes mother-child bonding
D. Breast feeding prevents pregnancy

392. Multiparous mother is attending at the outpatient clinic 2 weeks after er delivery for follow up. While the
nurse is assessing the mother, she would not palpate the fundus. Which of the following is the most appropriate
nursing action?
A. Document normal finding
B. Massage the fundus to be firm
C. Assess lochia amount and color
D. Admit the mother to the hospital

393. A women breastfeed her infant one or two hours and her infant cries most of the time and she feels pain in
her breast. Which of the following instructions are appropriate for the nurse to give the mother:
A. Regulate breast feeding every 3 hours
B. That’s normal feeding problem
C. Shift to bottle feeding
D. Start weaning your baby

394. A 17-years-old mother presented to the primary health after delivery. She is suffering from fatigue, anemia,
fever vaginal discharge (see lab results)
81/50 mmHg Blood pressure
98 /min Heart rate
26 /min Respiratory rate
39.6C Temperature
Result Test
4.6 RBC 4.7-6.1 × 1012/L (Male) Normal Values 4.2-5.4 ×1012 /L (Female)
88 Hb 130-170 g/L (Male) 120-160 g/L (Female)
2.50 Calcium 2.15-2.62 mmol/L
Which of the following is considered as the main maternal postpartum haemorrhage complication ?
A. Death
B. Candidacies
C. Cervical cancer
D. Uterine prolapsed

395. A pregnant woman she come to gynocolical word she have vaginal bleeding and no FHS and cervex not
dialted they diagnosis invetable abortion the treatment for this ledy :
A. surgical treatment
B. induction of labor(syntocinon)
C. dialitation and curtage

396. A gravid 8 para 8 women has just delivered a 4.5Kg infant a pregnancy. Which of the following is a possible
complication?
A. Postpartum depression
B. Maternal hypoglycemia
C. Postpartum hemorrhage
D. Pregnancy-induced hypertension
ِ‫أوليسِيُجزىِ ُمحسنٌ ِاالِباحسان‬
397. A primigravida woman who is pregnant at 30 weeks gestation told the nurse that she is worried that anything
happens to her baby. Which of the following should be the proper nurse's response?
A. Ask the woman not to worry
B. Ask the woman to express her concerns
C. Attract the woman's attention to other issue
D. Reassure the woman about the baby condition

398. Which of the following should be included in the nursing care to a woman during 2nd stage of labor?
A. Shave the perineum
B. Administer enema to the woman
C. Careful evaluation of prenatal history
D. Watch breathing, bear down with each contortion

399. Puerperal sepsis symptoms is ?


A. Temperature of 38c or higher 2 – 10days postpartum

400. A 17-year-old mother presented to the primary health center ten after delivery. She is suffering from fatigue,
anemia, fever and vaginal discharge (see lab results)
Blood pressure 80/50 mmHg
Heart rate 112 /min
Respiratory rate 35 /min
Temperature 39.6 C
Test Result Normal Values
RBC 4 4.7-6.1 × 1012 /L (male) 4.2-5.4 × 1012 /L (female)
Hb 90 130-170 g/L 120-160 g/L (female)
HCT 0.29 0.42-0.52 (male) 0.37-0.48 (female)
WBC 12.8 4.5-10.5 × 109/L
Which of the following is the best diagnosis of health problem in this case?
A. Severe urinary track infiction
B. Vesico-vaginal fistula
C. Puerperal sepsis
D. Post-partum haemorrhage

401. What does the relationship of the site of the presenting part to the location on maternal pelvis refer to ?
A. Fetal lie
B. Fetal position
C. Fetal presentation
D. Fetal attitude

402. A 20 weeks pregnant , primary gravid woman visits the antenatal has sickle cell anemia trait and worried this
disease transmitted to her baby which of the following should be initial intervention
A. Plan for the fetal genetic screening
B. Educate mother that her disease is inactive
C. Discuss the chances of genetic disease in the fetus
D. Gather data about the other family members having the disease

403. woman who is 32 weeks gestation. Her weight was 66 kg last month and today it is 78 kg. Which of the
following is the best nursing action?
A. Assess the size of her fetus
B. Give health education good nutrition.
C. Advise her to exercise and lose some weight.
D. Check her blood pressure and test her urine for protein.
ِ‫أوليسِيُجزىِ ُمحسنٌ ِاالِباحسان‬

404. A nurse is caring is caring for a day 1 postpartum patient assessment revealed in addition to red lochia with
fleshy odor
Blood pressure 110/70 temperature 38 respiratory rate 20 heart rate 90
Which of the following would be the proper nursing interpretation of these findings ?
A. Normal
B. Infection
C. Dehydration
D. Hemorrhage

405. A primigravid client who is at 14 weeks gestation has been diagnosed with hyperemesis gravidarum. The nurse
explains to the client that the condition is related to high levels of ?
A. Testosterone
B. Estrogen
C. Aldosterone
D. Progesteron

406. Upon reviewing the pregnant client’s blood test results, the nurse that traces of mercury are present
even after the nurse had healthy dietary modifications. Which action of the client does the nurse discuss to
reduce risk client?
A. Client has five soake d walnuts every day
B. Client consumes king mackerel very often
C. Client eats one medium bowl of flax seeds daily
D. client has cooked soybean seeds as an evening snack

407. A nurse was assessing a newborn delivery by a mother infected with Neisseria gonorrhea. What is the
complication that can occur to the newborn?
A. Deafness
B. Blindness
C. Mental retardants
D. Hyperbilirubinemia

408. A full term mother presents in the antenatal clinic with mild lower abdominal contraction and show watery
discharge for her vagina. She told the midwife that she may be starting her child birth. She is admitted in the
labor and delivery unit, support well and is lying down on the bed.
What knowledge guide is necessary as first intervention ?
A. Assessment of cervix
B. Medication and induction
C. Reassurance and support
D. Mode of delivery

409. A pregnant woman visits the Outpatient clinic complaining of excessive vaginal secretion. Which of the
following is the appropriate nursing assessment?
A. Fetal heart rate
B. Fundal height
C. Signs of infection of labor
D. Fetal presentations and position

410. A pregnant mother with sickle cell anemia is in labor. What should the nurse to ensure the safety of the
infant?
A. Control pain
B. Administer oxygen
C. Monitor fetal heart rate
ِ‫أوليسِيُجزىِ ُمحسنٌ ِاالِباحسان‬
D. Monitor maternal vital signs

411. A nurse is preparing to administer RhoGam to Rh-negative women who has delivered a Rh-positive newborn
. Which of the following is prevented by this intervention ?
A. Maternal illness
B. Neonatal illness
C. Production of antibodies
D. Re- occurrence of Rh positive baby in next mother

412. A 34 years old woman was diagnosed with breast cancer and underwent surgery. She is currently receiving
monthly chemotherapy she telephone the clinic and notifies the nurse that she developed a sore throat. Which
of the following foods would be the most appropriate to recommended?
A. Fresh fruits and vegetables
B. Seafood
C. Dried fruits nuts
D. pasteurized cheese

413. Both parent have sickle anemia disease ?


A. 100%.
B. 50%.
C. 75%.
D. 25%

414. Both parent have sickle anemia trait the percentage for the child to have the disease ?
A. 100%.
B. 50%.
C. 75%.
D. 25%

415. A 7 year old child is assessed with a urinary tract infection BP 95l55 Tem 36.4 . which of the following is the
most appropriate nursing action?
A. Notify the physician immediately
B. Notify the nurse manager immediately
C. Document the findings in the nursing note
D. Put the physician

416. A nurse is caring for child who is post tonsillectomy and adenoidectomy. The nurse should the nurse plan to
assess which of the following complication?
A. Pulmonary hypertension
B. Hemorrhage
C. Hearing loss
D. Orthopnea

417. In the pediatric ward, a newly registered nurse observed that most of the patients have respiratory
problems. Which of the following condition needs priority of care?
A. Pneumonia
B. Epiglottitis

418. An infant was admitted with patent Ductus arteriosus. Indomethacin 0.2mg/kg with prostaglandin inhibitor
via intravenous route was prescribed for him. Which of the following is the function of indomethacin?
A. Assist closure of ductus arteriosus
ِ‫أوليسِيُجزىِ ُمحسنٌ ِاالِباحسان‬
B. Prevent infection
C. Improve tissue perfusion
D. Preserve hormone level

419. A newborn admitted to the NICU with tracheoesophageal fistula (TOF). which of the following nursing
intervention should be included?
A. Elevated the head for feedings
B. Elevating the head but keep the child NPO
C. Insert a nasogastric tube for feeding
D. Encourage the mother to breast feed

420. A 10 year old child admitted to the pediatric ward with rheumatic fever. Which assessment data should the
nurse consider when obtain child’s history to be most significant?
A. Family history of congenital heart disease
B. Recent episode of streptococcal tonsillitis
C. Lack of interest in certain types of food
D. Projectile vomiting and diarrhea for 2 days

421. A nurse is planning to give a 5 year old child an intramuscular injection. Which of the following is the best
nursing intervention to minimize fear?
A. Show a cartoon video injection attention
B. Allow child to give injection to a doll
C. Observe another child receiving injection
D. Get parents to reassure child

422. A full-term newborn admitted to the neonatal intensive care unit with diagnosis of tracheoesophageal
fistula. Which of the following nursing measure to prevent pulmonary complication and improve good
prognosis of the newborn condition?
A. encourage exclusive breast feeding
B. insert nasogastric tube for gastric decompression
C. intermittent suction by double-lumen catheter
D. prepare for the insertion of tracheostomy

423. A four year old boy is admitted with diabetes insipidus. The nurse is assessing him. Which of the following
finding findings should the nurse anticipated.?
A. Bradycardia
B. Low urine output
C. Excessive thirst
D. High systolic blood pressure

424. A midwife is discussing the birth spacing measures with a mother whose first baby boy is a 1 year old and
was born with spina bifida. The midwife has explained the possible causes of condition of the baby and the
measures is necessary if the mother is planned for a next pregnancy?
A. Increase iron and calcium supplements
B. Multivitamin and folic acid intake
C. Genetic screening
D. Immunization before and during pregnancy
ِ‫أوليسِيُجزىِ ُمحسنٌ ِاالِباحسان‬
425. A newborn admitted to the NICU with tracheoesophageal fistula (TOF). which of the following nursing
intervention should be included?
A. Elevated the head for feedings
B. Elevating the head but keep the child NPO
C. Insert a nasogastric tube for feeding
D. Encourage the mother to breast feed

426. You have learned that in babies and children with developmental dysplasia of the hip (DDH), the hip joint has
not formed normally. Which of the following is the most common form of DDH?
A. acetabular dysplasia
B. dislocation
C. Preluxion
D. Subluxion

427. Childlern with cerebral palsy 30 -50% have cognative impairment

428. A 6 years old child diagnosed with Sickle cell anemia is admitted with sever pain in upper and lower
extremities. Which of the following priority should the nurse concern?
A. hydration and pain management
B. nutrition and restricted hydration
C. Nutrition and manage infection
D. Manage pain and infection

429. A 2 years old girl is hospitalized with a diagnosis of bronchiolitis. She is receiving oxygen therapy. Which of
the following would be the best indicator the effectiveness of the of oxygen therapy?
A. Oxygen saturation 84%
B. Oxygen saturation 90%
C. Respiration rate 12bpm
D. Respiration rate 25bpm

430. A nurse is caring a one year old child with congestive heart failure . which nursing goals appropriate?
A. Relieving pain
B. Improving myocardial efficiency
C. Maintaining thermoregulation
D. Promoting oral feeding

431. 10 years old child had and appendectomy yesterday which pain scale should the nurse use to assess the child
pain ?
A. Numeric
B. Faces
C. Flacc
D. Cries

432. A nurse was assessing a newborn undergone circumcision what is the immediate risk for this newborn the
nurse need to observe ?
A. Bleeding
B. Fever
C. Infection
D. Pain
ِ‫أوليسِيُجزىِ ُمحسنٌ ِاالِباحسان‬

433. A child is diagnosed with sickle cell diseases is admitted with severe pain in upper and lower extremities. What
type of crisis is that the child experiences?
A. Sequestration
B. Aplastic
C. Vasoocclusive
D. Hyper haemolytic

434. - A 30-year-old pregnant lady in her 33 weeks pregnancy. When the nurse assesses the health condition of the
lady, she provides nursing care and health teaching. Which of the following types of home visiting is the best to
be conducted?
A. Selective
B. Follow up
C. Fieled trip
D. systemtic

435. Which of the following circumstances is most likely to cause uterine atony and lead to postpartum
hemorrhage?
A. endometriosis
B. urine retention
C. cervical and vaginal tears
D. hypertension

436. Mother of nine children, three of them with congenital anomales down syndrome; she is a primary school
graduate, with low first status. She is not using any method of family planning. According primary health care
nurse has refered her counselling. Which of the following phases of home visit accomplishes intervention?
A. Initial
B. Closing
C. Action
D. Terminal
Rationale .. intervention phase

437. Mother of nine children, three of them with congenital anomalies Down syndrome; she is a primary
school graduate, with low status. She is not using any method of family planning. According primary
health care nurse referred her for counseling Which of the following types of home visits that the
community her nurse should conduct for t client?
A. Systematic routine
B. Selective
C. Follow up
D. Field trip
Rationale Selective Home Visiting Programs. These programs focus on individuals or subgroups of the population
whose risk of developing a disorder are higher than average, as evidenced by biological, psychological or social risk
factors. These families need more sustained or intensive support.

438. Mother of nine children, three of them with congenital anomalies and down syndrome: she is a primary school
graduate, with low financial status. She is not using any method of family planning Accordingly, the primary
health care nurse has referred her for counseling. Which of the following terms describe any restriction or lack of
ability to perform an activity?
A. Impairment
B. Abnormality
C. Handicapped
D. Disability

439. Mother of nine children, three of them with congenital anomalies and one down syndrome; she is primary
school graduate, with low financial status. She is not using any method of family planning. So, the primary health
ِ‫أوليسِيُجزىِ ُمحسنٌ ِاالِباحسان‬
care nurse has referred her for counseling. Which of the following is the best health education method that can
be used ?
A. community organization
B. individual counseling
C. group discussion
D. health class

440. Mother of nine children, three of them with congenital anomalies Down syndrome; she is a primary
school graduate, with low status. She is not using any method of family planning. So, the health care
nurse has referred her for counseling. Which of the following application the counselor can help Mrs.
regarding family planning?
A. Prevention level
B. Gather model
C. Group teaching
D. Rejection of Mrs. M expression
Rationale
When you counsel a new client in your village about family planning, you should follow a step-by-step process.
GATHER is an acronym that will help you remember the 6 basic steps for family planning counselling:-
G Greet the client respectfully.
A Ask them about their family planning needs.
T Tell them about different contraceptive options and methods.
H Help them to make decisions about choices of methods.
E Explain and demonstrate how to use the methods.
R Return/refer; schedule and carry out a return visit and follow up.

441. Mother of nine children, three of them with congenital anomalies and one down syndrome; she is a primary
school graduate, with low financial status. She is not using any method of family planning. Accordingly, the
primary health care nurse has referred her for counseling. Which of the following is a barrier facing the team
responsible for providing health services to handicapped individuals?
A. Sensory limitations
B. Rigid rules
C. Developmental disabilities
D. Deafness & hearing limitations

442. A 9- her of nine children, three of them with congenital anomalies and one n syndrome; she is a primary
school graduate, with low financial us. She is not using any method of family planning. So the primary care nurse
has referred her for counselling. Which of the following must be focused on by the community health care se to
provide an effective health education?
A. Educate regardless realistic objectives
B. Use clear and concise language
C. Use scientific terms during explanation
D. Explain the negative consequences in the family

443. Mother of nine children, three of them with congenital anomalies and one down syndrome, she is a primary
school graduate, with low financial status, she is not using any method of family planning . according, the
primary health are nurse has referred her for counseling. Which of the following is a model concerned with
disability as from of social injustice due to stigma, or discrimination
A. Health belief
B. Biomedical
C. Sociopolitical
D. Economic
444. A pregnant mother is in the active labour. Her cervix is fully effaced dilated with crowing of the fetal head. The
mother is exhausted, perspiring, and is having increasingly painful and severe uterine contractions. Which of the
following intervention is best ?desired by the assisting midwife
ِ‫أوليسِيُجزىِ ُمحسنٌ ِاالِباحسان‬
A. exert pressure on abdomen to aid in child birth
B. increase the rate of intravenous medication
C. wipe face with wet towel and hold hands
D. instruct mother to push harder

445. A 15 month old child is admitted for hypospadias repair. After surgery, which of the following instruction
should be given to the parents?
A. Limit activity for 2 weeks
B. Avoid apply ointment or powder
C. Give a child a diet that is high in protein
D. Isolate the child from other children with the infection

446. A two hour old newborn baby is given to the mother to start on breastfeeding. The mother is reluctant as
the thinks is too early for the baby. The mother finally agrees to breast feed the baby after the midwife has
discussed the advantages of early of breast feeding. Which of the following statement mad by the mother
suggests her correct understanding?
A. Improve natural immunity
B. Strengthens sucking reflex
C. Stimulates gut movement

447. A nurse is preparing a teaching session for elementary school students about the safety measures of
hazards. Which of the following is the most appropriate topics for safety measure for children at this age?
A. Sport activities
B. Riding bicycle
C. Swimming
D. Driving

448. Establishes baby's hydrationA 11 month old boy with cleft lip & palate was discharged from the hospital
after surgery. The nurse provided the parents with teaching. Which of the following statement by the parents
indicate appropriate understanding of the teaching instruction?
A. I should avoid that my infant's cry's as much as possible
B. I will use a spoon to feed my child after the surgery
C. I will put my child in his abdomen after feeding
D. I will use lotion to ease my child's lip irritation

449. A newborn born by elective caesarian section under general anesthesia ter 28 weeks of pregnancy. His
weight is 850 gm, and he is in (20) th percentile in intrauterine growth chart. He is admitted to Neonatal tensive
Care Unit. Which of the following is the classification of this newborn according to stational age and birth weight
using intrauterine growth chart?
A. He is appropriate for gestational age
B. He is extremely low birth weight
C. He is small for gestational age
D. He is very low birth weight.

450. A baby born at 38 weeks of gestation with birth weight 1800 ram. which of the following is the classification of
this infant?
A. Low birth weight
ِ‫أوليسِيُجزىِ ُمحسنٌ ِاالِباحسان‬
B. Very low birth weight
C. Appropriate for gestational age
D. Small for gestational age

451. Which of the following would be the proper interpretation by the nurse for a chromosomal analysis showing
46, XY?
A. Normal male
B. Abnormal male
C. Normal female
D. Abnormal female

452. Which of the following would be the proper interpretation by the nurse for a chromosomal analysis showing
46, XX?
A. Normal male
B. Abnormal male
C. Normal female
D. Abnormal female

453. To measure fatal heat rate fetus daily activity through ?


A. Doppler sound
B. Weighting measures
C. Non-stress test
D. Daily fetus kicks

454. Immediately following the birth of a full-term newborn, which of the following is the priority nursing
diagnosis for this newborn?
A. airway clearance related to nasal & oral secretions
B. ineffective thermoregulation related to environmental factors
C. risk for imbalance fluid volume related t weak sucking reflex
D. risk for injury related to immature defense mechanisms

455. A full term newborn is admitted to NICU with a diagnosis of meningocele. Which of the following admission
assessment is needed?
A. Specific gravity of urine
B. Head circumference
C. Weight and length
D. Palpation of the abdomen

456. A nurse is caring for a full term newborn who was delivered five minutes ago. The infant's Apgar score was 8
at one minutes and five minutes. Which of the following has the highest priority ?
A. Assessing the infant's red reflex
B. Preventing heat loss from the infant
C. Maintaining the infant in the supine position
D. Administering humidified oxygen to the infant

457. A30week gestational preterm admitted to NICU 2hours ago the neonate starts to have grunting ,nasal flaring
which of the following the nurse recognize regarding signs and symptoms?
A. Neonate has RDS
B. It is normally in the first 24 hours of birth
C. This is not significant unless become cyanosis
ِ‫أوليسِيُجزىِ ُمحسنٌ ِاالِباحسان‬
D. Neonate has hypoglycaemia

458. A 30 weeks of gestational age preterm is admitted to the neonatal intensive care unit 2 hours . the neonate
starts to have grunting tachypnea, and nasal flaring. Which of the following is responsible factors for diagnosis of
the premature ?
A. Bronchial spasm
B. Immature bronchioles
C. Lack of surfactant
D. Pulmonary over load

459. A 7 years old child is admitted to pediatric medical. Ward his urine analysis result revealed massive
proteinuria. T he child was diagnosed with Nephrotic Syndrome. Which of the following medication can prevent
the relapses of the child condition?
A. Antibiotics
B. Corticosteroids
C. Diuretics
D. Immunosuppressive

460. An infant was admitted with patent Ductus arteriosus. Indomethacin 0.2mg/kg with prostaglandin inhibitor
via intravenous route was prescribed for him. Which of the following is the function of indomethacin?
A. Assist closure of ductus arteriosus
B. Prevent infection
C. Improve tissue perfusion
D. Preserve hormone level

461. A newborn is diagnosed with Ventricular Septal Defect (VSD) Which of the following information should the
nurse give to the newborn's mother?
A. Cyanosis will occur most of the time during sleeping
B. Breast feeding is not recommended for your child
C. The defect might close spontaneously after 6 months
D. Blood pressure is different on the child's arm and the leg

462. A 10 years old child had an appendectomy yesterday. Which pain scale should the nurse use to assess child
pain?
A. Numeric
B. Faces
C. FLACC
D. Cries

463. Infant mortality is a critical indicator of children health status. Which of the following statements probably
describes the " infant mortality " term?
A. Deaths of infant during the first year of the life per 100 live births
B. Death of infants during the first year of life per 1000 live births
C. Death of infants during the first year of life per 10000 live births
D. Death of infants during the first year of life per 10000 live births

464. A child with rheumatic fever is having sever pain , tenderness and swelling in his left joint what should the
nurse do to alleviate his pain?
ِ‫أوليسِيُجزىِ ُمحسنٌ ِاالِباحسان‬
A. Apply heat on the affected joint
B. Put the joint in flexion position
C. Apply clod compresses to the joints
D. Ask the child to walk for 15 minutes

465. 9 YEAR old child is admitted to the Postoperative Care Unit following in heart transplant. The nurse
administers cyclosporine by intravenous infusion. Fifteen minutes later the child has difficult breathing, his skin
feels cold and clammy and he appears restless. Which of the following is the most appropriate initial nursing
action?
A. Ensure airway patency
B. Administer oxygen therapy
C. Discontinue intravenous infusion
D. Administer intramuscular epinephrine

466. A nurse in the Neonatal Intensive Care Unit is caring for premature newborn, is diagnosed with Respiratory
Distress (RDS) and the doctor ordered administrating surfactant Surfactant should be given by which of the
following routes?
A. Intravenous
B. Subcutaneous
C. Intramuscular
D. Endtracheal

467. A nurse is performing physical examination on the new born she notes that the baby has cephalhetoma this
baby is risk to develop which of the following?
A. Sudden death
B. Pathological jaundice
C. Infected umbilical cord
D. Increased intracranial pressure

468. A 7 year old child is assessed with a urinary tract infection BP 99/55 TEMP 36.4 . which of the following is
the most appropriate nursing action?
A. Notify the physician immediately
B. Notify the nurse manager immediately
C. Document the findings in the nurse note
D. Put the patient in Trendelenburg position

469. In an education session with pregnant mothers, the midwife explains the purpose of amniotic fluid sac or the
water bag. After the completion, the midwife is revising the concepts by asking questions. Which of the
following statement made by the mothers is most appropriate regarding the fluid bag.
A. It provides the baby with immunity against diseases
B. It provides important nutrition and fluid to the baby
C. It protects and maintains baby's temperature
D. It protects the baby from harmful chemicals in mother's blood

470. A23-year-old gravid 1 ,para 0 mother is presented to the Antenatal clinic. The health educator educates group
of mothers in the waiting area. Which information regarding fetal circulation the educator should stress on?
A. One umbilical artery and one umbilical vein
B. One umbilical artery and two umbilical veins
C. Two umbilical arteries and one umbilical vein
D. Two umbilical arteries and two umbilical veins

471. 37 weeks of gestation neonate is admitted to the Neonatal Intensive Care Unit immediate after delivery
because the mother has fever in the last 2 days before delivery. Which of the following is most expected finding
for neonatal sepsis?
ِ‫أوليسِيُجزىِ ُمحسنٌ ِاالِباحسان‬
A. decreased urinary output
B. poor feeding
C. stable body weight
D. sudden hyperthermia

472. Which of organism can cause neonate to develop septicemia including respiratory distress apnea and
hypotension within 12 hours of birth
A. Escherichia coli
B. Cytomegalovirus
C. variclla zoster virus
D. Group B streptococcuss

473. while a nurse is assessing an infant born 11 hours ago caesarean section, she auscultated moist lung sounds.
Which of the following is the most likely interpretation?
A. Abnormal finding
B. Normal finding
C. Pneumothorax
D. Surfactant aspiration

474. Lanugo hair is fine hair in preterm baby less than 30weeks

475. Vernix caseosa, also known as vernix, is the waxy or cheese-like white
substance found coating the skin of newborn human babies. It is produced
by dedicated cells and is thought to have some protective roles during
fetal development and for a few hours after birth.

476. An 11 years old child has been diagnosed with type 1 diabetes mellitus . which of the following education
should the nurse explain about the exercise?
A. Extra insulin is require during exercise
B. Extra snacks are needed before exercise
C. Exercise will increase blood glucose
D. Exercise should be restricted

477. newborn has small, whitish, pinpoint spots over the nose, which the nurse knows are caused by retained
sebaceous secretions. When charting this observation, the nurse identifies it as:
A. Milia
B. Lanugo
C. Whiteheads
D. Mongolian spots

478. A 25 years old primipara is admitted for labor . the infant is delivered by forceps because of breech
presentation and full body assessment shows alrage blue macular marking over left buttocks which o the
following the most likely cause ?
A. Echymosis
B. Nervus flames
C. Telangiectasia nevi
D. Mongolian spots

479. A nurse was observing the stool color for a newborn on the first day after delivery. What is the expected color
of stool for this newborn?
A. Brown
B. Light green
C. Light brown
ِ‫أوليسِيُجزىِ ُمحسنٌ ِاالِباحسان‬
D. Brownish green

480. An infant with developmental dysplasia of the hip is discharging with a Pavlik harness. Which of the following
instructions should the nurse give to the infant's parents
A. Put the baby's diaper over the harness
B. Fasten the straps for at least 2 hours every day
C. Press both legs together before applying the harness
D. Soft, thin clothing should be worn under the harness

481. A neonatal is admitted to the NICU with a meningomyelocele. HR130 .. RR 28….. TEM 36.7 which of the
following action the nurse should perform to prevent infection of the meningomyelocele sac ?
A. Wash the sac with betadine every shift
B. Expose the defect to the room air
C. Apply antibiotic cream every 24 hours
D. Cover he sac with moist sterile saline dressing

482. A baby born at 38 weeks of gestational with birth weight 1800gram. Which of the following is the
classification of this infant?
A. Low birth weight
B. Very low birth weight
C. Appropriate for gestational age
D. Small for gestational age

483. 24 weeks of gestation neonate is admitted to the Neonatal Intensive Unit immediately after delivery with
respirator distress syndrome de 1.
hart rate 140 /min
respiratory rate 77 /min
Temperature 36.5
Which of the following method of feeding is recommended to promote with of this premature?
A. Enteral feeding of breast milk
B. Enteral feeding of premature formula
C. Oral breast feeding
D. Oral premature formula

484. A newborn is delivered by the midwife. The umbilical cord i cut safely by following necessary aseptic
techniques. Furth newborn is to be taken. Which of the following intervention is the most desired?
A. Assess sucking response
B. Increase mother child bonding
C. Assess and record APGAR score
D. Keep dry and maintain thermoregulation

485. A 2-year-old child is admitted to pediatric ward. The mother cannot stay the child and she will visit him in the
weakened only. Which of the following nursing action indicates an understanding of the emotional needs this
child?
A. Give her a warm bath to calm down
B. Allow the child to suck on a pacifier
C. Ask the parents to bring the child favorite toy
D. Tell he parents that frequently visiting is unnecessary

486. When assess newborn wight in scale the nurse must avoid heat loss by;
A. Radiation
ِ‫أوليسِيُجزىِ ُمحسنٌ ِاالِباحسان‬
B. Evaporation
C. Conduction
D. Convection

487. A nurse is caring for a newborn in Well Born Nursery she warps the baby with blanket and ensures the
nursey temperature is suitable for the babies. What type of heat loss is the nurse preventing?
A. Radiation
B. Conduction
C. Convention
D. Evaporation

488. A mother is asking the nurse how is sickle cell could be deducted. Which of the following is at test detected
sickle cell disease of traits?
A. Hemoglobin electrophoresis
B. Bone marrow aspiration
C. Complete blood count
D. Free erythrocyte protoporphyrin

489. An 11-year-old child has been diagnosis with diabetes insipidus?


A. Posterior pituitary
B. Adrenal medulla
C. Anterior pituitary
D. Adrenal cortex

490. A term baby boy has diagnosed with Down syndrome. Physical examination revealed flattened nose, low set
ears, upward slanting eyes, single palmer crease. Which of the following is the most common congenital
anomaly associated with the this disease?
A. Developmental dysplasia of hip (DDH)
B. Congenital heart disease
C. Hypospadias
D. Pyloric stenosis

491. A neonate is admitted to the neonatal care unit with a meningomyelocele HR 130 RR 28 TEM 36.7 which of
the following actions should the nurse perform to prevent infection of the meningomyelocele sac?
A. Wash the sac with betadine every shift
B. Expose the defect the room air
C. Apply antibiotic cream every 24 hours
D. Cover the sac with sterile dressing

492. If full term infant weight 3 kg at birth, approximately should the infant weight be at 12 months old ?
A. 7 kg
B. 9 kg
C. 11kg
D. 13kg

493. A nurse is performing physical examination on the new born she notes that the baby has cephalhetoma this
baby is risk to develop which of the following?
A. Sudden death
B. Pathological jaundice
C. Infected umbilical cord
ِ‫أوليسِيُجزىِ ُمحسنٌ ِاالِباحسان‬
D. Increased intracranial pressure

494. A4 days old baby diagnosed with physiological jaundice . his father is distressed and wants to know why he
have this condition . what the nurse should the nurse tell the about the most prominent physiological jaundice
A. Immature hepatic function
B. Decrease milk intake
C. Rh incompatibility
D. Red blood cell enzyme defects

495. A newborn with hyperbilirubinemia was started on phototherapy What will be the nurse's instruction
regarding feeding?
A. Feed glucose drinks
B. Breastfeed two hourly
C. Bottle feed till the bilirubin level reduce
D. Breastfeed alternatively with bottle feeds
496. Which action a nurse needs to include when caring for a newborn receiving phototherapy?
A. Expose all surfaces
B. Prevent stimulation
C. Cover the eyes with shield
D. Change position every four hourly

497. A baby born at 35-week was admitted in neonatal intensive 27hours ago, physical examination
revealed yellow discoloration sclera and mucus membrane. The result of bilirubin level every 170mol.
The infant was diagnosed with neonatal jaundice physician order to start single phototherapy. Which
of following should the nurse consider as a priority during phototherapy of this newborn?
A. ensure proper fitting of eye covering (patches)
B. monitor bilirubin levels every 48 hours
C. feed the infant formula every 4 to 5 hours
D. avoid removing the infant from phototherapy

498. A nurse prepares to administer a vitamin K injection to a full term the mother wants to know the
importance of the injection Which of the following is the best nurse response to the mother
A. needed for blood clotting to prevent haemorrhage
B. accelerate the growth and development of infants
C. help in maintain healthy gut and passage of meconium
D. protect the infant from developing sever respiratory distress

499. A nurse is caring for child who is pot tonsillectomy and adenoidectomy the nurse should plan to assess which
of the following complication?
A. Pulmonary hypertension
B. Hemorrhage
C. Hearing loss
D. Orthopnea

500. Term baby boy is admitted to Neonatal Intensive Care Unit. Physical examination revealed flattened nose,
low set ears, upward slanting eyes, gle palmer crease. Which of the following is the possible diagnosis of the
new-born?
A. Cushing syndrome
B. Down syndrome
C. Intra-Uterine Growth Retardation
D. Congenital hypothyroidism

501. A nurse performing nursing care plan for a neonate after a birth, which intervention has the highest nursing
priority neonate?
ِ‫أوليسِيُجزىِ ُمحسنٌ ِاالِباحسان‬
A. btained a dextrostix
B. give the initial bath
C. Give the vitamin k injection
D. cover the neonate head with a cap

502. During physical assessment of a male infant genitalia, the nurse found that one of the testes are enlarged.
Which of the following could be the reason for swollen testes?
A. Chordee
B. Cryptorchidism
C. Hydrocele
D. Hypospadias

503. A newborn is diagnosed with hypospadias. When teaching the parents of this child, the nurse should tell them
to avoid which of the following before the hypospadias repair?
A. circumcision
B. drinking acidic juices
C. urinary catheterization
D. riding a bicycle

504. A6-year-old child is diagnosed with sickle. cell disease is admitted with severe pain in upper and lower
extremities. What type of crisis that the child experiences?
A. Sequestration
B. Aplastic Lae.
C. Vasoocclusive
D. Hyperhemolytic

505. Mother of a sickle cell anemia child is asking why her child can't go hiking with his friends.Which of the
following complications hiking can leads to ?
A. Enhance iron absorption
B. Decrease oxygen consumption
C. Inhibit hemoglobin production
D. Precipitate vaso-oclusive crisis

506. The nurse admits a client who is in sickle cell crisis. The nurse should prepare for which intervention as a
priority in the management of the client?
A. Pain management with an opioid
B. Intravenous fluid therapy
C. Oxygen administration
D. Blood transfusion

507. When performing a new-born assessment, the nurse should measure the vital signs in the following
sequence:
A. Pulse, respirations, temperature
B. Temperature, pulse, respirations
C. Respirations, temperature, pulse
D. Respirations, pulse , temperature

508. Immediately following the birth of a full-term newborn, which of the following is the priority nursing diagnosis
for this newborn?
A. airway clearance related to nasal & oral secretions
B. ineffective thermoregulation related to environmental factors
C. risk for imbalance fluid volume related t weak sucking reflex
D. risk for injury related to immature defense mechanisms
ِ‫أوليسِيُجزىِ ُمحسنٌ ِاالِباحسان‬
509. An infant arrives in the Emergency Department not breathing and does not have a pulse. When starting CPR,
which of the following arteries is appropriate to perform the pulse checks?
A. carotid
B. Radial
C. Brachial
D. termporal

510. A nurse is assessing a 2 days old full-term male neon circumcision. She observed that the circumcised area is
re a large amount of fresh blood.
Heart rate 110 /min
Respiratory rate 40 /min
Temperature 36.6 C
Which of the following action should the nurse take?
A. Apply antibiotic ointment on the affected area
B. Give the infant another injection of vitamin K
C. Clean the area with betadine to prevent infection
D. Apply gentle pressure with a sterile gauze

511. A maternity nurse is performing a newborn assessment thirty minutes after delivery of a baby who did not
receive any prenatal care and has an unknown gestational age. The skin is extensively leathery, cracked and dry
and there is an absence of lanugo and vernix. How many weeks ’gestation is this neonate?
A. <30
B. 30-35
C. 36-40
D. >40

512. A nurse is caring for a female newborn who born with an imperforate anus. When assessing the newborns
urine, she should notify the doctor immediately if the newborn's urine contains which of the following?
A. meconium
B. sugar
C. albumin
D. crystals

513. A newborn has diagnosed Development Dysplasia of hip DDH and is using a Pavlik Harness as treatment .
A. Multiple
B. Acetabular dysplasia
C. Subluxation
D. Dislocation

514. Which of the Following Is the Most Commend Site to Obtain a Capillary blood sugar sample from neonate?
A. Earl ape
B. fingertip
C. Heel
D. Abdomen

515. A 6 years old preschool returned from the operating room after tonsillectomy. A nurse received the patient
from recovery room. Which position the nurse place the child post tonsillectomy?
A. Supine
B. Side lying
C. Low fowler
D. High fowler
ِ‫أوليسِيُجزىِ ُمحسنٌ ِاالِباحسان‬
516. female teenager was diagnosed with sickle cell anemia inform a nurse school arranged a hiking activity in
nearby mountains. She shows excitement while informing the nurse about her new water bottle for this trip.
What is the appropriate initial response by the nurse?
A. "what do your parents think?"
B. "you realize that this type of activity might cause you a pain crisis"
C. "you need to get approval from your doctor before attending this activity"
D. ."it is great that you are hydrating yourself in preparation of this activity

517. A child with thalassemia was given desferoxamine (Desferal);which of the following should alert the nurse to
notify the physician?
A. Decreased hearing
B. Hypertension
C. Red urine
D. Vomiting

518. A client is diagnosed as having secondary Cushing’s syndrome. The nurse knows that the client has most likely
been taking which medication:
A. Estrogen
B. Penicillin
C. Lovastatin
D. Prednisone

519. A nurse is planning to give a 5 year old child an intramuscular injection. Which of the following is the best
nursing intervention to minimize fear?
A. Show a cartoon video injection attention
B. Allow child to give injection to a doll
C. Observe another child receiving injection
D. Get parents to reassure child

520. A client who is breastfeeding her newborn requests assistance from the lactation nurse. Which reflex does the
nurse explain in order to assist with latching on?
A. Extrusion reflex
B. Rooting reflex
C. Swallowing reflex
D. Tonic neck reflex

521. A healthy baby is born normally via vaginal delivery and when transferred to newborn until the nurse
administered vitamin K intramuscularly. Which sits recommended for vitamin K injection ?
A. Biceps
B. Deltoid
C. Vastus lateralis
D. Gluteus maximus

522. The nurse determines that a client understands the purpose of a vitamin K injection for her new-born if the
client states that V K is administered for which purpose
A. New born lack vitamin
B. New born have low blood levels
C. New born lack intestinal bacteria
D. New born cannot produce vitamin k in the liver

523. A nurse is administering a shot of vitamin K to 30 days old infant . which of the following target area is the
most appropriate
A. Gluteus maximus
ِ‫أوليسِيُجزىِ ُمحسنٌ ِاالِباحسان‬
B. Gluteus minimus
C. Vastus lateralis
D. Vastus medialis

524. While a nurse is assessing the head of a newborn at the first hour after delivery , she observed a soft edema
over the vertex which crosses the suture line. Which of the following would be the proper nurses
interpretation?
A. Cephalohematoma
B. Hydrocephaly
C. Large head
D. Caput succedaneum

525. A mother asked the nurse that while she was changing the diaper for her female newborn, she noticed a brick
red stain on it. What is the best response by the nurse?
A. It is a sign of low iron excretion
B. It is expected in female newborn
C. It is due to medication given to the mother
D. it due to medication given to the newborn

526. A nurse is assessing a4-month-old formula fed infant .the parent reported that the infant was irritable, crying
excessively, not sleeping well, and vomiting, gastro esophageal reflux is expected. What nursing intervention
should the nurse expect to each parent ?
A. Place in an infant seat after eating
B. Give frequent feedings
C. Position the child in a swing
D. Thin formula with water

527. The normal systolic Bp for new Born:-


A. 40-60
B. 60-80
C. 80-100
D. 100-120

528. Neonatal mortality rate :


A. 1:100
B. 1:1000
C. 1:100000
D. 1:1000000

529. New born stomach capacity


A. 6ml
B. 12ml
C. 28ml.

Age for surgery repair


530. Cleft lip 3 – 6 month after surgery consideration elbow restrain after surgery the infant should not be
allow to cry and the infant not allow to breast feed by sucking . never to put the baby prone position after cleft
lip repair surgery
531. Cleft palate 12 – 18 month position after surgery prone position ,
532. Consideration after cleft lip and cleft palate is :
ِ‫أوليسِيُجزىِ ُمحسنٌ ِاالِباحسان‬
 Burping the child frequently every 15 minutes
 To prevent cleft lip and cleft palate the pregnant woman u should advice:
 Folic acide 400 iu
 Increase green vegetables & Citrus intake
 Feeding up right setting position

Questions
533. A 32year 0i847ytfc10
534. old gravid 1 and para, is now planning to become pregnant within the next year. The patient herself had
born with born with neural tube defects, which were surgically repaired in early childhood. The nurse
recommends that the patient begin taking tablet of folic acid. How many micrograms would be most appropriate
for this patient?
A. 400
B. 600
C. 1800
D. 4000

535. A 2-month-old infant with cleft lip is seen in the primary health care to get the regular vaccine of 2 months.
The mother asked proper time for the corrective cleft lip surgery of her infant. Which of following is the best
nurse response?
A. No specific age for repair of cleft lip
B. It is too late, repair should be done immediate after delivery
C. The age of 2 months old is the time for repair
D. The proper time for repair after the age of one year,

536. A nurse is assigned to care for a child after a cleft palate repair which of the following types of restraints is
very effective for child?
A. Mummy restraint
B. Elbow restraint
C. Wrist restraint
D. Mitt restraint

537. A 4 month old infant returned immediately from OR room post cleft lip repair which of the following nursing
intervention should be considered?
A. Apply elbow restrain
B. Apply suction when needed
C. Measure temperature
D. Put infant in prone position

538. The nurse have been teaching a new mother how to feed was born with a cleft lip and palate before surgical
repair of Which of the following action from the mother indicate teaching has been successful?
A. burping the baby frequently
B. Prevent the infant from crying
C. Placing the baby flat during feeding
D. Keep the infant prone following feedings
ِ‫أوليسِيُجزىِ ُمحسنٌ ِاالِباحسان‬
539. Mother came to the Outpatient Department with an infant having cleft and palate. The infant was
underweight, so the nurse has to consider Teaching the proper way of feeding the child in the treatment plan.
Which of the following is the proper way of feeding
A. Use a non-squeezable bottle during feeding
B. Feed infant in an upright, sitting position
C. Enlarge nipple holes of bottle to allow more milk to pass through
D. Feed infant longer than 45 minutes to allow more food to be small

540. A 40-year-old women is a gravida 2, para 2 and is current conceive. Her previous pregnancy resulted in the
birth of a cleft lip and palate. The patient is anxious and concerned pregnancies and the nurse provides genetic
counselling and Which foods would most effectively prevent recurrence a palate?
A. Green vegetables and citrus fruit
B. Eggs, milk and dairy products
C. Wheat, corn, rice, oats and rye
D. Beef, chicken and yellow vegetables

541. A nurse is caring for a new-born with cleft lip. At which age would the nurse expect the doctor to perform?
A. 3-6 months
B. 6-10 months
C. 11-14 months
D. more than 14 months

542. A 2-day-old new-born is admitted to the nursery. While the nurse is administrating oral feeding, the milk
returns through the child's nose and mouth and the infant become cyanotic. Which of the following condition
the new-born should have?
A. Anorectal malformation
B. Tracheoesophageal fistula
C. Cleft lip and palate
D. Cardiac condition

543. A newborn admitted to the NICU with tracheoesophageal fistula (TOF). which of the following nursing
intervention should be included?
A. Elevated the head for feedings
B. Elevating the head but keep the child NPO
C. Insert a nasogastric tube for feeding
D. Encourage the mother to breast feed

544. A full-term newborn admitted to the neonatal intensive care unit with diagnosis of tracheoesophageal
fistula. Which of the following nursing measure to prevent pulmonary complication and improve good prognosis
of the newborn condition?
A. encourage exclusive breast feeding
B. insert nasogastric tube for gastric decompression
C. intermittent suction by double-lumen catheter
D. prepare for the insertion of tracheostomy

545. A nurse is caring for 14 month old immediately after a surgical repair of cleft palate. In which position of the
following should the nurse put the child?
A. Prone
ِ‫أوليسِيُجزىِ ُمحسنٌ ِاالِباحسان‬
B. LateralSupine

546. A 9-month-old child who has a repair cleft palate the nurse explaining mother on how she will give feeds to
her child. Which of the following instruction can be expected to include feeling education?
A. Open cup
B. Tea spoon
C. Bottle feed
D. Special bottle feed
Spina bifida
547. 26- Which of the following vitamin supplements can decrease the incidence of Neural tube defects such as
anencephaly and spina bifida new-borns ?
A. Vitamin A
B. Riboflavin
C. Folic Acid
D. Vitamin K

548. All of the following are types of spina bifida EXCEPT:


A. Myelomeningocele
B. Hemophilia
C. Meningocele
D. Spina Bifida Occulta

549. While caring for a neonate with a meningococcal, the nurse should avoid positioning the child on the:
A. Abdomen
B. Left side
C. Right side
D. Back

550. infant is born with spinal bifida Which of the following complication is always found in these Infants?
A. Hydrocephalus
B. Craniosynostosis
C. Meningitis
D. Cerebral palsy

551. On the second day of hospitalization for ventriculoperitoneal shunt revision, a child with spina bifida
developed hives, itching and wheezing. The nurse should determine if the patient has been exposed to:
A. Peanuts
B. Strawberries
C. Eggs
D. Latex

552. Infant with spina bifida the nurse should monitor


A. Head circumference
B. Abdomen circumference

553. Position for the baby with spina bifida is :


A. prone position
B. back
C. supine
ِ‫أوليسِيُجزىِ ُمحسنٌ ِاالِباحسان‬
554. A full term infant is admitted to NICU with a diagnosis of Spina vital signs are stable. Which of the following
positions is suitable for this infant?
A. Supine position
B. Semi-fowler position
C. Prone position
D. Sitting position

Hirschsprung’s Disease :

555. Definition :is a congenital anomaly also known as congenital aganglionosis or aganglionic megacolon. The
disease occurs as the result of an absence of ganglion cells in the rectum and other areas of the affected
intestine

556. A 24 hours after delivery , the nurses noted that the newborn failed to pass meconium. This indicates which
of the following condition?
A. GERD
B. Pyloric stenosis
C. Failure to thrive
D. Hirsch sprung disease

557. A nurse is assessing a 6-month-old infant that has retar reduced responsiveness and interaction with the
environment to smile or make eye contact. The nurse notices that the attempt to hold or comfort the crying
infant. What diagnosis should the nurse anticipate?
A. Celiac disease
B. Failure to thrive
C. Cystic fibrosis
D. Growth hormone deficiency

558. A 3 days old newborn is diagnosed with Hirschsprung disease. The nurse is conducting a physical examination.
Which of the following findings will alert the nurse to suspect this disease in the newborn?
A. palpable sausage-shaped mass
B. cyanosis of fingers and toes
C. failure to pass meconium within 24-48 hours of life
D. weight less than expected for height and age

559. A 9 month old child is diagnosed Hischsprung disease scheduled for surgical operation which of the following
should the nurse understand the purpose for surgical intervention?
A. To remove the aganglion portion of the bowel to relieve obstruction
B. To maintain optimum nutritional status growth the intertinal
C. To stimulate intestinal adaptation with internal feeding
D. To minimize complication related to the disease

560. A 9 month old child is diagnosed with Hirschsprung disease he is scheduled for surgical operation which of the
following is the most common complication expected during this age ?
A. Mechanical obstruction
B. Entro colitis
C. Pleural effusion
D. Esophageal Artesia

561. An 8-month-old infant is admitted with Hirschsprung disease. Which of the following would be a significant
finding in this infant?
A. Depressed anterior fontanel
B. Polyuria, hematuria
ِ‫أوليسِيُجزىِ ُمحسنٌ ِاالِباحسان‬
C. Weight gain, edema
D. Failure to thrive, constipation

562. The nurse is assessing a 2 -years-old child with Wilms surgery Which of the following should the nurse avoid?
A. Putting the child in lateral position
B. Palpating the child's abdomen
C. Putting the child in a private room
D. Provide mouth hygiene 30 minutes after meal

Intussusceptions
563. A nurse is taking a history from the infant parents who have a suspected diagnosis of intussusception. Which
of the following assessment question would be most helpful?
A. Do you breast feeding your child ?
B. What does your child's stool look like?
C. How often your child urinating?
D. What is the colour of your child' urine?

564. A 5-month-old boy has been vomiting green coloured vomit He has intermittent abdominal pain during
which he draws his chest, turns pale and cries forcefully. On observation, the in the stool which has a jelly-like
consistency. Abdominal pal a long. tube-like mass. There is no fever, rash nor diarrhea are hyperactive in all
quadrants which is the most likely form of initial treatment?
A. Manual manipulation
B. Surgical resection
C. Normal saline enema
D. Laparoscopy

565. A 7-month-old infant seen in the Emergency Department suffering from episodes of severe abdominal pain,
and the infant's stool became like red jelly. Abdominal examination revealed palpable sausage- shaped mass in
the right upper quadrant. Which of the following is the first line of therapeutic management for this infant?
A. Non-surgical hydrostatic reduction
B. Surgical simple reduction
C. Pyloromyotomy
D. Endorectal pull-Through

566. 7month-old infant seen in the Emergency Department suffering from episodes of severe abdominal pain,
and the infant’s stool become like red jelly. Abdominal examination revealed palpable sausage-shaped mass in
the right upper quadrant which of the following is the best diagnosis?
A. Hirschsprung disease
B. Hypertrophic pyloric stenosis
C. Infant colic
D. Intussuception

567. Nurse is preparing to care for a child with a diagnosis of intussusception. The nurse reviews the child's record
and expects to note which sign of this disorder documented?
A. watery diarrhea
B. rib bone-like stools
C. profuse projectile vomiting
D. Red jelly stool
ِ‫أوليسِيُجزىِ ُمحسنٌ ِاالِباحسان‬

568. A 5year-old child was seen to the Emergency Department abdominal pain, palpable sauge-shaped mass, and
Intussusception is suspected Which of the following is the best diagnostic evaluation to?
A. X-ray
B. endoscopy
C. Rectal biopsy
D. Ultrasonograph
Pyloric stenosis
569. A 5-week-old newborn was admitted to pediatric Ward with pyloric stenosis, the newborn has weight loss,
and projectile vomiting during feeding . They scheduled surgical repair of pyloric stenosis Which of the following
postoperative intervention for this
A. IV fluid infant is retaining adequate amount by mouth
B. Administration of proper analgesia until infant discomfort resolve
C. Start feeding immediately after postoperative
D. Vomiting is uncommon in the first24-48 hrs

570. Surgery for pyloric stenosis movement?


A. Pylorotomy
B. Pylorectomy
C. Pylorostomy
D. Pyloromyotomy

571. PYLORIC stenosis peristalsis movement ?


A. From right to left
B. From left to right

572. 1 month-old infant is admitted to the surgical unit with hypertrophic pyloric stenosis and scheduled for the
surgery. Which of the following is the findings of abdominal examination?
A. palpable olive-like mass in the left side
B. palpable olive-like mass in the right side
C. Palpable olive-like mass moved from left to right
D. Palpable olive-like mass moved from right to left

573. A 7-week-old infant boy is admitted with projectile vomiting decreased urine output, decreased bowel
movements and weight loss. He has poor turgor and appears hungry. The nurse observes left-to right peristaltic
waves after he vomits. The nurse would expect to find which of the following during the physical assessment?
A. Hepato-splenomegaly
B. A palpable pyloric mass
C. Lymphadenopathy
D. Bulging fontanelles

574. The nurse is assessing a child (an infant ) with pyloric stenosis . which of the following is likely to note?
A. Diarrhea
B. Projectile vomiting
C. Swallowing difficulties
D. Currant jelly like stool

575. A 5 week old newborn admitted paediatric ward with pyloric stenosis the newborn has weight loss, and
projectile vomiting after feeing. Which of the following abdominal organs are directly affected when stenosis
diagnosed?
A. Stomach and duodenum
ِ‫أوليسِيُجزىِ ُمحسنٌ ِاالِباحسان‬
B. Stomach and oesophagus
C. Liver and spleen
D. Liver and bile duct

576. Tetralogy of Fallot where is it located :


A. right heart side
B. left heart side

577. The Foley Family is caring for their youngest child, Justin, who is suffering from tetralogy of Fallot. Which of
the following positions is used for congenital heart condition?
A. Semi fowler's position
B. knee chest position
C. prone position

578. Which of the following instruction should be given to the parents about colostomy care?
A. Use baby powder after stoma cleaning
B. Empty the pouch when it is completely full
C. Avoid tight diapers around the infant abdomen.
D. Use baby wipes to clean the skin around the stoma

579. A Community Nursing nurse is working with a family in their home The parents complain that their eight
year-old son is "wild" and that he never listens to them. They become upset at his antics to gain attention from
the nurse and send him out of the room. Which of the following responses from the nurse is the most
appropriate?
A. " That was the right thing, it teaches him who is in charge”
B. " Don't worry, he is a boy and with time, he will grow out of this”
C. " Let's talk about how we can teach your child the right behavior and your expectations
of him”
D. " You are only reinforcing his behavior when you do that. Try putting him in time out
instead”

580. 11- child is admitted to the pediatrics ward with acute lymphocytic LEUKEMIA , and has had joint pain for
several weeks. Physical findings ude widespread ecchymosis, generalized lymphadenopathy,
hepatosplenomegaly, and pallor. Lab work shows a low haemoglobin level, RBC count, low WBC count low
haematocrit , and low platelets. Which high risk is expected?
A. Infection
B. Fractures
C. Dehydration
D. Hepatitis

581. A nurse is teaching a group of new mothers in the post-natal ward on how to manage breast engorgement
after they are discharged to home. Which of the following statements by the mother will indicate to the nurse
that they understood how to prevent engorgement of the breasts?
A. breast feed every 4 hours
B. breast feeding during the day and bottle feeds at night
C. breast feed every 2-3 hours during the day and stop at night
D. breastfeed the baby every 2-3 hours during the day and night
ِ‫أوليسِيُجزىِ ُمحسنٌ ِاالِباحسان‬
582. When would the nurse schedule a woman for a glucose tolerance test?
A. 6-10h week of pregnancy
B. 12th-16h week of pregnancy
C. 24th-28 week of pregnancy
D. 32h-36 week of pregnancy

583. Which of the following is the most common site to obtain a capillary blood sample from Neonates?
A. Heel
B. Earlobe
C. Fingertip

584. mother came to the clinic and afraid that her baby maybe will have meningitis as his brother already have
What should the nurse tell the mother?
A. The vaccine is reducing the risk for this decease

585. Temperature 41 c is ……………Hyperpyrexia

586. When checking rectal temperature how depth thermometer should be inserted
A. 2cm
B. 3.5cm
C. 4cm
D. 4.5

587. Patients with cardiac disease, the nurse should avoided which of the following place when taking temperature
A. Rectal
B. Oral
C. Axillary
D. Tympanic

588. A 15-year-old girl is admitted to the hospital with diarrhea. She has been repeatedly vomiting for now weak
and lethargic. She is oriented to time to questions appropriately. The nurse prepares temperature using an
electronic thermometer. Which measurement would be most appropriate?
A. Oral
B. Rectal
C. Axillary
D. Tympanic

589. Fastest and most sanitary thermometer is :


A. Glass
B. Electronic
C. Chemical
D. Tympanic
ِ‫أوليسِيُجزىِ ُمحسنٌ ِاالِباحسان‬
Vaccination

.
590. Vacciine refrigator temerature is ?
ِ‫أوليسِيُجزىِ ُمحسنٌ ِاالِباحسان‬
A. 0-3
B. 1-5
C. 2-8
D. 3-9

591. How to storage vaccines in ICE

592. Vaccination refrigerator temperature 2-8 c

593. 9 month vaccine :


A. MCV4
B. Hepatitis A
C. Hib
D. MMR
594. 1 year vaccine :
A. Opv
B. Hib
C. DTap
D. Hepatitis A and varicella

595. 18 month vaccine :


A. Rota
B. Hepatitis B
C. Hepatitis A

596. First dose of hepatitis B should be given when ?


A. At birth
B. At 2 month
C. At 6 month
D. At 1 year

597. Measles vaccine administration route


A. Intramuscular
B. Subcutaneous
C. Intradermal
D. Intravenous

598. DTap vaccine administration route


A. Intramuscular
B. Subcutaneous
C. Intradermal
D. Intravenous

599. What of the following disease are prevented by MMR vaccine?


A. Mumps-measles-Scarlet fever
B. Mumps-measles-rotavirus
C. Mumps-measles-rabies
D. Mumps-measles-germen- measles

600. A mother brought her 6-month-old healthy infant to the well-baby clinic Which immunization should the
nurse anticipate to administer as per World Health Organization's recommendation?
A. Varicella (Chicken pox)
B. Rotavirus and hepatitis
ِ‫أوليسِيُجزىِ ُمحسنٌ ِاالِباحسان‬
C. Measles, Mumps, Rubella
D. Diphtheria, Tetanus and pertussis

601. 6month-old boy with hydrocephalus is admitted to the pediatric surgical Ward for ventriculoperitoneal Shunt
(VPS) insertion. Which of the following findings should be of the most concern when assessing the child
postoperative?
A. Sunken fontanelle and irritability
B. decreased head circumference
C. poor feeding and pupillary change
D. headache and excessive sleepiness

602. Which of the following routes is used to administered Diphtheria, Tetanus and pertussis DTP vaccine?
A. Oral
B. Intramuscular
C. Subcutaneous
D. Intradermal

603. The nurse are reviewing the immunized schedule if the 11-month of baby Which of the disease should the
nurse expect the infant that here immunized against?
A. Pertussis, Tetanus, polio, and varicella
B. Polio, Pertussis, Tetanus, and Diphtheria
C. Varicella, polio, Tuberculosis, and pertussis
D. Measles, Mumps. Rubella, and Tuberculosis

604. Dexamethasone ............ accelerates maturation of fetal lung .

605. Hirschsprung disease failure of neonate to pass stool )Meconium ) for 24 hours

606. Apgar score done at 1 minute and 5 minutes at birth

607. most common cause of diarrhoea in paediatric ........................ virus rotavirus ,

608. Meconium is................. Dark green stool in newborn

609. Less than 24 hours after postpartum calls a nurse in the Mother/Infant Unit. She reports that she has very
heavy bright-red bleeding. being discharged, a mother who is newly What would be the best advice to give her?
A. "Don't worry about it; it is normal."
B. "Call your doctor and ask what to do."
C. "Lie down, massage lower abdomen. If it does not work then come to the hospital
immediately."
D. "Lie down for about an hour. Then check your bleeding, if it is still as heavy, call me
back."

610. A 20 weeks pregnant , primary gravid woman visits the antenatal has sickle cell anemia trait and worried this
disease transmitted to her baby which of the following should be initial intervention
A. Plan for the fetal genetic screening
B. Educate mother that her disease is inactive
C. Discuss the chances of genetic disease in the fetus
D. Gather data about the other family members having the disease

611. A pregnant woman of 15 week gestation age and Rh –ve has an abortion admitted in the ward what should
the nurse do for this woman ?
A. Administer Rheum within 72hour
B. Do not give Rheum since it is not used with abortion
ِ‫أوليسِيُجزىِ ُمحسنٌ ِاالِباحسان‬
C. Do not give Rheum since the pregnancy is more than 12
D. Make certain she received Rheum on her first clinic visit

612. A4 days old baby diagnosed with physiological jaundice . his father is distressed and wants to know why he
have this condition . what the nurse should the nurse tell the about the most prominent physiological jaundice
A. Immature hepatic function
B. Decrease milk intake
C. Rh incompatibility
D. Red blood cell enzyme defects

613. In determining the one minute APGAR score of a male infant the nurse assesses a heart rate of 120 beats per
minute and respiratory rate of 44 per minute. He has flaccid muscle tone with slight flexion and resistance to
straightening. He has a loud cry with colour is acrocyanotic What is the APGAR score for the infant
A. 7
B. 8
C. 9
D. 10

614. Five minutes post-birth, a neonate has a heart rate of 98, irregular breathing, actively moves all extremities,
but has bluish hands and feet, as ll as a weak and timid cry. Which is the correct APGAR assessment score?
A. 9
B. 8
C. 7
D. 6

615. A nurse documented assessment on a newborn as listed (see


table) Points Indicator less than 98 /min Heart rate 28 /min irregular Respiratory rate Muscle tone
Minimal flexion of the
extremities Reflex irritability Grimace Body pink, extremities blue Colour
What is the total Apgar Sore?
A. 0
B. 3
C. 5

Paediatrics
616. A 5 years old child brought to emergency room with fractured right forearm he had several bruises on his body
but showed no signs of pain while palpating them. seemed scared and did not answer any questions asked
Which of the following should the nurse do to gain the child's body trust?
A. Arrange meal
B. Comfort measures
C. Show the playing area
D. Therapeutic communication

617. During clinical assessment, the nurse suspected that a child has been psychological abuse. Which statement
by the child is most likely support the suspicion?
A. My parents tell me that iam stupid
B. My parents hurts me to get attention from the doctors
C. My mother forces not give meals on times
D. My uncle show me picture of nude people
ِ‫أوليسِيُجزىِ ُمحسنٌ ِاالِباحسان‬
618. While a nurse is assessing vital signs of newborn infant first hour of delivery. HR 170 RR 70 TEM 36 . the
nurse would interpret these finding as in the discharge instruction
A. Anemia
B. Cold distress .
C. Heart defects
D. Hyperglycemia

619. Toddler is admitted to the pediatric room with several episodes of diarrhea 3 days . the child is diagnosed with
gastroenteritis. Which organism is responsible about the most diarrhea episodes in children?
A. Rota
B. Bacillus magissterium
C. Shigella
D. Staphylococcus

620. A child with a diagnosis of Tetralogy of Fallot is scheduled to be discharge from the hospital. the nurse who is
planning discharge education should instruct the caregivers that during a hyper cyanotic spell, which position is
most likely to benefit the child?
A. Supine
B. Prone.
C. side-lying
D. Knee-chest

621. A 12- year- old boy was brought to the Emergency respiratory arrest due to drowning. Cardiac resuscitation
what is the major complication that might happen if treated after drowning quickly?
A. Sepsis
B. Alkalosis
C. Acidosis
D. Hypothermia

622. which of the following is the leading cause of injury for children who are more than five years old ?
A. accidental suffocation
B. motor vehicle
C. congenital anomalies
D. drowning

623. A 1-year-old girl admitted to podiatric medical unit significant weight loss, diminished mid-arm
circumference diarrhea, and red hair. Which of the following type of malnutrition do the nurse suspect
A. Marasmus
B. Spitting up
C. Kwashiorkor
D. Rickets

624. 33 month-old infant is admitted to the Emergency Department (ED) with fractured arm. The mother
indicated that while the child was crawling fell down the stairs and broke his arm. Which of the following
observation would lead the nurse to suspect that this is a victim of abuse?
A. Age inappropriate injury
B. Pattern and shape of the injury
C. Child is appearing malnourished
D. Improper explanation of the cause of injury
ِ‫أوليسِيُجزىِ ُمحسنٌ ِاالِباحسان‬
625. 5-year-old child was brought to the Emergency Room with a fractured right forearm. He had several bruises
on his body but showed no signs of an while palpating them. He seemed scared and did not answer any
questions asked. Why should the nurse discuss this case with the nurse manager?
A. Continuity of care
B. Rule out child abuse
C. Psychological support
D. Fracture management

626. A four-year old child is seen in the Emergency Department with a spiral fracture of the left arm . The x-ray
examination showed previously broken healed bones. What is your immediate action?
A. Call social services to immediately arrange foster care for the child
B. Ask the child about the previous accidents and management
C. Report the child abuse to the local authorities
D. Try to establish rapport and trust with the child’s family

627. A 62-year-old women admitted to the emergency department for the fourth time this year, each time the
patient comes with severe injuries and bruises in the body. What is your responsibility as a nurse to prevent
such incident to happen again?
A. Reports the assault to the local police and write a report
B. Provides information about safe shelter and support
C. Instructs the women to move away from her home
D. Discharge the patient to a safe shelter

628. 5-year-old child admitted to the pediatric ward with fracture arm. While assessing the child, the se notices a
bruises in the child's back. This is the third time in 1 month that the parent brings the child to hospital. ich time,
the child family provides vague explanations for various buries. Which of the following e pediatric nurse's priority
intervention?
A. Prevent the parents from visiting the child
B. Question the parents about the child's injury
C. Encourage the child to tell the truth about his injury
D. Report suspicion of abuse the proper hospital authorities

629. A child with a ventriculoperitoneal shunt discharge is anticipated after 2 days, the nurse is teaching the
parents about signs of shunt malfunction. Which of the following sings indicate shunt malfunction?
A. Depressed fontanel
B. Vomiting , lethargy
C. Increase heart rate
D. Hematuria

630. A 3 year old child is seen to the emergency department experiencing a seizure at home. Three is no previous
of seizure. The mother inform that she does not believe the child epilepsy. Which of the following is the best
response?
A. No need to worry because epilepsy is easily treated
B. Very few child have actual epilepsy
C. The seizure may or may not epilepsy
D. Your child has had only one seizure

TPN
631. A 16-year-old girl was found unconscious in her home by her mother. She brought to the hospital by
ambulance. The patient has no previously known medical history. On arrival, the nurse performs an assessment
for Level of consciousness and notes a Glasgow Coma Score of five. The nurse notes the patient's breathe smells
like acetone or fingernail paint remover. What is the priority therapeutic goal?
ِ‫أوليسِيُجزىِ ُمحسنٌ ِاالِباحسان‬
A. Increase blood glucose
B. Increase serum osmolarity
C. Increase circulatory volume
D. Decrease intracranial pressure

632. patient has a defect with the beta cells of the Islet of Langerhans. Which of the following should the nurse
most likely expect this patient to exhibit?
A. Anemia
B. Appendicitis
C. Cholelithiasis
D. Hyperglycemia

633. The nurse is planning care for several children who were admitted during the shift. Daily weights should be
the plan of care for the child who is receiving:
A. Total parenteral nutrition (TPN)
B. Supplement oxygen
C. Intravenous anti-ineffective
D. Chest physiotherapy

634. A nurse has just started total parenteral nutrition (TPN) as prescribed
for a patient with severe dysphagia low prealbumin levels. In one to two hours, the nurse should anticipate
assessing the patient’s:
A. Blood glucose level
B. Weight
C. Liver
D. Spo2

635. client with type 1 diabetes mellitus is admitted in which medication if found by the nurse in the should be
clarified with the physician?
A. Humalog (lispro) sliding scale before meals
B. Glargine (Lantus) 10 units subcutaneously
C. Metformin (Glucophage) 500 mg per oral
D. Dextrose 50% ampule intravenous push for 50 mg/dL

636. 28-year-old male is recovering from a moderate concussion following a motor vehicle accident 2 weeks ago,
when he suddenly develops an increased thirst, craving Coldwater. The patient urinates very large amount of
dilute, water like urine with specific gravity of 1.001 to 1.005 the patient is MOST likely developing
A. Diabetic mellitus
B. Diabetic insipidus
C. Hypothyroidism
DIET
637. While planning for discharge education for a mother or rickets, the nurse knows to include the need for an
adequate Which food should the mother choose for her child?
A. Potato and squash
B. Orange and tomatoes
C. Egg yolk and fish
D. Milk and yogurt
ِ‫أوليسِيُجزىِ ُمحسنٌ ِاالِباحسان‬

638. year-old woman with inflammatory bowel disease is scheduled to undergo a procedure in which a stoma will
be formed in the right lower quadrant, five centimeters below the waistline. The nurse advises the patient on
how to avoid potential post-operative intestinal obstruction. Which of the following types of food best
recommended post-operatively?
A. Broccoli and fish
B. Meats and cauliflower
C. Yogurt and parsley
D. Corn and seeds

639. A nurse is providing education to a patient who is being discharged after olecystectomy and has been placed
on a low fat diet. Which of the following foods should be avoided by the patient?
A. Canned beans
B. Whole milk
C. Rice
D. Fish

640. The nurse is caring for a patient who has abdominal pain constipation last three days. The nurse teaches the
patient about the most likely ative-producing foods. What are the foods that are mostly useful to relieve
constipation?
A. Cheese, pasta and eggs
B. Rice, eggs, and lean meat
C. Bran(Oats), figs, and prune
D. Cabbage, bananas and apple

641. Discharge teaching for a child with celiac disease would include instructions about avoiding which of the
following?
A. Rice
B. Milk
C. Wheat
D. Chicken

642. 5year-old child is postoperative after tonsillectomy. The nurse should ask the parents to give the child which
of food after discharge from the hospital?
A. Meat and rice
B. Hot dog and potato chips
C. Mashed potatoes and soup
D. Cucumbers and tomato salad

643. A 40-year-old women is a gravida 2, para 2 and is current conceive. Her previous pregnancy resulted in the
birth of a cleft lip and palate. The patient is anxious and concerned pregnancies and the nurse provides genetic
counselling and Which foods would most effectively prevent recurrence a palate?
A. Green vegetables and citrus fruit
B. Eggs, milk and dairy products
C. Wheat, corn, rice, oats and rye
D. Beef, chicken and yellow vegetables
ِ‫أوليسِيُجزىِ ُمحسنٌ ِاالِباحسان‬
644. A nurse is caring for a patient diagnosed with abdominal x-ray reveals evidence of pancreatic sign of biliary
disease. What would the nurse most likely instruct the patient
A. Consume a high-protein diet
B. Gradually increase dietary fibre
C. Avoid ingestion of alcoholic beverages
D. Limit activities when the patient is fatigued

645. A 6 year old male is diagnosed with Nephrotic syndrome. When nursing care for the patient the plan of diet
should be?
A. high salt, High fat
B. high salt, High cholesterol
C. low salt, low fat
D. low protein, high

646. 5-year-old child was admitted with Nephrotic Syndrome. A nurse noticed that the child has slight facial
puffiness with mild pitting edema on his hands and feet. there was no distended abdomen Which type of diet
the nurse should order for the child?
A. High protein, high salt diet
B. Low protein, low fibre diet
C. Low protein, normal salt diet
D. Normal protein, low salt diet

647. An Indian patient, who is vegetarian, is being discharged from after an elective surgical procedure. The unit
nurse is teaching using visual aids and pictures about food combinations complete protein. Which of the
following food items should the nurse recommended dietary list of the patient?
A. Lentils
B. Potatoes
C. Macaroni
D. Green salad

648. An adolescent with a juvenile diabetes mellitus develops chronic renal failure. Which of the following diets is
suitable?
A. Low fat
B. Low mineral
C. Low protein
D. Low carbohydrate

649. A 60 years old patient was admitted with hepatic coma in the intensive care unit. The physician has ordered
protein restriction diet for the patient. Which of the following substances is most likely causes harmful
effects when the patient increases protein intake?
A. urea
B. creatinine
C. ammonia
D. amino acid
ِ‫أوليسِيُجزىِ ُمحسنٌ ِاالِباحسان‬
650. A 40-year-old women patient with Parkinsonism Medical Ward. The patient stated that she has the past two
weeks. The nurse was planning to Which type of diet is most suitable for parkin?
A. Solid
B. Liquid
C. Semi solid
D. Clear liquid

651. A 68-year-old patient admitted to the Emergency Room with clinical manifestation of pulmonary embolism.
Arterial Blood Gas (ABC) and chest ay were ordered. Which of the following tests is used to diagnose this
condition?
A. Computed Tomography Scan (CT scan)
B. Magnetic Resonance Imaging (MRI)
C. Pulmonary angiography
D. Pulmonary function test

652. 39- A 23-year-old woman presents to the clinic with an intense headache that rates at a level 8 on the 1-10
scale. Additionally, she feels nauseous is sensitive when exposed to bright light. Cerebrospinal fluid samples
sent to the laboratory for analysis and complete blood count. Testing both kerning’s and Brzezinski’s sign are
positive. The patient is admitted the hospital and one hour later she has a seizure. In the period mediately
following the seizure, the nurse stays at the bedside to vide on-going care. Which of the following
assessments has priority at this time?
A. Blood pressure
B. Respiratory drive
C. Pupillary changes
D. Level of consciousness

653. During a night shift a medical doctor complains of back pain and asks the t nurse to give him morphine 5 mg
IM. Which of the following actions indicates professionalism in handing the action by the nurse?
A. Call another doctor to manage
B. Refer him to Emergency Room
C. Administer morphine to doctor
D. Ask him to write a prescription first

654. 3-year-old child with an elevated body temperature is administered oral aspirin . The nurse records the body
temperature of the child two hours and finds that it is still elevated. What is the most likely underlying
physiology for the delayed response in action of the aspirin?
A. High gastric pH
B. Thin epidermis
C. Low muscle tone
D. Short intestines

655. 65-year-old man is undergoing pre-operative preparation for endoscope procedure in which the physician
will visualize the large and distal part of the small bowel with a camera attached to the flexible tube. Which of
the following positions is the most appropriate?
A. Left lateral Sim's
B. Right lateral recumbent
C. Trendelenbreg
D. Prone
ِ‫أوليسِيُجزىِ ُمحسنٌ ِاالِباحسان‬

656. 60-year-old man is being discharged from the post-operative Care Unit following a transurethral resection of
the prostate. The nurse provides discharge information regarding the care of the bladder catheter. Which
method would be most effective in bladder retraining for this patient?
A. Scheduled urination every 2-3 hours
B. Limit fluid intake before sleeping time
C. Perform pelvic floor exercises daily
D. Increase fluid intake during the daytime

657. The unit nurse conducts initial assessment and observes the dressing is wet and requires change. The
patient looks drowsy with warm skin
(see lab results)
Blood pressure 110/70 mmHg Heart rate 99 /min
Respiratory rate 24 /min Temperature 38.8 C
Test Result Normal Values
RBC 5.5 4.7-6.1 × 1012 /L (male) 4.2-5.4 × 1012 /L (female)
Hb 133 130-170 g/L (male) 120-160 g/L (female)
WBC 25.5 4.5-10.5 × 109 /L
What is the most likely underlying diagnosis?
A. Sepsis
B. Gangrenous tissue
C. Acid base imbalance
D. Hyperthermia secondary to infection

658. nurse is completing the preoperative checklist for one of the patients who ring is wearing a ring. What is the
most appropriate action?
A. Give ring to security office
B. Lock ring with patient's valuables
C. Call patient's family to give them the ring
D. Respect patient's choice and leave ring on patient's finger

659. NA 17-year-old women went to the Emergency Department complaining of fever, sore throat, and
hoarseness of voice. Laboratory tests were done (see lab results).

Test Result Normal values


Hb 120.5 120-158 g/L
HCT 0.39 0.36-0.47 g/L
RBC 4.0 3.8-5.1 × 1022/L
WBC 14.2 4-10.5 × 109/L
Throat culture positive Beta Haemolytic Non found
Streptococcus
Which of the following would be the possible complication for the patient condition?
A. Cellulitis
B. Bacterial cholangitis
C. Infective endocarditis
D. Rheumatic heart disease
ِ‫أوليسِيُجزىِ ُمحسنٌ ِاالِباحسان‬
660. A postoperative patient has on order to discontinues the foley 11:30 AM, the nurse removes the Foley
intact. At 1:00 PM, the patient complains of feeling the need to urinate and voids 30 ce. The nurse should assess
the patient for signs of which of the following?
A. Urinary retention
B. Urinary tract infection
C. Cystitis
D. Haematuria

661. A 3-year-old child is admitted to the Medical Ward for vomiting, and dehydration. The nurse sat with the
parents to comply admission interview and wanted to get as much information as Which of the
following communication techniques should the nurse
A. Use of question containing the word "how"
B. Use of question with direct comments to clarify
C. Use of statements that indicate patient will be fine
D. Use of leading question and those involving yes or no

662. An ICU nurse reviews the chart of a 47-year-old man patient mechanical ventilator for a long time. Artorial
blood gas resul see lab results).
Test Result Normal Values
ABG HCO3 24 22-28 mmd/L
ABG PCO3 10.66 4.7-6.0 KPa
PH 7.16 7.36-7.45
ABG PO2 6.13 10.6-14.2 KPa
SA O2 81 95-100 %
What condition the patient is experiencing presently?
A. Metabolic acidosis
B. Metabolic alkalosis
C. Respiratory acidosis
D. Respiratory alkalosis

663. To decrease the anxiety of a 10-year-old girl who is undergoing surgery. Which of the following should the
nurse do?
A. Use a heart model to show her how the surgery will go
B. Provide her with verbal explanation of the upcoming surgery
C. Give her a book to read about the surgery 2 weeks prior
D. Let her parents talk to her about the importance of having surgery

664. patient who is admitted denies any medication allergies medical records state, "No known drug allergies." A
medibeen sent to the pharmacy for gentamicin. The pharmacist record in the pharmacy of an allergy to the
medication. Which of the following is the best way for the nurse to pro
A. Administer the medication with Benadryl (Diphen
B. Notify the doctor of the allergy and request new orders
C. Investigate further then call the doctor for clar
D. Give the medication as ordered by the physician

665. An 81-year-old bed ridden patient in the Geriatric Ward was pyretic through his nasogastric tube. After an
hour, the patie deep sleep, his breathing pattern changed and he did not rep calling his name. Which of the
following should be the immediate nursing intervention
A. Call the physician to examine the patient
B. Check for the correct dosage given
ِ‫أوليسِيُجزىِ ُمحسنٌ ِاالِباحسان‬
C. Check vitals and inform physician
D. Try to wake up the patient

666. At the beginning of the afternoon shifts in the receiving handover from a morning shift nurse post coronary
angioplasty. The nurse waveform on the monitor, upon assessment the carotid pulse (see image). What should
be the most appropriate next action?
A. Give a DC shock
B. Refuse to receive patient
C. Check ventilators settings
D. Shout for help and start chest compression

667. After activating the emergency call system ,what should be the next immediate action ( seeimage ) ?
A. Initiate ventricular pacing
B. Administer a bolus of lidocaine as prescribed
C. Defibrillate the patient
D. Open the patient's airway

668. A nurse enters the room of a patient who is on the patient complains of not feeling well, the (see image).
What is the most likely interpretation?
A. Sinus tachycardia
B. Ventricular fibrillation
C. Normal sinus rhythm
D. Ventricular tachycardia

669. 48-year-old man was admitted to the extensive anterior MI. During the night consciousness. The cardiac
monitor showed VF2 checked carotid and found no pulse. Which is the best emergency management?
A. Cardiac thumb twice
B. Cardioversion, 50 joules
C. Electrical shock, 200 joules
D. Chest compression for five cycles

670. A nurse is caring for a 58-year-old patient


(see lab result).
Test Result Normal Value
Magnesium 2.8 0.7-1.2mmoI/L
Which ECG change is the nurse expected to note?
A. Prolonged QRS
B. Multiple P waves
C. Prominent U waves
D. Depressed ST segment

671. During surgery requiring general anaesthesia, and a carotid pulse is not palpated. How many compressions
per minute should be
A. 50
B. 60
C. 80
D. 100

672. which of the following indication for patient with MI:-


ِ‫أوليسِيُجزىِ ُمحسنٌ ِاالِباحسان‬
A. s t depression
B. St elevation
C. short ST
D. none of the above

673. 40-year-old man is admitted to a Coronary pain. The ECG has normal sinus rhythm with
leads V1-V4.
Blood pressure 123/69 mmHg
What is the most likely nursing diagnosis?
A. Acute chest pain
B. Myocardial in fraction
C. Decreased cardiac output
D. Ineffective tissue perfusion

674. Which of the following is most appropriate to delegate assistance


A. Insertion of an oral airway
B. Changing postoperative dressing
C. Initial interview on a newly admitted patient
D. Repositioning a patient to side-lying position

675. Which of the following patients the nurse should see first?
A. Patient complaining of muscle aches and fever
B. Patient scheduled for electrocardiography
C. Patient newly diagnosed with hypertension
D. Patient with complaint of chest pain

676. The unit nurse conducts initial assessment and observes the dressing is wet and requires change. The patient
looks drowsy with warm skin
(see lab results)
Blood pressure 110/70 mmHg Heart rate 99 /min
Respiratory rate 24 /min Temperature 38.8 C
Test Result Normal Values
RBC 5.5 4.7-6.1 × 1012 /L (male) 4.2-5.4 × 1012 /L (female)
Hb 133 130-170 g/L (male) 120-160 g/L (female)
WBC 25.5 4.5-10.5 × 109 /L
What is the most likely underlying diagnosis?
A. Sepsis
B. Gangrenous tissue
C. Acid base imbalance
D. Hyperthermia secondary to infection

677. A nurse diagnosis a patient with readiness for the This diagnosis is classified within which of the Nursing
diagnoses?
A. Acute nursing diagnoses
B. Risk nursing diagnoses
C. Wellness nursing diagnoses
D. Possible nursing diagnoses
ِ‫أوليسِيُجزىِ ُمحسنٌ ِاالِباحسان‬
678. A man was restless and observed to have an increased physical activity after a heated argument with
another person. After the assessment, the following data was gathered: agitated, diaphoretic, distorted What is
the patient's level of anxiety?
A. Mild
B. Panic
C. Severe
D. Moderate

679. A 70-year-old man underwent a colectomy two days prior. In the past 24 hours, his blood pressure has
decreased and heart rate risen. He now weight two kilograms more than he did prior to admission. Urine output
from the indwelling catheter is 30 ml per hour and he has pitting + bilaterally in the lower extremities. The
abdomen appears distended and is firm to the touch.
Blood pressure 112/62 mmHg
Heart rate 110 /min
Respiratory rate 20 /min
Which of the following is the best nursing intervention?
A. High flow oxygen by mask
B. Position in Trendelenburg
C. Hemoglobin and hematocrit

680. 9-year-old woman patient in the Medical Ward is in a semiconscious .Her pancreatic cancer is metastasized
to her liver and lungs and she admitted for supportive treatment. Her physicians discussed with the ily that she
will not be given the Cardiopulmonary Resuscitation to save life if she goes into the cardiac arrest. Her two sons
agreed but the daughter is indecisive. Which of the following is the critical thinking behind not providing
discounted pulmonary resuscitation?
A. Unilateral judgment of health professionals
B. Refusal of patient’s right to treatment
C. Ethical dilemma and indecisiveness
D. Mercy killing to ease suffering

681. 30-year-old construction worker was stung by roof structure. He then began to complain of feel
An ambulance was called and on arrival, they four the ground and having breathing difficulties. His
The radial pulse is weak and thread.
Blood pressure 92/52 mmHg
Heart rate 118 /min
Respiratory rate 26 /min
Which white blood cell is most likely be elevated?
A. Eosinophils
B. Basophils
C. Monocytes
D. Neutrophils

682. The nurse is preparing to transfer a 55-year-old patient on a the CT-scan unit. The patient is too heavy for the
nurse to transport, nurse went to get an assistive device to transfer the patient. Which of the following transfer
device is the most appropriate for the nurse to use?
A. Board
B. Handle
C. Trapeze
D. Mechanical lift
ِ‫أوليسِيُجزىِ ُمحسنٌ ِاالِباحسان‬

683. A newly RN nurse is about to remove a nasogastric tube to a client with Guillan Barre Syndrome. To
determine the proper Time of removing the tube, the nurse should do it when
A. when the client feeling hungry
B. when the client want to remove it
C. when the client totally conscious
D. when the client exit flatulence or gases

684. which type of study design provides the strongest evidence?


A. Qualitative study
B. Randomized control trial
C. Systematic review of descriptive studies
D. Systematic review of correlational studies

685. 28-year-old women is accompanied by her mother to the ward. She is admitted for her affective disorders
with anger recent events and stammering spells. She has panic followed by apathy and listless Ness. How
should the nurse manage the patient?
A. Be supportive and use therapeutic communication
B. Improvise sign language to control forgetfulness
C. Maintain calm environment and avoid argument
D. Anger management and speech therapy

686. A nurse accidently dropped a medication ampoule, informed the charge nurse, and completed an incident
report form. The charge nurse arranges medication replacement. Which of the following is the immediate
nursing action required?
A. Revise protocol for medication related incidences
B. Allocate senior nurses to medication assignments
C. Provide missed medication dosage to patient first
D. Incident reporting must be given the priority

687. 42-year-old woman is brought to the Emergency severe chest pain radiating to the left neck physician
instructs nurses to start the Morphine Aspirin (MONA) protocol. What is the top priority of nursing care?
A. Administer oxygen according to the physician
B. Administer aspirin according to the physician
C. Administer morphine according to the physician
D. Administer nitro-glycerine according to the physician

688. Home Health Care Department is newly established ar a hospital. At launch, nurses asked the charge nurse
about work at this department Which of the following is the first step of home health care?
A. Referral
B. Planning
C. Home visit
D. Assessment

689. The nurse is assessing a child who has Tetralogy of Fallot observed that the child is having clubbing in his
fingernails Which of the flowing might be the reason for this clubbing?
A. Prolonged tissue hypoxia
B. Delayed physical growth
ِ‫أوليسِيُجزىِ ُمحسنٌ ِاالِباحسان‬
C. Inactive bone marrow
D. Pulmonary fibrosis

690. A 59-year-old woman is admitted in the Medical ward for pyomyositis in her arms and legs. The muscular
strength Progressively decreased within one year. She has irregular breathing pattern and has difficulty in
swallowing. Which of the following nursing diagnosis requires?
A. Risk of choking due to disturbed swallowing
B. Weakness and fatigue due to lower muscle
C. Disturbed breathing due to chest muscle
D. Disturbed activities of daily living

691. A 9- her of nine children, three of them with congenital anomalies and one n syndrome; she is a primary
school graduate, with low financial us. She is not using any method of family planning. So the primary care nurse
has referred her for counselling. Which of the following must be focused on by the community health care se to
provide an effective health education?
A. Educate regardless realistic objectives
B. Use clear and concise language
C. Use scientific terms during explanation
D. Explain the negative consequences in the family

692. high school girl, who has fears of being obese thin, visited the primary healthcare centre with her The
mother reports that her daughter refuses to with the family, and often pretends being a sleep Which of the
following disorders best describe girl?
A. Bulimia
B. Obesity
C. Substance Abuse
D. Anorexia Nervosa

693. An 12-year-old child has been diagnosed with Diabetes insipidus which of the following is disorder?
A. Posterior pituitary
B. Adrenal medulla
C. Anterior pituitary
D. Adrenal cortex

694. the summer months, a five year-old girl presents with a sore throat and a dry cough that has slowly become
worse over the past three weeks. Her body temperature is 38.0oC. On auscultation, there is wheezing and
shortness of breath. She lives in an overcrowded house with three brothers, parents and grandparents in a low-
income neighbourhood where she attends school. Which is the greatest risk factor?
A. Residing in a low-income neighbourhood
B. Exposure to a pathogen in summer season
C. Attending school
D. Living in crowded conditions

695. A patient returns to a clinic for a follow-up visit and is diagnosed as positive for Human Immunodeficiency
Virus (HIV). The patient expresses fear related to lack of finances, fear of social avoidance, and hopelessness.
Which of the following nursing action should
A. Referral to a physician infectious disease specialist
B. Referral to a community-based HIV clinic
ِ‫أوليسِيُجزىِ ُمحسنٌ ِاالِباحسان‬
C. Referral to the local public health department
D. Recommendation to disclose the diagnosis to family

696. While taking the history from a new patient, the nurse densified that he had hypomanic episode which was
alternating with a mirror depressive\episode for the last two years. what is the most likely diagnosis?
A. Bipolar I disorder
B. Bipolar II disorder
C. Dysthymic disorder
D. Cyclothymic disorder

697. A 62-year-old women admitted to the emergency department for the fourth time this year, each time the
patient comes with sever injuries and bruises in the body. What is your responsibility as a nurse to prevent such
incident to happen again?
A. Reports the assault to the local police and write a report
B. Provides information about safe shelter and support
C. Instructs the women to move away from her home
D. Discharge the patient to a safe shelter

698. Five years old patient was brought to emergency room with several bruises on his body but showed
fractured right forearm. He had no signs of pain while palpating them. He seemed scared and didn't answer any
questions asked. Why should the nurse discuss this manager with the nurse?
A. care continuity
B. abuse role out
C. support psychological
D. management fracture

699. A nurse calls together an interdisciplinary team with members from medicine, social service, the clergy, and
nutritional services to care for a patient with a terminal illness. Which of the following types of care would the
team most likely be providing?
A. Palliative
B. curative
C. respite
D. preventive

700. A nurse is assigned to care for a Muslim female emergency caesarean section. The patient asks the to wear
a head scarf during the surgery. Which of the following is the most appropriate?
A. Ask the patient to remove the scarf
B. Tell the patient that it is ok to wear the scarf
C. Ask the patient to wear a surgical cap
D. Tell the patient that a request will be during the surger

701. A nurse is trying to secure a peripheral V access in a traumatic patient who is deteriorating. The nurse has
two failed attempts What should the nurse do next?
A. Activate code
B. Use Interosseous needle
C. Consider arterial line
D. Call another nurse who is more exprienced
ِ‫أوليسِيُجزىِ ُمحسنٌ ِاالِباحسان‬
702. A 70-year-old women with a history of hypertension companies of lack of appetite due to bloating and
constipation. The abdomen is distended and examination shows a positive fluid wave. palpation of the abdomen
confirms guarding and tenderness over the right upper quadrant lower eg edema 3+ and pitting bilaterally are
also present. She has kilograms since her last appointment three weeks before4.5 164/92 mmHg Blood pressure
90 /min Heart rate
26 /min Respiratory rate
37.1C Temperature
92% on room air
Oxygen saturation
What is the most likely underlying health problem?
A. Hepatic congestion
B. Pulmonary hypertension
C. Left-sided ventricle failure
D. Splenomegaly

703. A 70-year-old man presents to the clinic with difficulty sleeping at night. He has not had a good night's rest
for several months and feels exhausted. He needs to place three pillows behind his back in order to sleep.
Examination of the lungs reveals crackles and wheels Auscultation of the heart confirms an S gallop. Which of
the following is the most likely underlying health problem?
A. Asthma
B. Pulmonary stenosis
C. Right-sided heart failure
D. Left-sided heart failure

704. During a CPR of an admitted patient in cardiac arrest, a family member tasks the unit nurse to be at the
bedside and say final words to their loved one. The nurse explains to the family member that the scene is very
disturbing and the medical team is doing its job. the family member still insists in witnessing the resuscitation
efforts What would be the most appropriate action by the unit nurse?
A. Allow family member to be at bedside
B. Wait and ask permission from team leader
C. Call security to escort family member out
D. Refuse because there is not enough space in the room

705. A 30-year-old women was admitted with ectopic pregnancy on the sixth gestational week. The patient was
scheduled for resection of the involved fallopian tube with end to end anastomosis Which is the initial nursing
diagnosis for this patient?
A. Grieving
B. Acute pain
C. Hyperthermia
D. Knowledge deficit

706. What is the most appropriate blood product for a patient with haemorrhage due to upper GI bleeding?
A. Plasma
B. Whole blood
C. packed red cell
D. Serum albumin
ِ‫أوليسِيُجزىِ ُمحسنٌ ِاالِباحسان‬
707. A new community nurse in the rural clinics has to have good background about the health status of the
community in order to assess their needs What is the appropriate way to assess the health status of the
community?
A. Home visit
B. Community assembly
C. Mass information campaign
D. Community health assessments

708. A 75 year-old bedridden patient is hospital Medical ward for the treatment of her in Area. Her wound is
infected with multiple Longer than normal time to treat. She Crying spells and is talking about death Which of
the following nursing diagnosis must be
A. Health and wellness
B. Coping mechanism
C. Self-perception
D. Belief system

709. a nurse is caring for a child with traction of fractured bone. In the chart, a doctor has placed a reminder to
maintain even and constant traction What would be the most likely understanding of the nurse for this order?
A. Add or remove weights every other day
B. Allow weights to hang free continuously
C. Elevate head and foot of the bed alternatively when in pain
D. Allow weights to hang free every 12 hours to achieve good circulation

710. A patient complains of pain when standing upright hump on the upper back. In the past year slightly shorter.
The doctor has suggested tests to What is the best appropriate intervention?
A. Instruct the patient in the use of prescribed magnesium supplements
B. Have the patient sleep propped on two pain
C. Prepare the patient for a CT scan of both hip
D. Instruct the patient in the use of vitamins
711. A 56-year-old patient was diagnosed with instructions about administration of insulin Regular Insulin and
Neutral Protamine Hagedo subcutaneously every morning? Which of the following is right period for(in minutes)
after administration?
A. 10-15
B. 30-60
C. 60-90
D. 240-360

712. In a post-operative patient in the Surgical Unit most indicative of a developing complication? ,
A. Increasing alertness
B. Weak and rapid pulse
C. Negative Homans' sign
D. Minimal bowel sounds in four quadrants

713. A mother in the postnatal Ward is being provided with the her family and she has been instructed to be on
the bed members believe that the diet with the specific home increase the milk production and will promote her
healing Which of the following intervention is the most desired?
A. Respect the cultural believes and encourage activities
B. Teach and monitor for correct breast-feeding practices
ِ‫أوليسِيُجزىِ ُمحسنٌ ِاالِباحسان‬
C. Discuss dietary plan and monitor weight gain
D. Explain post-partum care and follow ups

714. A child was admitted to the hospital three hours ago with a injury. The child responds appropriately, but
sluggishly to drifts in and out of sleep Which of the following best describes this patient's level of?
A. Lethargic
B. Obtunded
C. Comatose
D. Semi comatose

715. A nurse is planning to discharge a known HIV, a the Isolation Unit after the recovery from upper, Which of
the following nursing problem requires?
A. Risk of infection due to altered immune
B. Fluid volume deficit due to frequent diarrhoea
C. Anxiety due to disease, fear and social
D. Weight loss due to higher metabolism rate

716. An elderly patient who has an aortic aneurysm Intensive Care Unit to a Medical Surgical Unit on day. While
assessing the client, a nurse notes extremities and is unable to palpate the pedal pulse Which intervention
should the nurse implement?
A. Wrap the lower extremities with warm
B. Use a Doppler ultrasound to reassess the
C. Elevate the extremities above heart level
D. Place a bed cradle over the bed to levitate

717. A 60-year-old man client had a permanent complains of chest pain and dyspnea with rapid feels suffocated
and appears restless
100/70 mmHg Blood pressure
96 /min Heart rate
32 /min Respiratory rate
37.2C Temperature
What is the immediate nursing intervention?
A. Monitor and report findings of chest
B. Chest X-ray to identify dislocation of
C. Manage pain with medication as ordered
D. Administer oxygen as ordered

718. A nurse is preparing a sterile field prior the Which of the following procedures illustrates the nurse to the
principles of aseptic technique?
A. Touch the outer 2-inch border of the sterile gloves
B. Place the sterile linen-wrapped package on waist level
C. Hold the bottle lip 1-2 inches above sterile pouring the solution
D. Open sterile item while holding outside hand then throw object onto sterile field

719. Which of the following is the appropriate nursing advice to mother complaining of epigastric burning
sensation and do use of drugs?
A. Chewing gums
B. Increase fluids at bedtime
ِ‫أوليسِيُجزىِ ُمحسنٌ ِاالِباحسان‬
C. Drink orange juice on getting up
D. Eat small meals every 2 to 3 hours

720. A nurse is caring for a 72-year-old man patient , is unsteady. The patient requests the nurse to help nurse
asked the patient to wait few minutes device to transfer him. Which of the following transfer devices is the
nurse to use?
A. Belt
B. Board
C. Handle
D. Mechanical lift

721. 62 year-old female patient is admitted for her left eye cataract removal. She has cataract in both eyes with
no visibility in her left sys. She is scheduled for the surgery .What essential safety precautions must be assessed
preoperatively?
A. Assess and document visual acuity of both eyes
B. Knowledge to prevent eye injury after Surgery
C. Cleanliness of eye and removal of eyeglasses

722. A nurse is providing postpartum care for a GSP4 mother who had a rapid labor of an infant weighing 4000
gm. Assessment revealed a boggy uterus, heavy lochia and stable vital signs. After fundal massage and bladder
evacuation. the fundus remains soft. which of the following is the most appropriate next nursing action?
A. Inform the physician ..
B. Reassess the vital signs
C. Continue fundal massage
D. Take venous blood sample

723. A 46-year-old man is presented to Emergency Room with severe chest and unstable angina. The ECG of the
patient shows ST segment pression on his 12-lead. e rhythm strip example of specific ECG changes (see image).
Which of the following cases the nurse determines ST segment depression?
A. Injury
B. Necrosis
C. Ischemia
D. Nothing significant
724. A 35 year-old man with pulmonary edema is admitted to the Intensive re unit. He is on a Mechanical
Ventilator(MV) without sedation. After the client is placed on the MV, the nurse diagnosed the case as impaired
communication related to intubation and MV. What is the best method for the nurse to communicate with the
patient?
A. Use normal verbal communication
B. Communicate with the patient after MV removal
C. Use non-verbal communication, such as head-nodding and writing

725. The nurse observes the client csf ,Which of the following of csf color indication for patient with bacterial
meningitis?
A. Cloudy
B. Clear
C. Red
D. Brown
ِ‫أوليسِيُجزىِ ُمحسنٌ ِاالِباحسان‬
726. child has a third- degree burns of the hand, chest, face, which nursing diagnosing takes priority?
A. Ineffective airway clearance
B. disturbed body image
C. impaired urinary eliminations
D. risk for infection

727. A woman has polycystic ovary syndrome which is 3cm in size what should the nurse do?
A. A repeat the ultrasound after menstruation
B. Remove it by laparoscopy.
C. Give the medication
D. None

728. A home health nurse has entered a home to complete an admission assessment on a patient who has a
methicillin-resistant Staphylococcus aureus (MRSA) urinary tract infection. The patient will receive intravenous
anti-infective via a peripherally inserted central catheter (PICC) for 3 weeks. Which of the following actions
should the nurse take FIRST?
A. Shake the patient’s hand
B. Place the nursing supply bagon a clean, dry surface.
C. Obtain the patient’s written consent for home health care
D. Perform hand hygiene per the agency protocol

729. Which of the following patient is the most appropriate to delegate to a new nurse?
A. Patient on a strict bed rest
B. Patient for discharge teaching
C. Patient scheduled for an operation
D. Patient who is hooked to mechanical ventilation

730. An 8-year-old girl is seen in the Emergency Department because of a fracture in the right fibula and she has a
fiberglass cast on her leg instead of a plaster cast. Which of the following is an advantage to use a fiberglass
cast?
A. Cheaper
B. Dries rapidly
C. Smooth external
D. Shape closely to body parts

731. 62 year-old female patient is admitted for her left eye cataract removal. She has cataract in both eyes with
no visibility in her left sys. She is scheduled for the surgery .What essential safety precautions must be assessed
preoperatively?
A. Assess and document visual acuity of both eyes
B. Knowledge to prevent eye injury after Surgery
C. Cleanliness of eye and removal of eyeglasses

732. A newly graduated nurse is inserting an intravenous cannula into the mid-cephalic vein of a patient who is
being admitted to the unit. As she withdraws the needle, a nurse calls out for help from another patient’s room.
The new nurse rushes to help. She secured the intravenous catheter and threw the needle into the waste basket
instead of the sharp’s container. When asked who had thrown the needle into the bin, the new nurse admits
that she had made that mistake. Which professional act best describes the newly graduated nurse’s response?
A. Responsibility
ِ‫أوليسِيُجزىِ ُمحسنٌ ِاالِباحسان‬
B. Accountability
C. Assertiveness
D. Leadership

733. A 50-year-old man presents to the Emergency Department after having suffered a fall while practicing
mountain biking. He appears confused and disoriented. His friend reports that he had been thrown over the
handle bars and hit his head against a rock
Blood pressure 108/66 mmHg
Heart rate 102 /min
Respiratory rate 22 /min
Temperature 37.2 C
Oxygen saturation 99% room air
Which initial intervention is the most important?
A. Intravenous infusion
B. Oxygen administration
C. cervical immobilization
D. Trendelenburg position

734. A 24-weeks-pregnant mother is in the Antenatal Clinic. She is upset and crying about having her third baby
girl as her family wants a boy. She is requesting an abortion immediately before her family finds out about the
sex of the baby. Which of the following is the most appropriate plan for the mother?
A. Calm her down and reassure for an appropriate solution
B. Provide moral support and book her for procedure
C. Repeat ultrasound and wait for a few more weeks
D. Family counselling and follow religious guidance

735. A post-operative nurse assesses the newly formed stoma and skin of a patient who is one day post-
operative following a proctocolectomy with the formation of a permanent ileostomy. Which of the following
clinical findings would necessities immediate care
A. Bright red and moist stoma
B. Dark red and purple skin
C. Bloody liquid in pouch
D. Ulcerations with a rash

736. A 45-year-old man is admitted to the Surgical Ward for stoma care. The stoma appears a dark red to purple
colour, is slightly swollen and leaks a scant amount blood. There is no presence of stool around or on the site.
Which observation requires immediate attention?
A. Colour
B. Edema
C. Absence of stool

737. A nurse is taking care of a patient who underwent abdominal surgery three days ago. The patient has not
been able to breathe deeply and uses to get out of bed since the surgery due to pain. Also, the patient nplains of
shortness of breath, and the lung sounds are diminished on auscultation.
Blood pressure 120/70 mmHg
Heart rate 75 /min
Respiratory rate 22 /min
Temperature 36.4 ℃
Oxygen saturation 89%
ِ‫أوليسِيُجزىِ ُمحسنٌ ِاالِباحسان‬
Which of the following conditions should the nurse suspect?
A. Sepsis
B. Atelectasis
C. Congestive heart failure
D. Emphysema

738. What do the standards of pain management dictate nurse do?


A. Avoid the use of the word "pain"
B. Screen for pain at each encounter
C. Discourage around-the-clock dosage of analgesics
D. Administer analgesics via injection whenever possible

739. A nurse is planning post-operative teaching for a patient undergone left partial mastectomy. On the third
post-operative day, the nurse teaches the patient to elevate the arm above the heart several times per day.
What does this exercise reduce?
A. Pain
B. Infection
C. Hematoma
D. Lymphedema

740. conscious victim of a motor vehicle accident arrives at the emergency department. The patient is Gasping for
air, is extremely anxious, and has a deviated trachea. What diagnosis should the nurse Anticipate?
A. pleural effusion.
B. tension pneumothorax
C. pneumothorax
D. cardiac tamponed

741. A nurse calls together an interdisciplinary team with members from medicine, social service, the clergy, and
nutritional services to care for a patient with a terminal illness. Which of the following types of care would the
team most likely be providing?
A. Palliative
B. curative
C. respite
D. preventive

742. The nurse is assigned to care for several patients, who should be assessed first by the nurse?
A. A diabetic patient who will be discharged
B. A patient with rheumatoid arthritis who has
C. An asthmatic patient who is due for
D. A patient with surgical incision who require

743. While administering medications, the nurse gives an anti-hypertension medication to the wrong patient (see
table)
A Document the entire scenario
B Document vital signs
C Notify doctor
D Notify doctor if blood pressure is below 90/50 mmH
ِ‫أوليسِيُجزىِ ُمحسنٌ ِاالِباحسان‬
E Induce vomiting
F Administer antidote
G Monitor the patient
What sequence should the nurse follow?
A. A, D, F, G
B. B, C, G, A
C. B, D, E, F
D. B, E, F, G

744. 9-year-old child is admitted to paediatric word diagnosed with Glomerulonephritis (AGN). The mother ask
the nurse why you the blood pressure frequently which of the following the nurse should be respond based on
knowledge.
A. Acute hypertension must be anticipated and identified
B. Hypotension leading to sudden shock
C. Blood pressure fluctuations are a common side effect of therapy
D. Blood pressure fluctuation are a sign that the condition has become chronic

745. Which of the following terms applis to the tiny. blanched. slightly raised end arterioles found on the face.
neck. arms. and chest during pregnancy?
A. Epulis
B. Linea nigra
C. Striae gravidarum
D. Telangiectasia
746. A patient teath brush &bathing shoud be write in:
A. daily care activity
B. medication chart

747. Doctor during new born assessment he toush the bed by the sterile gloves the nurse shoud:-
A. let him complet the cvl then take to him
B. stop him and tell him remove the gloves and give new gloves

748. During skin assessment of a patient the nurse observed a skin described as
round solid mass beneath the skin (see image).
Which of the following terms is best to describe the lesion?
A. Cyst
B. Papule
C. Pastule
D. Blister

749. A patient who has pain localized in the chest, now down the left arm. What is this type of pain called?
A. Phantom
B. Regimented
C. Shooting
D. Radiating

750. The nurse is preparing a patient for an insertion of a central venous pressure line into a neck vein .The nurse
should place the patient into which position?
A. Side-lying position
B. Fowler's position
C. Trendelenburg position
ِ‫أوليسِيُجزىِ ُمحسنٌ ِاالِباحسان‬
D. Semi-Fowler's position

751. Neonate is admitted to the neonatal intensive care unit for observation with a diagnosis of probable
meconium aspiration syndrome (MAS) the neonate weight 4.650 grams and is at 40 weeks gestation which of
the following would be the priority problem
A. Impaired skin integrity
B. Hyperglycaemia
C. Risk for impaired parent infant child attachment
D. Impaired gas exchange

752. Which action a nurse needs to include when caring for a newborn ceiving phototherapy?
A. Expose all surfaces
B. Prevent stimulation
C. Cover the eyes with shield
D. Change position every four hourly

753. Patient with ongoing potassium infusion he has burning sensation ant the site of infusion what should the
nurse do first
A. Stop infusion
B. Document as normal finding
C. Slow rate and call the doctor
D. Inform the doctor

754. Treating pregnant women diagnosed with syphilis is considered


A. Primary level of prevention
B. Secondary level of prevention
C. Tertiary level of prevention
D. Primary and secondary

755. A women breastfeed her infant one or two hours and her infant cries most of the time and she feels pain in
her breast. Which of the following instructions are appropriate for the nurse to give the mother:
A. Regulate breast feeding every 3 hours
B. That’s normal feeding problem
C. Shift to bottle feeding
D. Start weaning your baby

756. A 7-year-old child is admitted to the Emergency Department, injury The child is oriented to the place, person
and time spontaneously obeys commands. The nurse is doing aped Coma Scale (PGCS). Which of the following
score the nurse should record?
A. 3
B. 8
C. 12
D. 15

757. Brachial artery t check pulse in infants

758. Prolactin Hormone promote Milk production


759. TB test Mantoux / PPD test result within 48-72Hours accurate test is sputum culture

760. Umbilical Cord one big vein& Two small arteries

761. Pulse deficit is the different between apical beats and brachial pulse
ِ‫أوليسِيُجزىِ ُمحسنٌ ِاالِباحسان‬
762. Rligion that prohibits blood transfusion Jehovah

763. Spina pifida position Prone position

764. Sound lung in heart failure .cracks

765. Postion after hemorroidectomy ?...........................antriolatral postion


The left anterolateral position is an excellent alternative for hemorrhoidectomy under local anesthesia. The pain
over the pubis and the low backache, frequently encountered in the prone position, are totally eliminated. Exposure
of the operative fields is excellent.

766. Post Vitrect-omy position …Prone

767. Ear Drops in infant down and back , in Adult up and back

768. Ear drops in adult up and back , in infant down and bak
769. Hyperpyrexia above 40C

770. Alzheimer deficiency in serotonin

771. Abdominal assessment. 1. Inspection 2. Auscultation 3. Percussion 4.palpation........ very important


772. During new born assessment the nurse found extra finger in the new born hand Polydactyl

773. During new born assessment the nurse found two digits are fused together
Syndactyly

774. when we should stop magnesium sulphate ( Respiratory Depression, Hypotension


, Bradycardia deep tendon Reflex )
775. Antidote of magnesium sulphate is ( calcium gluconate )

776. Morphine antidote is naloxone / narcan

777. Hydrocephalus signs ( Bulging anterior fontanel & increase head circumference )

778. Naloxone - Digioxin should not be given if heart rate less than 60 or bradycardia

779. OPV oral polio vaccine ..... mixed of live attenuated polovirus

780. Postpartum perineal assessment acronym is REEDA ( Redness-Edema-Ecchymosis-Discharge-Approximation

781. When taking a BP, select the appropriate cuff size; a cuff that is too small will yield a falsely high reading, and a
cuff that is too large will yield a falsely low one.

782. Complication of diabetic mother hypoglycaemic and macrocosmic


ِ‫أوليسِيُجزىِ ُمحسنٌ ِاالِباحسان‬

783. OPV vaccine is live attenuated virus (poliovirus) vacine

784. Patient with BP230/180 is consider Hypertensive Crisis

785. A newly assigned nurse to the home health care department was conducting pre-entry phase of home visit.
Which of the following activities should the nurse do
A. Call physician for a referral order
B. Collect information and schedule visit
C. Establish mutually acceptable goals for care
D. Conduct a health history and physical assessment

786. A mother brought her 6-month-old healthy infant to the well-baby clinic which immunization should the
nurse anticipate to administer as per WHO recommendation?
A. Varicella (chicken pox)
B. Rotavirus and hepatitis
C. Measles , Mumps , Rubella
D. Diphtheria , Tetanus , pertussis

787. The nurse is teaching a 32-week pregnant women how to distinguish between pre labor (false ) contraction
and true labor contractions. Which statement about pre labor contractions accurate ?
A. they are regular and increase gradually
B. they are felt in the abdomen
C. they start at the back and radiate to the abdomen
D. they become more intense during walking

788. A 62-year-old women admitted to the emergency department for the fourth time this year, each time the
patient comes with sever injuries and bruises in the body. What is your responsibility as a nurse to prevent such
incident to happen again?
A. Reports the assault to the local police and write a report
B. Provides information about safe shelter and support
C. Instructs the women to move away from her home
D. Discharge the patient to a safe shelter

789. A 3 years old child is admitted to the hospital with seizures. He was alert, oriented and has a rash in his
extremities and is diagnosed with meningitis. While doing physical examination of him, he starts to develop
seizures.
Blood pressure 100/57 mmHg
Heart rate 110/min
Respiratory rate 30/min Temperature 39.5 C
Which of the following vaccine is used to prevent meningitis?
A. hib vaccine
B. varicella vaccine
C. BCG vaccine
D. Rubella vaccine
ِ‫أوليسِيُجزىِ ُمحسنٌ ِاالِباحسان‬
790. After receiving multiple mosquito bites and experiencing flu-like symptoms, a patient consults a nurse at a
clinic and asks whether an appointment to see a health-care provider is necessary. which of the following
statements should be the basis for the nurse'sresponse
A. antiviral medications can be prescribed to destroy virus
B. Clinical signs can be mild flu-like symptoms to fatal encephalitis
C. If patient has West Nile virus, symptoms will progressively worsen
D. If patient used insect repellent, virus would have been destroyed when mosquito made skin
contact

791. A male client has received a prescription for orlistat for weight and nutrition management. In addition to the
medication, client states plans to take a multivitamin. what teaching should a nurse provide?
A. Be sure to take the multivitamin and the medication at leasr two hours apart
B. As a nutritional supplement, orlistat contains all the recommended daily vitamins and
minerals
C. Multivitamins are contraindicated during treatment with weightcontrol medications such as
orlistat
D. Following a well-balanced diet is a much healthier approach to good nutrition than
depending on a multivitamin

792. Which of the following variables is considered as a major factor that affects community health?
A. Personal behavioral choices include socio-cultural factors
B. Number of health care providers in hospitals within the community
C. Quality of the public safety officers includes police officers, firefighters
D. The number of recreational services in the community

793. During meningitis outbreak in one of the hospitals, the nun-immune staff were given an immunoglobulin in
order to prevent them from the infection. What is the type of immunity that will be developed by these staff?
A. active
B. long-term
C. natural
D. passive

794. The Surgical unit nursing staff were instructed to follow the hospital protocol by reporting cases of any
patient's illnesses that are developed during their hospitalization especially the post-surgical wound infectors
Why must the nurses implement this protocol?
A. Control unnecessary hospitalization
B. Assess hospital infection rate
C. Report nosocomial infection
D. Improve reporting system

795. Which of the following statements indicate that a nursing staff need an education session related to
confidentiality and information security?
A. giving the password to patient’s family member.
B. leaving the computer in the nursing station secured.
C. leaving the personal electronic password safe.
D. allowing nursing students to read medical record to their patients

796. Which of the following is the best action to effectively increase surgery capacity?
ِ‫أوليسِيُجزىِ ُمحسنٌ ِاالِباحسان‬
A. Adapt outpatient departments for inpatients
B. Bring the injured people into the green zone
C. Provide care for the injured people with the available staff
D. Use the available supplies as it is difficult to arrange for additional

797. A patient has been experiencing recurrence of this media antibiotic therapy and the patient was scheduled
for preparative teaching, the patient asked the nurse about the purpose of the procedure.
What is the best answer that the nurse should provide?
A. Stimulates motion of the ossicles
B. Detects permanent hearing loss
C. Allows drainage of purulent fluid from the middle ear
D. Enables medication administration directly to the affected ear

798. After accessing patients' medical records, which behaviour nurse shows that patients confidentiality has
been breached?
A. Reviews patients medical record
B. Read patients care plan
C. Disclosing patients information
D. Documents medication administered

799. A nurse is assessing a 2 days old full-term male neon circumcision. She observed that the circumcised area is
re a large amount of fresh blood.
Heart rate 110 /min
Respiratory rate 40 /min
Temperature 36.6 C
Which of the following action should the nurse take?
A. Apply antibiotic ointment on the affected area
B. Give the infant another injection of vitamin K
C. Clean the area with betadine to prevent infection
D. Apply gentle pressure with a sterile gauze

800. Nurse manager prepares unit clinical operational plan What is top priority in the plan?
A. Infection control
B. Staff orientation
C. Quality projects
D. Safe patient care

801. While taking the history from a new patient, the nurse densified that he had hypomanic episode which was
alternating with a mirror depressive\episode for the last two years. what is the most likely diagnosis?
A. Bipolar I disorder
B. Bipolar II disorder
C. Dysthymic disorder
D. Cyclothymic disorder

802. A 65-year-old women visited the gynecological outpatient history reveals that she had 3 pregnancies, one
abortion gestational age, had 2 normal deliveries. She smokes 20 Her complaint is that she wets herself when
she cough embarrassing for her. Which of the following can be considered as risk factors pelvic floor muscles?
A. Chronic coughing
B. Diabetes mellitus
C. Excessive spot
ِ‫أوليسِيُجزىِ ُمحسنٌ ِاالِباحسان‬
D. Sedentary life style

803. A nurse is assigned to care for a Muslim female emergency caesarean section. The patient asks the to wear
a head scarf during the surgery. Which of the following is the most appropriate?
A. Ask the patient to remove the scarf
B. Tell the patient that it is ok to wear the scarf
C. Ask the patient to wear a surgical cap
D. Tell the patient that a request will be during the surgery

804. A gravid 8 para 8 women has just delivered a 4.5Kg infant a pregnancy. Which of the following is a possible
complication?
A. Postpartum depression
B. Maternal hypoglycemia
C. Postpartum hemorrhage
D. Pregnancy-induced hypertension

805. 62 year-old male patient in the Cardiac Surgery Ward is admitted for the coronary artery bypass surgery. he
has unstable angina for thepast one year and is apprehensive about the open-heart surgery .What essential
assessment needs immediate attention ?
A. Fear and anxiety regarding procedure
B. Need for spirometer and breathing exercise

806. A patient was on a regular does of lithium carbonate. The nurse that he has hand tremor, polyuria, diarrhea
and vomiting. What immediate action should be taken by the nurse?
A. Diuretics
B. Withholding lithium
C. Calling the psychiatrist
D. Monitoring serum lithium level

807. which of the following condition is a Contraindication for a woman oral Contraceptives?
A. Dysmenorrhea
B. Menorrhagia
C. Thrombophlebitis
D. Toxic shock syndrome

808. A 55 year-old man takes cyclosporine mg by mouth twice per day. He had a heart transplant two months
ago. He complains of stomach cramping, diarrhea and muscle twitches. Which electrolyte is most likely
elevated?
A. Calcium
B. Sodium
C. Magnesium
D. Potassium

809. A conscious victim of a motor vehicle accident arrives at the emergency department. The patient is
Gasping for air, is extremely anxious, and has a deviated trachea. What diagnosis should the nurse
Anticipate?
A. pleural effusion.
B. tension pneumothorax
C. pneumothorax
ِ‫أوليسِيُجزىِ ُمحسنٌ ِاالِباحسان‬
D. cardiac tamponed

810. Normal systolic Bp for new born:-


A. 40-60
B. 60-80
C. 80-100
D. 100-120

811. The doctor toush the bed by the sterile gloves the nurse should:
A. Ask him to change the gloves and to give him new one
B. Let him to complete the Central line procedure

812. MR ahmed admitted to ICU with congestive heart disease his vital signs BP 110/60 HR 120 . stroke volume
80 the nurse expected cardiac output to be
A. 180
B. 1700
C. 2400
D. 9600 cardiac output = stroke volume x Heart rate

813. A nurse is leading an educational session on the correct use of oral contraceptives. One of the attendees ask
the nurse what to do if she missed taking an oral contraceptive for one day? Which of the following should be
the nurse advice to her?
A. Continue as usual with no back up contraception
B. Take an active pill immediately and take the next pill at the usual time
C. Take two pills as soon as possible and then one pill daily at the usual time
D. Use back up contraception such as a condom for the next 7 day

814. Mother came with her baby with thalassemia the nurse will monitor Iron level and chelation therapy with
deferasirox or deferoxamine to prevent organ damage

815. A woman is 5 weeks gestation diagnosed with hyperemesis gravidarum what is the most important nursing
action?
A. sychological support
B. Folic acid supplementationr
C. strict intake and output record
D. providing the woman with a high protein diet

816. A nurse is preparing to administer 25 mg iron dextran inject patient with iron deficiency anemia .the nurse
knows this d to subcutaneous tissue and wants to administer the drug safely which of the best administration
techniques ?
A. Z-track
B. deep im
C. c. use large gauge
D. insert needle at 45 angle

817. A surgical nurse is assigned to care for a woman who had undergone vaginal hysterectomy. The nurse
performed assessment. Which of the following potential problem that the nurse should observe?

A. Pain
B. Fever
C. Anxiety
D. Bladder dysfunction
ِ‫أوليسِيُجزىِ ُمحسنٌ ِاالِباحسان‬
818. A 45 year-old man who is hospitalized feels tge constant need to keep things in order , particularly whilst
eating m the nurse observe him arranging the food on his plate into symmetrical and equal bite sized pieces , he
constantly worries that the food served could be outdate and potentially causes illness
Which nursing diagnosis is most important
A. Ineffective verbal communication
B. Self-esteem disturbance
C. Impaired social interaction
D. Anxiety

819. What do the standards of pain management dictate nurse do?


A. Avoid the use of the word "pain"
B. Screen for pain at each encounter
C. Discourage around-the-clock dosage of analgesics
D. Administer analgesics via injection whenever possible

820. 50year-old male was diagnosed with subdural underwent burr hole craniotomy for subdural hernatoma days
ago. In order to detect the sign of meningitis as one of which of the following indicates the patient has
meningitis ?
A. Negative Kernig's signs
B. Positive Brudzinski's sign
C. Absence of nuchal rigidity
D. Glasgow comma scale of 14 points

821. 10 years old boy diagnosis appendicitis has serve pain, the tool of pain scale to be use is:
A. Face pain rating
B. Flacc rating scale

822. A patient diagnosed with septic shock had an upward-trending glucose level (180-225 mg/dL) requiring
control with insulin. the patient's spouse asks why is insulin needed as the patient does not have dia Which of
the following is the most appropriate nursing response to educate the patient's spouse?

A. "It is common for critically clients to develop type diabetes. We give insulin to keep
glucose level under control (less than140 mg/dL)"
B. " Patient had diabetes before, you just didn't know it. we give insulin to keep glucose level
in the normal range (70-110 mg/dL)
C. " Increase in glucose is a normal response to stress by the body. We give insulin to keep
the level 1t 140-180-mg/dL"
D. " Increase is common in critically ill clients and effects their ability to fight off infection.
We give insulin to keep the glucose level in the normal range (70-110 mg/dL)"

823. 33-year old man presents to the emergency department with high grade fever, tachycardia, and tachypnea.
What is an appropriate nursing intervention for the patient’s fever?
A. provide dry clothing
B. keep limbs close to the body
C. cover the patient’s scalp with a cap
D. measure the patient’s fluid intake and output

824. A 78-year-old alert and oriented patient is returning from the radiology Department and the nurse is
preparing to transfer the patient from the wheelchair back into the bed. The nurse places the belt on the
Patient and prepares to lift the patient from the chair Which body mechanics would be most appropriate?
A. Widen leg stance
ِ‫أوليسِيُجزىِ ُمحسنٌ ِاالِباحسان‬
B. Bend over to lift
C. Rotate from the waist
D. Maintain base of gravity in the feet

825. A 62 year-old male patient , admitted in the surgical Ward is scheduled for the surgical removal of polyps
from his descending colon under general anaesthesia . he is experiencing fatigue , abdominal pain and blood
streaked stools for a couple of months . he is worried whether the bleeding in his stools is going to stop after
surgery .
What is most appropriate response by the nurse for the patient concern ?
A. Surgery often relieves the symptoms
B. Let us have a detail discussion with your physician
C. Your condition may or may not resolve , it depends
D. In fact surgery is the only treatment for the problem

826. A 70year-old male patient is brought to the emergency department on awheelchair he complain of
drowsiness , fatigue ,lack od appetite for the last 3days . with history taking , the patient stated that he did not
eliminate since 5 days and he is not ambulating a lot because he has arthritis in both knee . why constipation is
one of the common problem for immobilized Patients ?
A. Decreased peristalsis
B. Increased colon motility
C. An increased defecation reflex
D. Decreased tightening of the anal sphincter

827. A patient is admitted to surgical unit after the removal of foreign body from the trachea through the
bronchoscopy . he is exhausted and is monitored for stability .
What is the most desired outcome of nursing care?
A. Presence of cough reflex and expectorant
B. Relaxed pain free and easily inbreathing
C. Optimum oxygen saturation at room air
D. Increase energy and activity level

828. A patient has undergone pericardiocentesis as part of the management of cardiac tamponade Which of the
following would be most indicative of cardiac tamponade recurring?
A. Facial flushing
B. Declining heart sounds
C. Muffled heart sounds
D. Increasing blood pressure .

829. A 19-year-old girl was scheduled for the extraction under general anaesthesia. Her pre-operative was done
and the consent was signed by her, but her nail polish and trim her long nails, requirement.
What should be the nurse's action in response to?
A. Explain why nails need to be cleaned
B. Respect the patient's right to refuse
C. Record and inform physician
D. Remove as per protocol

830. A nurse is caring for a pregnant patient who is diagnosed with abruption placenta. Which of the following
findings of assessment would indicate a concealed hemorrhage?
A. Maternal tachycardia
B. Decrease in fundal height
C. Rigid, board like abdomen
D. Acceleration in fetal heart rate
ِ‫أوليسِيُجزىِ ُمحسنٌ ِاالِباحسان‬
831. In placenta Previa marginalia, the placenta is found at the :
A. Internal cervical os partly covering the opening
B. External cervical os slightly covering the opening
C. Lower segment of the uterus with the edges near the internal cervical
D. Lower portion of the uterus completely covering the cervix

832. An infection control nurse notices purulent exudates , redness and tenderness on the surgical wound site for
few post –operative patients in a surgical unit , She discussed with the ward nurse and emphasized that wound
infection after the surgery can be prevented. Which of the following is the best possible action to minimize the
incidence of wound infection ?
A. Perform assessment of pain on the wound site
B. Wash hand before and after each patient activity
C. Encourage adequate intake and early ambulation

833. 62.receiving multiple mosquito bites and experiencing flu-like symptoms, patient consults a nurse at a clinic
and asks whether an appointment to a health-care provider is necessary. Which of the following statements
should be the basis for the nurse’s bonse?
A. Antiviral medications can be prescribed to destroy virus
B. Clinical signs can be mild flu-like symptoms to fatal encephalitis
C. If patient has West Nile virus, symptoms will progressively worsen
D. If patient used insect repellent, virus would have been destroyed when mosquito made
skin contact

834. TB chest x-ray

835. 54- community health nurse visited a postnatal primigravida mother who ent back to work. The nurse
instructed the woman the different positions breast-feeding in order to lessen the burden of the mother at
night. Which of the following position will the nurse recommend for the mother night?
A. Cradle hold
B. Football hold
C. Reclining position
D. Side lying position

836. 45- multiparous mother is attending at the outpatient clinic 2 weeks after er delivery for follow up. While the
nurse is assessing the mother, she would not palpate the fundus. Which of the following is the most appropriate
nursing action?
A. Document normal finding
B. Massage the fundus to be firm
C. Assess lochia amount and color
D. Admit the mother to the hospital

837. What is a characteristic of an outstanding hospital vision?


A. Specific in scope
B. Easily understood
C. Focused on the past
D. Kept in a private place
ِ‫أوليسِيُجزىِ ُمحسنٌ ِاالِباحسان‬
838. A 7-week-old infant boy is admitted with projectile vomiting decreased urine output decreased bowel
movements and weight loss. He has poor turgor and appears hungry. The nurse observes left-to right
peristaltic waves after he vomits. The nurse would expect to find which of the following during the physical
assessment?
A. Hepato-spleenomegaly
B. A palpable pyloric mass
C. Lymphadenopathy
D. Bulging fontanelles

839. A woman is 5 weeks gestation diagnosed with hyperemesis gar what is the most important nursing
action?
A. sychological support
B. Folic acid supplementationr
C. strict intake and output record
D. providing the woman with a high protein diet

840. A 4-month old infant is admitted with diarrhea and vomiting.To prevent the recurrence of diarrhea the nurse
should instruct the mother to do which of the following?
A. Frequent hand washing
B. Increase milk intake
C. Increase water intake
D. Change diaper frequently
841. Before administering a dose of furosemide (Lasix) to a 2-year old with a congenital heart defect, the nurse
should confirm the child's identify by checking the hospital ID band and
A. Verifying the child's room number.
B. Verifying the identity with a second nurse.
C. Asking the parent the child's name.
D. Asking the child to tell you his name.

842. A patient diagnosed with obsessive-compulsive disorder (OCD) continually carries a toothbrush, and will brush
and floss up to fifty times each day. The healthcare provider understands that the patient's behavior is an
attempt to accomplish which of the following?
A. Experience pleasure
B. Relieve anxiety
C. Avoid social interaction
D. Promote oral health

843. Which of the following uses for abdominal examination


A. Percussion Inspection. Auscultation. Palpation
B. Auscultation .Percussion .Palpation Inspection
C. Inspection. Auscultation. Percussion. Palpation
D. Inspection. Palpation auscultation .percussion.

844. Before sending a client for a CT with contrast dye, what the nurse's most important action?
A. Teach about the need for post-procedure hydration.
B. Verify that the informed consent is complete.
C. Place the side rails of the bed up before transport.
D. Check the client's health record for allergies

845. A head nurse of a Coronary Care Unit delegated the staff a senior nurse in that unit what initial step must the
head nurse implement before?
A. Check the hospital policies for delegating tasks .
B. Explain the task to the senior nurse
C. Negotiate with the senior nurse
ِ‫أوليسِيُجزىِ ُمحسنٌ ِاالِباحسان‬
D. Take the signature of the senior nurse

846. Patient with cataract was admitted in the ward for cataract surgery. On admission, the patient perceived that
surroundings are mer as if her glasses need cleaning .What is the most important nursing problem at the early
stage?
A. Eye pain
B. Diplopia
C. Blurred vision
D. Light scattering

847. nurse is about to perform venipuncture to initiate intravenous therapy with 0.9% normal saline solution.
Before venipuncture, the nurse collects the require supplies and plans to Perform certain action.
Which of the following actions is the most important?
A. Apply a tourniquet below the site of venipuncture
B. Place an arm board at the joint above the venipuncture
C. Apply cool compress over the vein to be used for venipuncture
D. Inspect 0.9% normal saline solution date of expiry

848. 133-patient is being weaned off from the mechanical ventilator is about to hook the endotracheal tube to
oxygen at FiO2 of 40 Which of the following oxygen administration device is the best the nurse in
this situation?
A. Ambo bag
B. Ventura mask
C. Tracheostomy collar
D. T-piece/ Briggs

849. A 4-month old infant is admitted with diarrhea and vomiting. To prevent the recurrence of diarrhea the nurse
should instruct the mother to do which of the following?
A. Frequent hand washing
B. Increase milk intake
C. Increase water intake
D. Change diaper frequently

850. A patient is transferred to the post-anesthesia care unit (PACU) after surgery with epidural anesthesia. After
taking the patient's vital signs, what should the nurse assess next?
A. Spinal headache
B. Postoperative pain
C. Bladder distention
D. Ability to move legs

851. A post- portative patient in surgical unit most indicative of a developing complication?
A. increasing alertness
B. weak and rapid pulse
C. negative homans' sign
D. minimal bowel sounds in four quadrants

852. A 22-year-old patient is admitted in the male diagnosis of tonsillar abscess. He has high fever along
with dysphagia, difficulty in talking and patient is planned for needle aspiration of the intravenous
antibiotic including penciling.What is nursing problem need attention first?
A. An imbalance nutrion due to inadequate
B. acut pain related to throat inflammation
C. impaired swallowing related to dysphagia
D. hyper thermia related to acute infection
ِ‫أوليسِيُجزىِ ُمحسنٌ ِاالِباحسان‬

853. A nurse received the serum digoxin level result for the patient the day and notes that the result is
2.6 ng/mL (see lab result)

Test result normal value

Digoxin (men) 2.6 0.8-2ng/mL Which of following


nursing actions is the most important?

A. Notify the physician


B. Check previous vital signs of patient
C. Record normal value on nursing note
D. Administer scheduled dose of medication

854. All of the following are types of spina bifida EXCEPT:


A. Myelomeningocele
B. Hemophilia
C. Meningocele
D. Spina Bifida Occulta

855. In the 12th week of gestation, a client completely expels the products of conception. Because the client
is Rh-negative, the nurse must:

A. Administer RhoGam within 72 hours


B. Make certain she receives RhoGAM on her first clinic visit
C. Not give RhoGam since it is not the birth of a stillborn
D. Make certain the client does not receive RhoGAM since the gestation only lasted
12 weeks

856. Which of the following patient can be transferee from emergency unit to inpatient ward?
A. Patient with DKA has nausea and vomiting
B. Patient with respiratory distress
C. Patient has operation next day
D. Patient with hematoma on manitol

857. A nurse was newly appointed to work in the Surgical Ward. She was assigned to take care of an obese
patient who just had an open laparotomy. Her mentor told her to pay attention to the surgical risks associated
with obesity. Which of the following postoperative complications the nurse should consider as a high risk
complication?
A. Hunger
B. Gas pain
C. Hemorrhage
D. Impaired wound healing

858. A nurse noticed that the digit probe of the pulse oximeter that is attacked to the right forefinger of a patient
is not reading. Which of the following is the most appropriate action that the nurse must take at this time?
A. Change the location of the pulse oximeter
B. Get a new pulse oximeter sensor probe other than a digit probe
C. Maintain position and check it after two hours
D. Assume oxygen saturation is at normal level
ِ‫أوليسِيُجزىِ ُمحسنٌ ِاالِباحسان‬

859. A nurse has taught a patient with iron deficiency anemia about eating als that are high in iron. Which of the
following meal is low in iron?
A. Dried beans and brown rice
B. Eggs and whole wheat toast
C. Steak and a salad made with fresh spinach
D. Cheese pizza and pasta with tomato sauce

860. A 45-year-old woman patient was diagnosed with bronchial asthma and was admitted in the Medical Ward.
The nurse taught the patient to metered dose nebulizer. Which of the following action indicates that she
understands the instruction?
A. Inhale slowly and deeply puff
B. Does not shake the pre-packaged nebulizer
C. Administer the puffs rapidly between breath
D. Remove the aero chamber immediately after administering the puff

861. Which of the Following Is the First Step for Inhaler Technique ducation in Patients with Asthma
A. Breathe out gently.
B. Place mouthpiece between lips.
C. Shake the inhaler.
D. Wait a few seconds then repeat the above process

862. nurse is caring for a child with a diagnosis of pneumonia. The plan of nre includes nebulizer treatments and
chest physiotherapy. When should the nurse perform chest physiotherapy?
A. Prior to the nebulizer treatment
B. After the nebulizer treatment
C. Intermittently during the nebulizer treatment
D. Continuously during the nebulizer treatment

863. When assessing the funds level At 36 week it is at the Xiphoid process

864. Pregnant women with genital herpes …. Caesarean section

865. Caput succedaneum” refers to swelling, or edema,

866. Nephrotic syndrome urine color ……………….. Dark urine

867. Stages of grief after death The five stages, Denial, Anger, Bargaining, Depression and Acceptance ( DABDA)

868. Complication of MI is Dysrhythmias

869. Nursing supervisor have shortage of staff ........ call for help

870. Gestational diabetic mother can be treated with Diet management

871. Which of the following is the best describe of hospital Vision.


A. provide a quality of care we would want for ourselves,
ِ‫أوليسِيُجزىِ ُمحسنٌ ِاالِباحسان‬
B. Treat every individual in a non-judgmental manner, ensuring privacy, fairness and
confidentiality
C. Improve the responsiveness of our services for the benefit of our patients and their
families
D. Helping local people live longer, healthier lives.

872. Basal skull fracture signs................... Racoon eyes

873. Signs Intracranial pressure in infant with hydrocephalus is bulging anterior fontanel

874. For physical assessment steps for the foot is


A. Inspection , palpitation
B. Palpation , auscultation
C. Percussion, palpation

875. A nurse is teaching cord care to a group of new mothers during a baby bath demonstration. One of the
mothers ask the nurse on how to prevent infection of the cord. Which of the following is the best nursing
response?
A. Apply antibiotic cream to the cord after every diaper
B. change Clean the cord with water when necessary and keep dry
C. Clean the cord with surgical spirits twice a day
D. Cover the cord with a bandage and change daily

876. A37 years-old man present man present to emergency department with chest pain . An ECG shows significant
elevation in the ST segm II , III and aVF ,indicating MI related to occlusion in the artery what is the location of MI
?
A. Posterior MI
B. Anterior MI
C. Inferior MI
D. Lateral MI

877. which of the following indication for patient with MI:-


A. ST depression
B. ST elevation
C. short ST
D. none of the above

878. Complication of MI ....... Dysrthysmias , heart failure , pulmonary edema , angina , pericarditis , Dressler
syndrome

879. Causes of MI ...................atherosclerosis , coronary artery disease , elevated cholesterol level , hypertension,
smoking , obesity , stress

880. Watery rice diarrhoea ....... cholera

881. 71-year-old male was diagnosed with subdural underwent burr hole craniotomy for subdural hernatoma days
ago. In order to detect the sign of meningitis as one of which of the following indicates the patient has meninge?
A. Negative Kernig's signs
B. Positive Brudzinski's sign
C. Absence of nuchal rigidity
D. Glasgow comma scale of 14 points
ِ‫أوليسِيُجزىِ ُمحسنٌ ِاالِباحسان‬
882. A nurse evaluate dietary practices of a patient who had been as having acute glomerulonephritis . the patient
verbalized following the appropriate diet for acute glomerulonephritis . which of the following diet verbalized by
the patient is approved diagnosis ?
A. Restricted fluid intake
B. Restricted dietary protein
C. Increase intake of low fiber food
D. Increase intake of sodium-rich food

883. which of the following best describes the role of a case management?
A. promotes healthy lifestyle.
B. provides coordination of care to ensure continuity.
C. possesses highly skilled communication methods.
D. promotes the comfort of the patient

884. A 3-year-old child is admitted to the hospital with seizures. He oriented and has a rash in his extremities and
is diagnosed meningitis. While doing physical examination of him, he starts to seizures. Blood pressure
100/57 mmHg Heart rate 110 /min Respiratory rate 30 /min
Temperature 39.5 Which of the following is the priority of care during the seizure?
A. Put the child on right side
B. Protect the child from injury
C. Call the physician immediately
D. Administer oxygen 100%

885. A 5 years old child admitted with bacterial meningitis and is having seizure. Which of the following
intervention the nurse should initiate ?
A. restrain the child’s limbs to prevent injury
B. slowly put the child on his side on the floor
C. clear the area of objects and administer oxygen
D. roll the child to prone position to protect the airway

886. In the summer months, a five year-old girl presents with a sore throat and a dry cough that has slowly become
worse over the past three weeks. Her body temperature is 38.0oC. On auscultation, there is wheezing and
shortness of breath. She lives in an overcrowded house with three brothers, parents and grandparents in a low-
income neighborhood where she attends school. Which is the greatest risk factor?
A. Residing in a low-income neighborhood
B. Exposure to a pathogen in summer season
C. Attending school
D. Living in crowded conditions

887. 70 years-old woman is admitted to the cardiac care unit wit atrial fibrillation and is receiving intravenous
diltiazem and heparin What is the most likely nursing diagnosis?
A. High risk for infection
B. High risk for impaired gas exchange
C. High risk for decreased cardiac output
D. High risk for disturbed sensory perception

888. A patient hospitalized with chest pain is diagnosed with angina Which of the following discharge instruction
takes priority?
A. recognize signs and symptoms that require immediate report
B. Maintain low cholesterol, low sodium, and low potassium
C. Carry heavy objects away from the chest area
D. Maintain physical exercise daily

889. The nurse is assigned to care for a patient with Paget’s dis complaining of bone pain. Which of the following
conditions is this patient high risk for?
ِ‫أوليسِيُجزىِ ُمحسنٌ ِاالِباحسان‬
A. Hypomagnesaemia
B. Hypernatremia
C. Hypercalcemia
D. Hyperkalemia

890. A 25-year-old-primipare visited her primary health care clinic husband. He says that his wife delivered 5 days
ago. For the past days she is very irritable and crying for nothing, she is tired and sleep and he is very concerned
about her condition. Which of the following diagnosis by the attending physician will her symptoms?
A. Post-partum blues
B. Postpartum psychosis
C. Postpartum depression
D. Postpartum anxiety

891. A 44 year-old woman presented to the Emergency Department with eye pain and redness. She was
diagnosed with uveitis related to a viral infection. The doctor prescribe dexamethasone eye drops.Where is the
correct place to instill eye drops?
A. Above the eyelid
B. Directly into the cornea
C. Into the conjunctiva sac
D. Outside the conjunctiva sac

892. A 22-year-old man presents to the Emergency Department complaints of breathing difficulties. He appears
restless and reports that he has had a cough with thick green sputum for days. The nurse performs auscultation
over the lung fields.
Blood pressure 130/80
Heart rate 100
Respiratory rate 24
Temperature 39
Which clinical finding is most likely heard over the right lower?
A. Decreased vocal resonance
B. Decreased fremitus
C. Tympani
D. Bronchial

893. A hospitalized 72-years-old man who uses a walker is received medication and must use the bathroom
several times each night To promote the safety of the patient, which of the following appropriate nursing
action?
A. Keep the side rails up
B. Leave the bathroom light on
C. Provide a bedside commode
D. Withhold the patient’s diuretic medication

894. A man patient admitted to the Medical Unit was diagnosed with deep venous thrombosis complaining of
pain on both legs. Which of the following nursing diagnosis most likely describes problem?
A. Risk for injury
B. Fluid volume excess
C. Electrolyte imbalance
D. Impaired tissue perfusion

895. A client was admitted in the Emergency Room due to mild metabolic acidosis associated with dehydration
and potassium the doctor administered Hartmann’s (lactated Ringer’s) intravenous fluid and electrolyte
replacement. Which of the following elements of the lactated Ringer’s solution highest value?
A. Calcium
ِ‫أوليسِيُجزىِ ُمحسنٌ ِاالِباحسان‬
B. Sodium
C. Potassium
D. Magnesium

896. A client with a blunt trauma underwent an exploratory lapartomy repair an intra-abdominal injury. A
nasogastric tube attached suction and two closed-wound Hemovac abdominal drains He is receiving an
intravenous infusion of Ringer’s lactate epidural with continuous morphine. Twenty-four hour post- develops
hypotension, tachycardia, oliguria and sever nausea Which of the following is the priority nursing diagnosis?
A. Nausea
B. Risk for infection
C. Deficient fluid volume
D. Impaired urinary elimination

897. The nurse is mentoring a newly graduated nurse who is right-handed 39 year-old woman who underwent a
left-sided and is now in the medical-surgical unit. The intravenous fluid to infiltrate into the infusion site and the
student nurses equipment to insert a new intravenous catheter. Which catheter insertion site would indicate
that the new nurse needs to be corrected?
A. A site that has soft and elastic skin
B. A site next to a joint
C. The right arm
D. A distal site

898. A client in the second stage of labor is unable to push and lacks to bear down. What is the most appropriate
next step?
A. Assess fetal descent
B. Infuse intravenous fluids
C. Empty the client’s bladder
D. Administer oxygen to the mother

899. patient with a spinal cord injury states, "I have no constitution; I can’t do anything for myself." Which of the
following best describes this patient condition?
A. Powerlessness
B. Delusions
C. Suicidal
D. Resignation

900. A discharge planner is reviewing the file of a patient in prepare move him from the acute care to home care.
What activity should the discharge planner to do ensure the co care?
A. Make referral to the appropriate department
B. visit patient’s house to make sure it is safe
C. Call pharmacy to prepare medications for patient
D. Organize transportation for patient to be moved to

901. A newly assigned nurse to the home health care department was conducting pre-entry phase of home visit.
Which of the following activities should the nurse do?
A. call physician for referral order
B. collect information and schedule a visit
C. establish mutually acceptable goals for care
D. conduct a health history and physical assessment

902. A nine-month-old child who has had four ear infection six months is being discharged.
Which statement by the parent indicates the need for teaching?
A. I should never put my baby to bed with a bottle
B. My child should not use a pacifier after age six o
C. My child should drink his bottle while lying flat
ِ‫أوليسِيُجزىِ ُمحسنٌ ِاالِباحسان‬
D. My child should not be around people who sm

903. A mother who is postpartum calls nurse in the mother/Infant unit. She reposts has very heavy bright-red
bleeding. What would be the best advice to give her?
A. "Don’t worry about it; it is normal"
B. "Call your doctor and ask what to do"
C. "Lie down, massage lower abdomen. If it does not work to the hospital immediately"
D. "Lie down for about an hour. Then check your bleeding, if as heavy, call me back

904. A 15-year-old girl is admitted after a Motor Vehicle Accident child has a fracture in her left leg and a wound
in her face. When dealing with this child the nurse should be aware that important task to achieve during this
age is which of the following?
A. Initiative versus guilt
B. Industry versus inferiority
C. Trust versus mistrust
D. Identity vs. Role confusion

905. A 9-month-old child who has a repair cleft palate the nurse explaining mother on how she will give feeds to
her child. Which of the following instruction can be expected to include feeling education?
A. Open cup
B. Tea spoon
C. Bottle feed
D. Special bottle feed

906. A nurse calls together an interdisciplinary team with members from medicine, social service, the clergy, and
nutritional services to care for a patient with a terminal illness. Which of the following types of care would the
team most likely be providing?
A. Palliative
B. curative
C. respite
D. preventive

907. A nurse I,s preparing to assess the uterine fundus of a client in the immediate postpartum period. When the
nurse locates the fundus she notes that the uterus feels soft and boggy. Which of the following nursing
interventions would be most appropriate initially?
A. Massage the fundus until it is firm
B. Elevate the mother’s legs
C. Push on the uterus to assisting expressing clots
D. Encourage the mother to void

908. A 43 year-old man in the post-surgical area complains of abdominal pain radiating to the naval which is
increasing with examination his abdomen is guarded with marked tender lower quadrant. What is the
immediate goal of care to do?
A. Teach abdominal splinting during coughing
B. Administer pain medication as ordered
C. Assess pain and report immediately
D. Position on the left lateral side

909. 27 years old female brought to the Emergency Room accompanied by her husband. He described that she
had marked Weight loss with episodes of emesis in the past three months. She is diagnosed as having anorexia.
She reported feeling Febrile, but had not measured her temperature. Her White Blood Count was 11,000/mm3.
Which of the following most Likely describe her diagnostic criteria for her anorexia?
A. Restricting food intake
B. Fear of gaining weight
C. Problems with body image
ِ‫أوليسِيُجزىِ ُمحسنٌ ِاالِباحسان‬
D. Binge eating disorder

910. 66 old women presents to the emergency department triage unit with labored breathing. She appeared
anxious and is sweating profusely across the forehead. The symptoms had begun Suddenly one hour before. The
jugular veins appear distended and she has a productive cough. On Auscultation the respiration is wet and not
clear with coughing..
Blood pressure 148/88 mmHg
Heart rate 94/min
Respiratory rate 26/min
Temperature 37.0◦C
Which triage acuity rating is most appropriate?
A. Urgent
B. Emergent
C. Non-urgent
D. Resuscitation

911. A nurse is providing a health teaching to a client about the proper use of a cane during ambulation. The nurse
could include which of the following instructions about the correct use of cane?
A. Patient moves the stronger leg forward to the cane
B. Patient holds the cane on the strong side of the body
C. Patient leans on the cane to provide support to the weaker side
D. Patient moves the cane forward about 4 inches to the strong side

912. A child with Jaundice and treat with photography , how can you protect the child skin?
A. Avoid oily substance
B. Using Vaseline after skin cleaning
C. applying cream

913. A nurse is caring for a patient two hours after a pacemaker placement. The patient suddenly starts
complaining of chest pain. The nurse observes dyspnea, cyanosis and absent breath sounds on the right side. The
nurse should anticipate what complications?
A. Hemothorax
B. Perforation of the heart
C. Pneumothorax
D. Hemorrhage

914. Which of the following could adversely affect the function of a pacemaker?
A. Hair dryer
B. Electric razor
C. Television
D. Electric mixer

915. The nurse provides discharge instructions to a client after implantation of a permanent pacemaker. The nurse
should tell the client to avoid exposure to which item?
A. Hair dryers
B. Electric blankets
C. Electric toothbrushes
D. Airport metal detectors

916. The nurse provides discharge instructions to a client after implantation of a permanent pacemaker. The nurse
should tell the client to avoid exposure to which item?
A. Hair dryers
B. Electric blankets
C. Electric toothbrushes
D. Electric Mixer
ِ‫أوليسِيُجزىِ ُمحسنٌ ِاالِباحسان‬

917. A 58 year-old man had a permanent pacemaker inserted. The device is in place and functioning well. The skin
at the incision site is intact and free of infection. The nurse gives him the discharge instruction. What is the most
important nursing care outcome?
A. Understand functioning and operating of pacemaker
B. Monitor pulse rate and identify need for reporting
C. Able to identify the signs of detached pacemaker
D. Modify life style and wear pacemaker alert sign

918. A 9-year-old girl is seen in the emergency department because of a fracture in the right fibula. Which of the
following is the expected response to parent question about bone healing period of the girl?

a. 4-6 weeks
b. 2-4 weeks
c. 6-8 weeks
d. 8-10 weeks

(child (child 1) (child 2) (child 3) (child 4)


1)Behaviour of The child The child The child The child
the child completes the forgets apart of perform well completely meal
project on time their uniform and gets 90% on time
marks
Responses on The teacher The teacher The teacher The Maker gives
the child's rewards for wants singly doesn't an ice cream to
behaviour discipline instructs not to appears the the child
repeat AGAIN child

919. A school nurse is monitoring the behaviour of different children. Which child should the nurse expect to
show an aversive type of reinforcing stimulus (see image)?
A. Child 1
B. Child 2
C. Child 3
D. Child 4

920. A 13-weeks-pregnant, multi gravida women is anxious and apprehensive she has five children and is not
willing to continue with this pregnancy She is requesting the midwife to abort the fetus. She is under weight
malnourished and is over worked.
BMI 17 Kg/m2
What intervention is desired immediately?
A. Admission and intravenous line management for induction
B. Family planning and birth control measures
C. Dietary management and supplements
D. Support, reassurance and counselling

921. which of the following is the most appropriate action for a nurse Manager starting at a new hospital?
A. Observe staff while keeping presence and role unknown
B. Plan and coordinate new strategies with top level management
C. Introduce self and observe unit activities for at least three months
D. Meet with each staff member separately about needed unit charges

922. A nurse explains to a patient that an endoscopy tube will be inserted down the throat during a gastroscopy
which will cause gagging, but it must be swallowed. What type of communication the endoscopy nurse has used
here?
A. Assertive
ِ‫أوليسِيُجزىِ ُمحسنٌ ِاالِباحسان‬
B. Interactive
C. Formative
D. Informative

923. A nurse who works in the surgical unit at one of the hospitals was asked by the home health care nurse to
make a home visit to a patient with colostomy, who had been discharged the previous day in order to give him a
follow-up care and education which of the following nurses should do the assigned task?
A. Critical care nurse
B. Psychiatric nurse
C. Surgical nurse
D. Community nurse

924. When is the ideal time to administer analgesia to a women in labour?


A. A soon as she requests analgesia
B. When labour is well established
C. When the women enters into transition phase
D. when the women progress from latent to active phase

925. A 46-year-old patient is admitted in the female Medical Ward for seven back pain, which is graded 6 on the
scale of 10. Her vital signs are take with the slight elevation in her blood pressive. The patient has refused
eat lunch which is a low sodium diet. The attending nurse has documented that patient is uncooperative and has
refused to eat the lunch. What nursing intervention needs attention first?
A. Arrange for alternative diet
B. Teach importance of the low sait diet
C. Take appropriate measures to relieve pain
D. Inform the physician for elevated blood pressure

926. a 30-year-old man was brought to the hospital by ambulance sitter felling from a height of 10 meters. He
was mechanically ventilated on after a Glasgow Coma Score showed a level of six. The nurse is
observe for any changes in perceptual, sensory or cognitive
Which of the following is an expected patient's response at this time?
A. Slowly obeys commands
B. Exhibits no motor response
C. Reacts towards painful stimuli
D. Uses incomprehensible words

927. A 46-year-old women is admitted in the Medical Ward for systemic lupus erythematous, complaining of joint
pain with dull aching pain in the number region. She has butterfly rashes on her face. Which of the following
should be the first nursing intervention for this patient?
A. Skin care and cortisone ointment as ordered
B. Pain relieving measures and physiotherapy
C. Assess back pain and monitor urine output
D. Analgesics as ordered warm environment

928. In the Emergency Unit a nurse made an error that lead to an admission order for the client to be on a venous
thromboembolic protocol is not processed. Two days after, a nurse notices the omitted order for heparin
5000 units subcutaneous every eight hours. which of the following statement best describes the appropriate
follow-up?
A. " I will contact the supervisor immediately about this error"
B. " I need to contact a physician and complete a variance report"
ِ‫أوليسِيُجزىِ ُمحسنٌ ِاالِباحسان‬
C. " I am too busy to complete a variance report. I'll do it next week"
D. " I am so glad I didn't make that mistake, that other nurse is going to be in trouble"

929. A 75-year-old patient was bed ridden for the last one year, She was incontinent, restless, delirious and was
trying to pull her nasogastric tube out , but her arms were restrained; her forearms had bruises and her lips
were dry and cracked.
Blood pressure 100/64 mmHg
Heart rate 102 /min
Respiratory rate 28 /min
Temperature 39.2 C
Which of the following nursing intervention each Nasogastric feed?
A. Pull to check the tube is in place
B. Check the tube for patency
C. Aspirate the content
D. Insert water

930. A 45-year-old of man was admitted to the Surgical Ward after removed of pancreatic cyst. The registered
nurse checked the post-operative orders Ondansetron PRN was prescribed for the patient.
Which of the following complaints from the patient will require the registered nurse to serve the Ondansetron
after 12 hours of pos-operativeperiod?
A. Wound pain
B. Nausea and vomiting
C. Congestion of flame
D. difficulty in passing urine

931. A man nurse is caring for a women diagnosed with major depression. The client states " I never had a male
nurse before". which of the following is the most appropriate nursing response?
A. " Do you feel threatened"
B. "Most of the nurses here are males"
C. " How do you feel having a male nurse"
D. " Do you want me to find a female nurse"

932. Which of the following statements best describes for patients with myopia?
A. They must hold a paper at arm's length or greater to read it
B. They can see close-up objects clearly, but distant objects are out of focus
C. They can see distant objects clearly, but close-up objects are out of focus
D. They are unable to focus horizontal and vertical rays of light on the retina simultaneously

933. Which of the following tests measures the total quantity of prothrombin in the blood and monitors the
effectiveness of warfarin sodium (Coumarin) herapy and prolonged deficiencies in the extrinsic factor?
A. Thrombin time
B. Prothrombin time
C. Partial prothrombin time
D. Activated partial prothrombin time

934. A 72-year-old patient presents to the medical clinic drooling with a blank, masklike facial expression and a
high pitched, monotone, weak wike When walking, the patient does not swing the arms normally, but holds hem
stiffly rigid. What is the most likely diagnosis?
A. Parkinson's disease
B. Substance abuse
C. Traumatic brain injury
D. Transient ischemic attack
ِ‫أوليسِيُجزىِ ُمحسنٌ ِاالِباحسان‬
935. A 69-year-old obese women with a pelvic fracture has been immobile for four days. The patient is now
anxious, dyspneic, and domaining of substernal pain. The patient's capillary refill is greater than seconds.
Heart rate 122 /min
Respiratory rate 26 /min
Temperature 38.2 C
Which of the following the nurse suspects for this patient?
A. Panic attack
B. Pulmonary embolism
C. Aspiration pneumonia
D. Pneumothorax

936. The nurse is planning for a discharge teaching plan for a family of a30-year-old man with AIDS in relation to
sanitation practices. Which of the following statements should the nurse include in her instructions?
A. Wash used dishes in hot soapy water
B. Boil the dishes the patient used for 30 minutes and them wash with soap
C. Have the patient use disposable eating tools so it can be discarded after
D. Soak the kitchen tools in hot water for 24 hours before washing with soap

937. A 21-year-old in oversized clothing presents to the hospital with of felling dizzy and faint. The hair and nail
appear thin and dry. The skin appears pale and she has sunken eye sockets and tenting skin. Her body mass
index is 16. She often induces vomiting after eating blood is collected for analysis (see lab results).
Blood pressure 90/52 mmHg
Heart rate 118 /min
Respiratory rate 26/min
Temperature 37.2 ͦ C
Oxygen saturation 97%
Test Result Normal value
ABG¹ HCO₃ ̵ 31 22 – 88mmol/L
ABG PCO₂ 10.3 4.7-6.0 kPa
PH 7.50 7.36-7.45
Which nursing problems stem is the most appropriate?
A. Impaired nutrition
B. Decreased cardiac output
C. Infective airway clearance
D. Ineffective breathing pattern

938. Before preparing a client for the first surgical case of the day, a part-time scrub nurse asks the circulating
nurse if a three minute surgical hand scrub is adequate preparation for this client.
Which response should the circulating nurse provide?
A. Inform the nurse that hand scrubs should be three minutes between cases
B. Direct the nurse to continue the surgical hand scrub for five minute duration
C. Validate that the nurse is implementing the OR policy for a surgical hand scrub
D. Ask the experienced nurse to perform this scrub since it is the first one of the day

939. Which f the following statements describe the function of Immunoglobulin (IgG).
A. It does not cross the placenta and is present in colostrum and breast milk
B. It protects against gastrointestinal and respiratory system infections
C. It provides temporary immunity to bacteria and toxins to which the mother has developed
immunity
ِ‫أوليسِيُجزىِ ُمحسنٌ ِاالِباحسان‬
D. It protects against gram negative bacteria and can be found cord blood

940. An infection control nurse is responsible to conduct the surveillance on how the hospital staff take
precautionary measures against flu, based on her findings, she is responsible to conduct the awareness razing
for the doctors and nurses in the in-patient areas. Which of the following is the goal of the nursing practice here?
A. Infection control management
B. Research and survey
C. Primary prevention
D. Health education

941. A 57-year-old man is admitted to the Cardiac Unit with palpitation headache, and chest tightness. On
auscultation S3 gallop and
murmur can be heard (see image).

Blood pressure 97/60 mmHg


Heart rate 170 /min
Respiratory rate 25 /min
Temperature 37.3 C
What is the ECG rhythm of the patient?
A. Atrial fibrillation
B. Ventricular tachycardia
C. Ventricular fibrillation
D. Supra-ventricular tachycardia

942. Which of the following is the main disadvantages of home care visit
A. Time consuming for healthcare team
B. Inexpensive than hospital care
C. Easy access to equipment or consultation
D. Nurses have more control on mother's distraction

943. Patient has nausea, vomiting and muscle weakness comes to ER, after assessment the doctor consider that
sign and symptom due to:
A. Hypokalaemia
B. Hypernatremia
C. Hyperkalaemia
D. Hypoglycaemia

944. Patient has heart failure and suddenly has edema. What is the lung sound will be hearing?
A. Rales
B. Rhonchi
C. Wheezing

945. The physician has written an order for the nurse’s assignee have a 24-hour urine collection sent to the
laboratory for special. Which of the following should the nurse realize prior to urine?
A. Start the urine collection at either 12:01 a.m. or 12:01pm
B. Provide enough sterile receptacles for the urine collection
C. At the start of the collection period, have the patient discard this urine
D. Inform the patient that they must save all urine for 24 beginning at 12:01 a.m.

946. After Bee sting which of the white blood cell is the most likely elevated
A. Eosinophils
B. Basophils
C. Monocytes
D. Neutrophils
ِ‫أوليسِيُجزىِ ُمحسنٌ ِاالِباحسان‬
947. A nurse was educating a group of woman on prevention of infection. The nurse asked each woman to state
one preventive of vaginal infection. Which woman needs more education?
A. First woman "keep vaginal area clean & dry"
B. Second woman "wear cotton under wear"
C. Third woman "wipe from front to back after urination or
D. Fourth woman "Do vaginal douche twice a day "

948. Disadvantage of democratic leader ................ cannot be in Emergency situation


949. Post operative breathing exercise importance
A. Prevent atelectasis
B. Move secretion

950. The quality nurse manager teaches a new nurse graduate about the comparison between total quality
assurance (QA) and quality improvement (QI). What is the most appropriate statement regarding the common
aspect between QA and QI?
A. Aim to improve quality
B. focus on preventing errors
C. focus on staff development
D. view nursing activities as major tasks

951. client has atrial fibrillation the nurse should monitor the client for
A. cardiac arrests
B. cerebrovascular accident
C. heart block
D. ventricular fibrillation

952. Azithromycin is prescribed for an adolescent female who has pneumonia and recurrent chlamydia
What information is most important for the nurse to provide the
A. Use two forms of contraception while taking this drug
B. Have partners screened for human immunodeficiency
C. Decrease intake of high-fat foods, caffeine, and alcohol
D. Report a sudden onset arthralgia to the healthcare pro

953. Postpartum or puerperal infection is any clinical infection of the genital canal that occurs within 28 days
after miscarriage, induced abortion, or childbirth. The definition used in the United States continues to be the
presence of a fever of 38° C (100.4° F) or higher on 2 successive days of the first 10 postpartum days, starting 24
hours after birth.
954. Isoniazid for TB side effect numbness , hepatitis symptoms , stomach upset ,rashes

955. Neck rigidity .... nerve stiffness sings of meningitis

956. Alhj vaccine is Hib vaccine meningitis vaccine


957. Nerve cause tooth pain ..... V

958. Hypoglossal nerve 12th cranial nerve ............ innervates tongue muscle
959. MI ... ST elevation , Ischemia ..............ST depression
960. Pulse deficit : is the different between apical beats and brachial pulse
961. Common cause of death in school age is ................ accidents
962. Serotonin Alzheimer ... dopamine schizophrenia
ِ‫أوليسِيُجزىِ ُمحسنٌ ِاالِباحسان‬
963. When a patient was first diagnosed with schizophrenia, one of his family members asked the nurse about the
possible causes. The nurse said that one reason is that he may have had an excess secretion of a
neurotransmitter. Which of the following neurotransmitters?
A. serotonin
B. dopamine
C. glutamate
D. endorphins

964. 452. 11 month baby weight 8 kg has vomiting 7 times how many ml orally he can received ?
A. 200
B. 600
C. 800
D. 1200
Weight x 75

965. Patient has low platelet low hemoglobin low WBCs


A. Prevent bleed
B. Check vital
C. Reverse isolation

966. A child has diarrhea 7 times and he is irritable and loss his skin turgor what is the degree of dehydration?
A. Mild
B. moderate
C. sever
D. extensive

967. According to information provider in the accompanying graphic, a transverse colostomy would be expected
to produce which of the following (see image)?
A. Fluid faces
B. Mushy feces
C. Semi-fluid feces
D. Solid feces

968. Baby boy birth weight is 3 kg what the expected weight


ate age of 1 year ?
A. 7
B. 9
C. 11
D. 13

969. Gastric suction can cause :


A. Metabolic acidosis
B. Respiratory acidosis
C. Metabolic alkalosis
D. Respiratory alkalosis.

970. Basilic vein A


ِ‫أوليسِيُجزىِ ُمحسنٌ ِاالِباحسان‬

971. Oral iron supplements are prescribed for child with iron deficiency anemia the nurse instruct the mother to
administer the iron with which food
A. Orange juice
B. Apple juice
C. Milk
D. Water

972. A 27-year-old woman patient's umbilical hernia is repaired under local anaesthesia. Her gold amulet which she
kept under her pillow was missing at first but now is given to her after it is found from the bag of the
housekeeping staff. The nurse in charge is to write an incident report which includes the patient's name, hospital
identification number, date, time and place of incident and the details of the item stolen. What short term action
is required by the in charge nurse?
A. Explain patient her belongings are her responsibility
B. Personal counseling of the house keeping staff
C. Report incident to the security department
D. Dismiss the involved personnel

973. patient is scheduled for a total hip arthroplasty. The nurse reviews the chart and notes the following: serum
potassium 2.8 mEq/L, AB positive blood type, and elevated ST segment electrocardiogram. Which of the
following would be the most appropriate action to do next?
A. Report abnormal diagnostic results to the surgeon
B. Review the patient consent for the surgical procedure
C. Educated the patient on the risk factors and side effects surgery
D. Ensure that the patient has a post-surgery physical therapy

974. Cardiac Care unit nurse decide to gather relevant data of myocardia infraction patient who has breathing
difficulty, bradycardia. What is the best kind of databases relevant to his situation for the nurse?
A. Focused database
B. Complete database
C. Emergency database
D. Follow-up database

975. A nurse cares for a client with an epidural catheter for labor pain management and requests additional
medication. A student registered nurse anesthetist (SRNA) responds and administers medication via the
epidural catheter. Which action performed by the SRNA requires the nurse to stop administration of
medication?
A. Aspirating bloody fluid into a syringe
B. Obtaining a negative aspiration into a syringe
C. Cleansing the port with alcohol for 15 seconds
D. Assessing the client’s blood pressure

976. Patient with Osteoporosis what the best exercise he should practice
ِ‫أوليسِيُجزىِ ُمحسنٌ ِاالِباحسان‬
A. Walking
B. Bowling

977. Patient feeling hand numbness which medication should receive


A. Vitamin C
B. Vitamin K
C. B12
D. Folic acid

978. Post limb amputation pain :


A. Phantom
B. Radiating
C. Shooting

979. A co-worker informs that the nurse about experiencing increased level of stress associated with daily
responsibilities to help cope with professional stress, the nurse should encourage the co-worker to;
A. Make a list of unfinished tasks
B. Complete complex mental task before physical tasks
C. Acknowledge daily accomplishments
D. Spend time with colleague away from work

980. Beneficence refers to actions that promote the well-being of others. The nurse who is exemplifying
beneficence takes positive actions to help clients

981. After testicle removal the client should avoid heavy lifting

982. According to Ramsey sedation scale pt in icu on sedation on assessment patient deep sleep and not response
to light what is score
A. 2
B. 3
C. 4
D. 6

983. A 62-year-old women admitted to the emergency department for the fourth time this year, each time the
patient comes with severe injuries and bruises in the body. What is your responsibility as a nurse to prevent such
incident to happen again?
A. Reports the assault to the local police and write a report
B. Provides information about safe shelter and support
C. Instructs the women to move away from her home
D. Discharge the patient to a safe shelter

984. After rectal surgery what bath type …………….. Sitz bath

985. Mother came to ER with child diagnosed with rheumatic fever the nurse should ask the mother if there is
family members has
A. Congenital heart disease
B. Recently have tonsillitis

986. A 45 years old with severe upper GI bleeding is admitted to the medical ward . The doctor orders the nurse
to give the patient blood transfusion. Which peripheral IV catheter gauge is best for this procedure ?
A. 14 gauge
B. 16 gauge
C. 20 gauge
D. 24 gauge
ِ‫أوليسِيُجزىِ ُمحسنٌ ِاالِباحسان‬
987. Which of the following religion is prohibited the blood transfusion?
A. Christian
B. Muslims
C. Jewish
D. Jehovah

988. A nurse is caring for a client who is having an allergic reaction febrile , flushed what the first nursing action
A. Stop blood transfusion immediately
B. Replace the blood with saline.
C. Administer an antihistamine.
D. Place the client flat with the feet elevated.

989. The nurse is monitoring a client receiving a blood transfusion after 30 minutes when the client develops a
cough with shortness of breath. The client also complains of a headache and a racing heart. What should the
nurse do first?
A. Stop transfusion immediately
B. Replace the blood with saline.
C. Administer an antihistamine.
D. Slow the infusion rate

Important Notes : if reaction happened within the first 15 minutes ……. Stop blood transfusion immediately .
if reaction happened after 15 minutes……… slow infusion rate first cause of overload then inform
the physician
990. During a patient's blood transfusion, which of the following are associated with an allergic reaction? the nurse
should recognize that
A. Diarrhea and rapid onset of chills
B. Itching and asthmatic wheezing
C. Low back pain and acute renal failure
D. Distended neck veins and a rise in venous pressure

991. A nurse is assigned to care for a 32-year-old glomerulonephritis. The nurse is transcribing patient file.
Which of the following orders should the nurse clarify?
A. Bed rest
B. Daily weights
C. Strict intake and output check
D. Frequent blood pressure check

992. When is the best time to collect urine specimen for routine urinalysis and culture and sensitivity?
A. Early morning
B. Later afternoon
C. Midnight
D. Before breakfast

993. The nurse is assigned to care for a patient with Paget's disease complaining of bone pain. Which of the
following conditions is this patient high risk for?
A. Hypomagnesaemia
B. Hypernatremia
C. Hypercalcemia
D. Hyperkalemia

994. Mother came to ER with rheumatic fever baby the nurse should ask the mother about?
A. if there is any member family has Congenital heart disease
B. If he recently had tonsillitis
ِ‫أوليسِيُجزىِ ُمحسنٌ ِاالِباحسان‬
995. Young patient arrived to ER with burn in his left arm and his weight 48Kg how to calculate fluid requirement
for the patient according to parkland formula?
A. 1572ml
B. 1220ml
C. 1728ml
D. 2000ml
4 x WT x burned are ......................... 4 x 48 x 9= 1728ml

996. A75 year-old man is scheduled to undergo phacoemulsification surgery. The nurse meets with the family for a
pre-operative. The patient has a diagnosis of Alzheimer’s disease with memory decline. The family says that he
has become increasingly for and that his emotions are unstable. Which of the following actions the patient is
expected to do operation?
A. Lay supine for 20 minutes
B. Sit without moving under laser
C. Consistently speak clearly
D. Tolerate general anesthesia

997. A nurse admits a client sent from the health care provider’s office. The nurse calls the provider to obtain
prescriptions. Which action does the nurse take?
A. Hold treatments until the provider signs the prescriptions received.
B. Read each prescription back to the provider prior to ending the call.
C. Have the provider provide the prescriptions to another nurse as well.
D. Verify the prescriptions with the charge nurse before beginning treatment.

998. ABG reading was low PH ,HIGH PCO2 , NORMAL Hco3 what the interpretation ?
A. Compensated respiratory acidosis
B. Uncompensated respiratory acidosis
C. Metabolic acidosis
D. Metabolic alkalosis

999. A 47 years old newly been diagnosed as having gastroesophageal reflux disease (GERD) comes to the clinic for
follow up appointment. The nurse prepares health education on the cause and care of GERD. Which of the
following stomach area is associated with the diagnosis? (See image)

A- 1
B- 2
C- 3
D- 4

1000. three years old has returned to the clinic after 4 days of being
diagnosed with gastroenteritis and dehydration. A parent reports that the vomiting has stopped, and the child is
tolerating liquids, rice, applesauce, and bananas. The diarrhea persists, but seem to be decreasing in volume.
When evaluating for signs of dehydration, the nurse will assess the patient’s skin turgor by:
A. grasping the skin over the abdomen with two fingers and raising the skin with two fingers
B. grasping the skin over the forehead with two fingers and raising the skin with two fingers
C. holding the patient’s mouth open and assessing the tongue for deep creases or furrows
D. drawing two tubes of blood and running blood urea nitrogen (BUN) and creatinine (Cr)

1001. To decrease the anxiety of a 10-year-old girl who is undergoing surgery. Which of the following should the
nurse do?
A. Use a heart model to show her how the surgery will go
B. Provide her with verbal explanation of the upcoming surgery
C. Give her a book to read about the surgery 2 weeks prior
D. Let her parents talk to her about the importance of having surger
ِ‫أوليسِيُجزىِ ُمحسنٌ ِاالِباحسان‬
1002. An infection control nurse notices purulent exudate, redness and tenderness on the surgical wound site for a
few postoperative patients in a surgical unit. She discussed with the ward nurses and emphasized that wound
infection after surgery can be prevented. Which of the following is the best possible action to minimize the
incidence of wound infection?
A. perform assessment of pain on the wound site
B. wash hands before and after each patient activity
C. encourage adequate intake and early ambulation
D. support incision appropriately and avoidance of straining

1003. A nurse is giving health education regarding the management of premenstrual syndrome symptoms to a group
of first year nursing students. Which of the following information regarding diet will the nurse include in the
presentation?
A. take three healthy meals a day to prevent hypoglycemia and increase feeling of well-being
B. include simple sugars in the diet to prevent low blood glucose levels which cause mood swings
C. drink 2000ml of fluid (water, coffee, tea) per day to flush the kidneys and improve fluid retention
D. decrease intake of caffeine such as coffee and chocolate to minimize irritability, insomnia and
anxiety

1004. Hepatitis C complication include ; liver cirrhosis , Liver cancer, Liver failure

1005. A diabetic mellitus patient had left low-knee amputation. The nurse in the surgical unit has been doing wound
care for three days. The nurse evaluated the wound's condition to review the care plan. Which sign shows poor
glucose control in wound healing?
A. Swelling of the stump
B. Bleeding of the stump
C. Mild redness of the wound site
D. Separation of the wound edges

1006. When examining the tympanic membrane of adult by otoscope, the nurse should move the ADULT pinna in
which direction?
A. Down and forward
B. Up and back
C. Up and forward
D. Down and back

1007. When examining the tympanic membrane of infant by otoscope, the nurse should move the Infant pinna in
which direction?
A. Down and forward
B. Up and back
C. Up and forward
D. Down and back

1008. A client has just returned to a nursing unit after an above-knee amputation of the right leg. The nurse should
place the client i which position?
A. Prone
B. Reverse Trendelenburg's
C. Supine, with the amputated limb flat on the bed
D. Supine, with the amputated limp supported with pillows

1009. patient with ongoing blood transfusion and the nurse observe that patient has difficulty to breathing and
shivering what the nursing priority
A. stop infusion
B. call physician
C. slow rate of infusion
D. give Intravenous fluid
ِ‫أوليسِيُجزىِ ُمحسنٌ ِاالِباحسان‬

1010. A 70 years old man is brought to the emergency department on a wheelchair. He complains of drowsiness,
fatigue and lack of appetite for the past three days. With history taking, the patient stated that he did not
eliminate since five days and he is not ambulating a lot because he has arthritis in both knees. Why is
constipation one of the common problems for immobilized patients?
A. decreased peristalsis
B. increased colon motility
C. an increased defecation reflex
D. decreased tightening of the anal sphincter

1011. Tympanocentesis is a minor surgical procedure that refers to puncture of the


tympanic membrane with a small gauge needle in order to aspirate fluid from the
middle ear

1012. Otitis media surgery is myringotomy

1013. Suspect patient with H1N1 what is the highest priority nursing action ?
A. Isolate the patient in private negative pressure room
B. Take nasopharyngeal swab

1014. MAP (mean arterial pressure) definition : average pressure in a patient's arteries during one cardiac cycle

1015. A home health nurse visits a patient who is newly diagnosed with diabetes. The patient is compliant with
taking the prescribed hypoglycemic medications and eats three meals a day followed by desserts sweetened
with granulated sugar. The patient also exercises 30 minutes a day, three times a week. (See lab results)
Test Glucose HgbA1c Result Range from 6.6-8.36.9%Normal 3.9-5.5mmol/L 4.7-5.6% Which of the following
educational intervention takes priority?
A. glucoses Monitoring
B. dietary requirements
C. exercise regimen
D. medication

A Knowledge deficit related to diabetes mellitus


B Anxiety related to new diagnosis of diabetes mellitus
C Altered nutrition more than body requirement
D Altered health maintenance related to elevated blood glucose
E Altered in fluid and electrolyte balance to ketoacidosis
F Risk for infection related to chronic disease
1016. An insulin dependent diabetic patient is admitted to the hospital in ketoacidosis. Of the nursing diagnoses
shown in the table which is the two with the HIGHEST priority?
A. A, C
B. E, F
C. D, E
D. D-B,E

1017. A30 years old married man presents to the clinic with complaints of feeling sad for the past three months. He
is unable to maintain regular sleep routine; he lost his appetite and has difficulty concentrating. He is prescribed
a medication which prevent the reuptake of specific neurotransmitters that could contribute to his mental
health problem. Which side effects would be most important for the nurse to advise the patient of?
A. polyuria
B. photophobia
ِ‫أوليسِيُجزىِ ُمحسنٌ ِاالِباحسان‬
C. fluid retention
D. sexual dysfunction

1018. A surgical unit nurse assessed a 35 years old post appendectomy observe abdominal distention with absent
bowel sounds. Which of the following interventions is most appropriate?
A. Encourage ambulation
B. Provide liquid diet as tolerated
C. Ensure patency of nasogastric tube
D. Check surgical site for signs of infection

1019. The nurse assesses a client with an ileostomy for possible development of which of the following acid-base
imbalances?

A. Respiratory acidosis
B. Metabolic acidosis
C. Metabolic alkalosis
D. Respiratory alkalosis

1020. Postpartum perineal assessment acronym is REEDA in which letter A mean ?


A. Approximation ( how well edges come together )

1021. Which food consider low potassium diet ?


A. Bananas
B. Avocado
C. Berriers
D. Strawberry

1022. Which one of the following signs and symptoms is associated with increased intracranial pressure?
A. restlessness and confusion
B. bradycardia and hypertension
C. tachycardia and hypotension
D. respiratory depression and headache

1023. The Code of Professional Conduct is produced and published by:


A. The Nursing and Midwifery Council
B. The Royal College of Physicians
C. The Royal College of Nursing
D. The Department of Health

1024. During a prenatal examination. The nurse draws blood from a young Rh negative client and explain that an
indirect Coombs test will be performed to predict whether the fetus is at risk
for:
A. acute hemolytic disease
B. respiratory distress syndrome
C. Protein metabolic deficiency.
D. pathologic hyperbilirubinemia

1025. Excess iodine affects with organ first?


A. Brain
B. Heart
C. Liver
D. thyroid
ِ‫أوليسِيُجزىِ ُمحسنٌ ِاالِباحسان‬
1026. 46-year-old patient is in the male Urology Ward after the surgical removal of the stone from his left kidney
through percutaneous nephrolithotomy under general anesthesia. He has nurse and dull acting pain in left
lumbar region. His nephrostomy bag ia attached through atube in his left kidney for a few days ( see image)
What findings should alert the nurse to report to the physician
immediately?
A. Abdominal discomfort and conniption
B. Severe pain and discomfort at surgical site
C. Presence of blood and stone gravels in urine
D. Urine output less than the identified amount

1027. A 6-year-old child was admitted to pediatric medical ward with acute glomerulonephritis. Which of the
following is an indication of acute glomerulonephritis?
A. Pain in urination
B. Frequent urination
C. Difficulty in urination
D. Pharyngitis 15 days ago

1028. while school nurse was teaching a group of 14 years old children one of them remark 'you are too young yo be
our teacher! you are not much older than we are how should the nurse respond?
A. How do you think I am
B. We need to stay focused on the topic
C. Do you think you can teach it any better
D. I think I am qualified to teach this group

1029. A 9 A cardiac monitor for a patient in a Coronary Care unit shows abnormal ECG rhythm with heart rate of 159
beats, QRS complex (0.18 second), and absent P wave. What could be the type of possible dysrhythmia?
A. Sinus tachycardia
B. Ventricular tachycardia
C. Ventricular fibrillation
D. Supraventricular tachycardia

1030. 9year-old child has a fractured femur and full leg cast has been Which of the following is a physiologic effect of
immobilization?
A. A Venous stasis
B. B Increase metabolic rate
C. Positive nitrogen balance
D. D Increased need for oxygen

1031. A community health nurse screens a group of high-risk adults for tuberculosis Which gauge needle should the
nurse use for an intradermal injection on the volar surface of the forearm?
A. 16 gauge needle
B. 20 gauge needle
C. 22 gauge needle
D. 26 gauge needle

1032. The maximum time for suctioning in adults is:


A. 5 second
B. 8 seconds
C. 10 second
D. 15 second

1033. A registered nurse in the Intensive care Unit performed Glasgow scale assessment on a 50-year-old man who
sustained head 24 hours ago. The recorded Glasgow Coma score was 10.
which one of the following actions should be taken immediately nurse?
A. Inform the registered nurse in charge of the nursing
ِ‫أوليسِيُجزىِ ُمحسنٌ ِاالِباحسان‬
B. Protect the patient by raising the two side rails
C. Inform operation theatre for emergency surgery
D. Perform neurologic assessment every 15 minutes

1034. 3-week-old newborn is diagnosed with Hirschsprung disease Which of the following preoperative care should
included in the plan of care?
A. Restricting oral intake to clear fluids
B. Administering a tap water enema
C. Inserting a gastrostomy tube
D. Using povidone-iodine to prepare the perineum

1035. A 10 year old child has very limited vocabulary and interaction skills . she has an IQ of 45 , she is diagnosed to
have mental disease retardation of this classification ?
A. Profound
B. Mild
C. Moderate
D. Sever

1036. 8-year-old child was admitted to the pediatric ward diagnosed with B-thalassemia. The nurse is planning to
give health education to the mother. Which of the following educational needs is recommended for the mother?
A. compliance to hydroxyurea
B. compliance to hemosiderosis
C. compliance to disferal
D. compliance to Iron supplement

1037. A patient who is admitted denies any medication allergies. medical records state, "No known drug allergies." A
medic been sent to the pharmacy for gentamicin. The pharmacist record in the pharmacy of an allergy to the
medication. Which of the following is the best way for the nurse to pro
A. Administer the medication with Benadryl (Diphen
B. Notify the doctor of the allergy and request new orders
C. Investigate further then call the doctor for clar
D. Give the medication as ordered by the physician

1038. Which of the following is the best nursing action to ensure safe medication administration the nurse feels the
dosage is unsafe for the patient ?
A. Call pharmacy
B. Call the head nurse
C. Contact the prescribing doctor
D. Administer half of the dosage

1039. Which of the following consider contraindication of tonsillectomy ?


A. Child with adenoid infection more than 4 times per year
B. Child with tonsillitis more than 4 times per year
C. Child age less than three years old
D. Child with hypertrophied adenoids

1040. A child has been seizure free for 2 years . A mother asks the nurse how much longer that the child will need to
take the ant seizure medications
A. Medication can be discontinued at this time
B. The medication dosage can be reduced gradually
C. New medication can be order after one year
D. Medications can be discontinued after 5 years
ِ‫أوليسِيُجزىِ ُمحسنٌ ِاالِباحسان‬
1041. A teenage male patient with bipolar disorder is in the psychiatric ward for a week. he is speech is incoherent
and is complaining vision and hearing problems
Which of the following essential assessment need monitoring and reporting?
A. Visual and auditory hallucinations
B. Frequency and span of mood swings
C. Level of agitation and speech difficulty
D. Medication regimen and expected side effects

1042. Child with cerebral palsy was brought to the out patient department for a monthly follow up visit. One of the
strategies to maximize the growth and development of the child during feeding is to maintain a pleasant and
distraction and distraction free environment , which of the following instruction should be recommended to th
mother ?
A. Serve food that will no stick to the spoon
B. Place the child in a sitting position during eating
C. Encourage child to eat with other children
D. Encourage finger foods that child can handle alone

1043. A nurse is preparing to transfer a patient from the operating table to the postoperative bed. The patient had a
right total hip arthroplasty . the nurse must maintain the patient hips of the following position ?
A. Flexion
B. Abduction
C. Extension
D. Adduction

1044. A 62 year old diabetes type II patient is admitted to hospital for amputation of the right foot due to gangrene
and ischemia. Preoperative , the patient rates the pain as 9 using the 1 – 10 pain sacle ) how can this patient
phantom pain best be controlled?
A. Post operative elevation of limb
B. Apply pressure bandage to stump
C. Control pain pre operatively
D. Apply ice to site for twenty minutes

1045. A diabetic patient on subcutaneous insulin needs to rotate the site of injection to prevent which of the
following?
A. Insulin intolerance
B. Insensitivity to insulin
C. Increased rate of absorption
D. Hard painful lumps development
1046. A nurse is making an assessment for the manifestation of cyanosis in a dark-skinned client . which of the
following sites is the best for the nurse to check for presence of cyanosis
A. Lips
B. Palms
C. Nail bed
D. Conjunctiva

1047. A nurse was assessing a newborn undergone circumcision . What is the immediate risk for newborn the nurse
to observe ?
A. Hypospedias

1048. What the rationale of the initial postpartum assessment of the mother?
A. Determine of the uterus is well contracted
B. Monitor amount and color of the lochia
C. Obtain vital signs and determine pain level
D. Determine whether she is physically stable
ِ‫أوليسِيُجزىِ ُمحسنٌ ِاالِباحسان‬
1049. Position after head trauma ?
A. Prone
B. Elevate head of bed 30degree to decrease intracranial pressure

1050. When a patient tell his nurse “ I need to maintain medication compliance to avoid relapse “and nurse replies
“this correct” which of the following therapeutic communication techeniques being used ?
A. Focusing
B. Exploring
C. Accepting
D. Reflecting

1051. Incentive spirometry is ?


A. Use to prevent atelectasis after surgery

1052. A nurse is inserting a nasogastric tube for a 65-year-old patient with paralytic ileus Immediately after insertion
200 ml of milky appearing drainage was noted in the bag. The nurse is not sure if the distal end of the tube is in
the stomach. What is the most appropriate technique confirm positioning?
A. X-ray of stomach
B. Check pH of drainage
C. Aspirate more fluid and check if color change
D. Flush with 100 ml saline and auscultate over stomach

1053. patient who had aortic valve replacement with a mechanical heart valve has just completed discharge
teaching provided by a nurse on duty. Which statement by the patient indicates the effectiveness of the nurses
teaching?
A. "I am glad that I can continue taking my Gingko Biloba"
B. "I will increase my intake of green and yellow vegetables"
C. . "I will start applying vitamin E to my chest inaision after showing"
D. "I will shave using an electric razor from now on"

1054. Height of enema is ?


A. 50 cm
B. 150 cm
C. Blow red edge

1055. A 67-year-old patient was brought to the Emergency Room with history of diabetes mellitus, hypertension and
heart failure complaining of acute shortness of breath and productive cough. The sputum was pink and frothy.
During the assessment, the nurse noticed crackles and wheezes. Blood pressure 80/45 mmHg Heart rate
124/min 36/min Respiratory rate 36.5°C Temperature Which of the following disorders should the nurse
suspect?
A. Pneumothorax
B. Uns0table angina
C. Pulmonary edema
D. Pulmonary embolus

1056. A 62 year-old diabetes type II patient is in the post-operative care unit following amputation of the right foot.
Post-operatively, the patient tells the nurse of tingling, coldness and cramping in the right foot. Which of the
following describes best the type of pain?
A. Visceral
B. Somatic
C. Ischemic
D. Neuropathic
ِ‫أوليسِيُجزىِ ُمحسنٌ ِاالِباحسان‬
1057. A 16-year-old man is in the Emergency Department with S Ose bleeding as a result of falling down while
climbing up the wall. On assessment, his nose appears slightly deviated and 2 swollen. He is breathing from his
mouth What sign helps evaluate that his nose is still bleeding
A. Complain of sharp pain in the nasal bone
B. Experiencee more difficulty in breathing
C. patientt is swallowing frequently
D. Increased swelling of the nose

1058. 25-year-old woman has a family history of breast cancer. The nurse reviews the procedure for breast self-
examination (BSE) and tells her that the best time for a woman to perform a breast self-examination is:
A. a few days before her period.
B. during her menstrual period.
C. on the last day of menstrual flow.
D. 3- 7days after the beginning of her period

1059. The nurse is instructing a female client how to do breast self exam. Which of the following is the best time to
perform this exam?
A. After ovulation
B. After period
C. Two weeks after period
D. Three days before period

1060. Mass media disadvantage ?


A. One way communication
B. Two way communication
C. Large number of people

1061. A nursing is given education to a patient with blood pressure 170/95 to avoid eating of which of the flowing ?
A. Canned food
B. Protein
C. Vegetables and fruits
1062. A nurse is giving health education to 32 years old female to prevent uterine prolapsed ?
A. Kegel exercise

1063. A patient has depression and on antidepressants medication .while the nurse caring him he asked ( how long
the medication takes to reach the effectiveness )The proper nurse response is?
A. 3-4 days
B. 10-4 days
C. 14-18 days
D. a month

1064. A 25 years client is admitted to the emergency department with sudden onset of right lower abdomen pain.
Which of the following physician orders should the nurse question at this time?
A. Apply heating pads to Abdomen
B. Obtain abdomen x ray
C. Start intravenous dextrose 5%
D. Nothing by mouth

1065. In anticipate of further emergency treatment for a client with salicylate overdose, which of the following
medications should the nurse have available?
A. Vitamin K
B. Atropine sulfate
C. Dextrose 50%
D. Sodium thioslfate
ِ‫أوليسِيُجزىِ ُمحسنٌ ِاالِباحسان‬
1066. Contraindication immunization for a patient who is receiving which medication ?
A. Steroids
B. Insulin
C. Midazolam

1067. Radon cancer mostly affected ?


A. Lung
B. Cervical
C. Prostate

1068. Radon cancer mostly affected ?


A. pancreas
B. Cervical
C. Prostate

1069. Post MI medication ?


A. Heparin
B. Aspirin
C. Warfarin

1070. A nurse researcher wrote in her explanatory statement: "The researcher will - 35 Use a questionnaire to
measure nursing job satisfaction," which ethics principle is the researcher addressing here?
A. Justice
B. Beneficence
C. Confidentiality
D. Non - maleficencge

1071. A 3-year-old child is admitted to the Medical Ward for vomiting, and dehydration. The nurse sat with the
parents to comply admission interview and wanted to get as much information as Which of the following
communication techniques should the nurse
A. Use of question containing the word "how"
B. Use of question with direct comments to clarify
C. Use of statements that indicate patient will be fine

1072. A 68-year-old diabetic women undergoes a below knee amputation to vascular insufficiency and infection. On
admission to care unit, the nurse makes the priority diagnosis: Risk for ineffective peripheral tissue perfusion.
Which intervention is most appropriate?
A. Ensure adequate pain relief
B. Elevate residual limb on a pillow
C. Administer oxygen by facemask
D. Apply ice to the stump

1073. A nurse plans to teach a group of 20to25-year-old women about oral contraceptives. The nurse should instruct
that oral contraceptives may:
A. Increase the risk of pelvic inflammatory disease
B. Cause acne to worsen
C. Decrease the risk of breast and cervical cancer
D. Decrease the risk of endometriosis

1074. A nurse is working in a medical unit and has been assigned to care for 65-year-old man with cerebrovascular
accident, which caused right-sided hemiparesis. The contractures due to immobility. is worried about the patient
developing muscle contracture nurse most important to be Which of the following nursing actions
implemented? is
A. Provide diet rich in protein
B. Turning the patient every 2 hours
ِ‫أوليسِيُجزىِ ُمحسنٌ ِاالِباحسان‬
C. Provide range of motion exercises
D. Apply moisturizing cream on the affected side

1075. A patient is scheduled for a magnetic resonance imaging (MRI) scan for suspected lung cancer. Which of the
following is a contraindication to the study for this patient?
A. The patient is allergic to shellfish
B. The patient has a pacemaker
C. The patient suffers claustrophobia

1076. A 45 year-old patient is admitted in the Intensive Care Unit and has been on a mechanical ventilator for five
days. Which weaning parameter indicates the patient has enough muscle strength to breathe?
A. Normal electrolytes
B. Positive fluid balance
C. Negative inspiratory force
D. Increased minute ventilation

1077. A nurse is inserting a nasogastric tube for a 65-year-old patient with paralytic ileus Immediately after insertion
200 ml of milky appearing drainage was noted in the bag. The nurse is not sure if the distal end of the tube is in
the stomach. What is the most appropriate technique confirm positioning?
A. X-ray of stomach
B. Check pH of drainage
C. Aspirate more fluid and check if color change
D. Flush with 100 ml saline and auscultate over stomach

1078. Which of the following patient education is the most important in case of eye infection?
A. Cover eye for protection
B. Apply warm pads
C. Wash face two times a day
D. Take more fluids and eat balanced diet

1079. A client is receiving mechanical ventilation when the alarm sounds and displays an alert showing low tidal
volumes. All connections and the ET tube are checked and secured but the alarm persists while O2 saturation
continues to drop. Which of the following is the most appropriate initial action?
A. Increase oxygen delivery to 100%
B. Manually ventilate through the endotracheal tube
C. Call the respiratory therapist to assess the patient
D. Elevate the head of the bed and apply a non-rebreather mask

1080. A nurse provides care for a client who is 5 days post gastrectomy. The provider prescribes total parenteral
nutrition (TPN). Which statement by the nurse to an orienting nurse explains the client's need for TPN?
A. TPN is used for clients who are at increased risk for aspiration due to swallowing inability.
B. TPN is for clients who are unable to receive nutrients by mouth for an extended period of time.
C. TPN is the usual method of providing nutrition to clients after major abdominal surgery.
D. TPN is a temporary solution and can be used for the first 24 hours after surgery

1081. Vitamin A deficiency cause ?


A. Night Blindness

1082. post-operative patient who underwent an abdominal procedure request pain medication from the nurse
and rates the pain at a level nine. There a standing order for narcotic administration. When the nurse opera the
narcotic box and performs a count, the number of pills remaining in the box is different than the number of pills
recorded on the sheet. What is the most appropriate initial nursing action?
A. Notify the nursing supervisor
B. Write the finding on the narcotic sheet
C. Administer the patient's requested medication
ِ‫أوليسِيُجزىِ ُمحسنٌ ِاالِباحسان‬
D. Identify the last nurse who used the narcotic

1083. A 58 years old man had a permanent night pacemaker inserted. The device is in place and functioning well.
The skin at the incision site is intact and free of infection. What is the most important nursing care outcome?
A. understand functioning and operating of a pacemaker
B. monitor pulse rate and identify need for reporting
C. able to identify the signs of detached pacemaker
D. modify lifestyle and wear pacemaker alert sign

1084. An infection control nurse is responsible to conduct the surveillance on how the hospital staff take
precautionary measure against flu, based on her findings, she is responsible to conduct the awareness rating
sessions for doctors and nurses in the patient areas.
which of the following is the goal of nursing practice here?
A. infection control management
B. research and survey
C. primary prevention
D. health education

1085. Which of the following obstacles up to the implementation of the primary health care strategy weds
A. Increase the number of health care workers
B. Misinterpretation of PHC concept
C. Decentralization management
D. Few political supervision

1086. The nurse listens to patient complains and discussed how each of the complaints can be managed which of the
following is the best description the nurse action ?
A. Promoting patient autonomy
B. Demonstrating sympathy
C. Ensure patient support
D. showing concerns

1087. a nurse on unite to conduct a research project what is the most important step to be identified ?
A. research problem
B. research setting
C. analysis plan
D. sample size

1088. A 16 years old girl admitted to inpatient unit accompanied by her mother who supports her to walk . she
looked very weak and held her head up with her hands . she diagnosed with anorexia nervosa . what is the
primary nursing diagnosis?
A. Risk for injury related to low potassium
B. Body image disturbance related to obesity
C. Imbalance nutrition relate to compulsive overeating
D. Imbalance nutrition related to restricting caloric intake

1089. A nurse is conducting a fire safety class and she was explaining that elderly are twice was likely to die at home
from fire accidents compared to younger people therefore the nurse has suggested some safety measures
What should the nurse suggest to prevent fire accidents at home?
A. Have a smoke detectors installed in the houses
B. Invite a family member to sleep with the elderly at nigh
C. Encourage practicing evacuation I case of emergency

1090. What is the major task of the staff nurse during a code blue? )
A. Direct the code
ِ‫أوليسِيُجزىِ ُمحسنٌ ِاالِباحسان‬
B. Intubate the patient
C. Make treatment decision
D. Coordinate use of the crash cart

1091. Contraindication immunization for the patient who is received which medication ?
A. Steroids
B. midazolampl
C. insulin

1092. colonized infection is …germs are on the body but do not make you sick

1093. Isoniazid side effect ?


A. Hepatitis symptoms
B. Blindness

1094. A43-year-old woman fell and hit her head. She was admitted to the hospital and put on observation for a
possible closed head injury. The patient's orders include hourly checks for increasing intracranial pressure and
nursing interventions to reduce intracranial pressure.during the assessment, the nurse notes the intracranial
pressure is 16 mmhg . Which is the most appropriate nursing Intervention?
A. Provide an intravenous fluid bolus
B. Position patient in semi-fowler's
C. Prepare for hypothermia induction
D. Hyperventilate with positive pressure

1095. A 46-year-old man diabetic patient is admitted to the surgical department because he has gangrened leg due
to uncontrolled blood sugar and contamination of leg ulcer. The admission plan is to do under knee amputation
of the left leg. The nurse is explaining to the patient that this will cause a permanent disability.
Which of the following statements is the best description for permanent disability?
A. Problems in body function and loss
B. Difficulties to experience life situations
C. Difficulties in executing activities
D. Loss of interaction with community

1096. A 35-year-old woman presents to the clinic with complaints of itching and pain in the eyes. there are large
amounts of watery discharge and mucous from both eyes. The whites of the eyes are pink-colored and irritated.
the doctor orders a prescription for eye drops that contain prednisolone. The nurse educates the patient on
correct administration. Which advice is most important?
A. Irrigate the eye with saline before administration
B. Continue taking medication until the bottle is empty
C. Prior to administration block tear ducts with fingers
D. Apply to the inner aspect of conjunctival sac

1097. Diet for patient with cancer ……………… fresh fruits and vegetables

1098. Diagnostic test for sickle cell anemia before birth is Amniotic fluid aspiration sample

1099. A nurse is assigned to care for a patient with a diagnosis of thrombotic stroke. The nurse knows that this type
of stroke is MOST LIKELY caused by:
A. Blockage of large vessels as a result of atherosclerosis
B. Emboli produced from valvular heart disease
C. Decreased cerebral blood flow due to circulatory failure
D. A temporary disruption in oxygenation of the brain

1100. During hospital admission of a homeless patient with mental illness , the nurse tells him that he needs an
immediate shower because he is smelly and looks horrible which o the following explains the nurse action?
ِ‫أوليسِيُجزىِ ُمحسنٌ ِاالِباحسان‬
A. Judgmental
B. Responsible
C. Professional
D. Empathically

1101. Which of the following interventions is appropriate for the diagnosis of risk for injury of a patient that has a
pacemaker?
A. Offer back rubs to promote relaxation
B. Instruct patient in dorsiflexion exercise of ankles
C. Have patient avoid exposure to magnetic resonance imaging
D. Observe incision site for redness purulent drainage warm soreness

1102. Identification and quantification of health problem in a community as a whole in term of mortality and
morbidity. Which of the following explains this phrase?
A. Diagnosis
B. Assessment
C. Participation
D. Involvement

1103. A 3 year is brought by the mother to the emergency Department with fever diarrhea and vomiting. She
passed four loose motions and three vomiting the last 24 hours , she is anorexic, irritable , has dry lips and
moderate skin turgor. She is given ORS to drink, but she refused it after the first stip.
O2 sat 96 HR 36 TEM 38.8
What is the immediate nursing intervention is required to encourage the baby to drink the ORS?
A. Give In a cup once cold a day
B. Help her to drink with a syringe
C. Give in a small amount frequently
D. Engage in playing and help to her drink it
1104. A 43 year old man is 30 hours post-operative following placement of a partial thickness skin auto graft for a
burn injury on the lower anterior leg. During routine assessment, the nurse notices the wound is bleeding
continuously. Which of the following is the nursing action?
A. Use a pen to outline and monitor area
B. Perform a wound swab for laboratory analysis
C. Incise and drain fluids from the wound bed
D. Apply firm and direct pressure for 10 minutes

1105. The charge nurse is evaluating the skills of a newly registered nurse who is providing care to a client with
shock. Which action taken by the new nurse would most likely indicate the need for further instruction?
A. Administers furosemide to a client in cardiogenic shock with a pulmonary artery wedge pressure
of 24 mmHge
B. Increase norepinphrine infusion rate to maintain arterial pressure greater than 65mmHge in a
client with anaphylactic shock
C. Moves pulse oximeter sensor from the finger to the forehead of a client with septic shock
D. Place the head of the bed in high fowler position for a client with hypovolemic shock

1106. A 28 year old man is recovering from a moderate concussion following a motor vehicle accident two week
ago, when he suddenly develops an increased thirst, craving cold water, the patient urinated very large amounts
of dilute, water like urine with a specific gravity of 1.001 to 1.005.
ِ‫أوليسِيُجزىِ ُمحسنٌ ِاالِباحسان‬
Which of the following is the patient most likely developing ?
A. Diabetes mellitus
B. Diabetes insipidus
C. Hypothyroidism
D. Thyroid storm

1107. A25 years old patient telephone the nurse in the day surgery unit and complains of intense pain in the
frontal lobe. Two days previously, she had undergone an elective surgical procedure that had required a spinal
anesthetic block. Which of the following remedies is the most alleviate the patient symptoms?
A. 1000ml fluid intake per day
B. Decrease exposure to light
C. Lie down in a flat position
D. Acetaminophen 200 mg by month

1108. A 32 year old man is admitted to the isolation unit with acquired immune deficiency syndrome. He is
married and has one child. His wife is a depressed because of his diagnosis.
Which of the following nursing problems requires immediate planning?
A. Risk of infection due to autoimmune cell destruction
B. Nutrition deficit due to due body cell mass wastage
C. Risk of ineffective ineffective psychosocial and family support
D. Knowledge deficit about prognosis and treatment

1109. A 56- year old man with history of COPD is complaining of chest, shortness of breath , lethargy, fever and
productive cough. Upon examination, crackles could be heard in the lower lobes
BP 11./70 HR 111 RR 18 TEM 37.4 Oxygen Sat 90%
A. Prone
B. Supine
C. High fowler
D. Trendelenburg

1110. A 55 year patient stats states that he has difficulty in sleeping in the hospital because of the noise. The nurse
kept trying reassuring the patient but still cannot sleep.
To assist an adult patient to sleep better the nurse recommends which of the following?
A. Eating a large meal one hour before bedtime
B. Consuming a small glass of warm milk at bedtime
C. Performing mild exercise 30 minutes before going to bed
D. Administer a sedative, as ordered by physician

1111. A 47 year patient received spinal anesthesia five hours ago in the operation theatre and was transferred to
the surgical unit. Few hours later the patient complains of incisional pain. The surgical site is dry and intact but
the patient looks pale irritated.
BP 175/90 HR 112 RR 27 TEM 37.6
What is the most appropriate nursing intervention ?
A. Take a 12 lead ECG
B. Administer pain medication
C. Notify treating physician
D. Strat IV infusion and place patient in supine
ِ‫أوليسِيُجزىِ ُمحسنٌ ِاالِباحسان‬
1112. A patient is being followed in the clinic for hypertension, adult onset diabetes and obesity. The patient is
apathetic about the learning about nutrition guidelines to reach the goals of weight loss and consumption of a
healthy diet . the patient admitted to eating " whatever is put in front of me". which of the following actions
would the nurse take?
A. Collaborate with the patient to set goals
B. Add a nursing diagnosis of non- compliance
C. Refer to psychiatric screening for depression
D. Discuss nutritional interventions with the spouse

1113. A 60 year old man is admitted to the male medical ward with coronary artery disease. The patient has a
history of hypertension, stress high cholesterol, diabetes and smoking. His father died of myocardial infraction .
Which of the following are modifiable risk factors for coronary artery disease?
A. Age and gender
B. Gender and diabetes
C. Family history and smoking
D. Smoking and high cholesterol

1114. A nurse completes discharge instructions for a patient who was admitted five days ago with pneumonia.
Which of the following statements by the patient would alert the nurse that more discharge teaching is needed?
A. I need to gradually increase my actives
B. I need to have another chest X-ray in 4-6 weeks
C. I will not need the infuenza or pneumonia vaccine
D. I may experience fatigue and weakness for a prolonged time

1115. A 45 year old man is admitted to the surgical ward for stoma. The stoma appears a dark red purple color, is
slightly swollen and leaks scant amount of blood. The is no presence of stool around or on the site. Which
observation requires immediate attention?
A. Color
B. Edema
C. Absence of stool
D. Presence of blood

1116. During her break time, the nurse was eating in the hospital cafeteria when she noticed a female colleague
grasping her neck and unable to speak . which of the following maneuvers is the best to be performed by the
nurse?
A. Epley
B. Heimlich
C. Valsalva
D. Leopold's

1117. Patient with meningitis after treatment with effective MCV4. Which of the following confirm that was
effective ?
A. CBC
B. Negative brudzinskis signs

1118. Which of the following medication types is given rectal ?


A. Tablet
ِ‫أوليسِيُجزىِ ُمحسنٌ ِاالِباحسان‬
B. Capsule
C. Injection
D. Suppository

1119. A 45 years old man admitted with second degree burn injury of 10% in the left arm due to hot water spillage.
During assessment, the nurse would assess the general health of patient and include risk factor for wound
healing process. Inflammatory process of wound healing with which of the following?
A. Vasoconstriction of blood vessels
B. Antibody reaction towards antigen
C. Redness due to vessel dilation
D. Migration of macrophages to ingest the microorganism

1120. A 56 year old patient, who is post triple coronary artery bypass surgery, remained intubated for two days.
While assessing the patient. Body temperature, which of the following routes the nurse must exclude to avoid
cardiac dysrhythmia?
A. Skin strip
B. Axillary
C. Tympanic
D. Rectal

1121. The doctor ordered to give nitroglycerine sublingual to a patient with angina. What is the most likely effect
that drug has on the patient body?
A. Increase heart rate
B. Increase blood pressure
C. Increase blood supply to the heart
D. Increase myocardial oxygen consumption

1122. A 26-year old patient came for lithotripsy of her kidney. She was accompanied by her husband. Her vital
signs were stable and was overweight. Her investigations released her pregnancy test as positive and were two
moths pregnant.
Which of the following action the nurse should take for this patient?
A. Prepare for physical examination
B. Hold procedure and inform
C. Have the consent signs
D. Prepare for lithotripsy

1123. A 66 year old woman was admitted to the hospital with a history of hypertension he presents with breathing
difficulties that worsen with activity and while sleeping. He is weak and feel that her heart misses beats. An
electrocardiogram confirms atrial fibrillation, right ventricular hypertrophy and deviation towards the right.
Which of the following is the most likely origin of the disorder?
A. Hypertension
B. Rheumatic fever
C. Atherosclerosis
D. Genetic predisposition

1124. A female teenager was diagnosed with sickle cell anemia informs a nurse that her school arranged a hiking
activity in nearby mountains. She shows excitement while forming the nurse about her new water bottle for this
trip? What is the appropriate initial response by the nurse?
ِ‫أوليسِيُجزىِ ُمحسنٌ ِاالِباحسان‬
A. What do your parents think?
B. You realize that this type of activity might cause a pain crisis
C. You need to get approval from your doctor before attending this activity
D. It is great you are hydrating yourself in preparation of this activity

1125. A 35 years old patient was admitted to the medical ward through emergency department accompanied by
her . her chief complains include severe epigastric pain, abdominal tenderness and distension for the last 24
hours . she was anorexia and had passed sex watery diarrhea since few hours. She was feeling lethargic due to
frequent elimination action is required?
A. Provide fluids to drink
B. Arrange soft diet
C. Check vital signs
D. Inform physician

1126. A 36 year old man present to the hospital with complaints of breathing difficulties and fever. The patient had
undergone a liver transplant one year before upon assessment a yellowish colored skin was noted on entire
body
HR 102 RR29 BP 110/80 TEM 39.2 HB 11 WBCS 4 RBS 3.5
Which complication is most likely to occur?
A. Bleeding
B. Embolism
C. Superinfection
D. organ rejection

1127. A 23 year old patient was admitted to evaluate pyrexia of unknown origin. According to her mother, her
fever spiked every night to 38.5 until morning for the last two weeks. The patient was quiet, uninterested in her
surroundings, had language barrier and her personal hygiene was not maintained. At lunch time , she refused to
eat the hospital food , but her yelled at her in front of others and forced her to eat . what basic need of the
patient is disturbed according to maslow hierarchy ?
A. Self esteem
B. Self-actualization
C. Safety and security
D. Food water and shelter

1128. A 42 year old man with thalassemia received a packed cell transfusion. The nurse assess the stability of his
condition after transfusion by monitoring the vital signs and general condition every two hour. When should the
nurse immediately report the patient condition ?
A. Sever headache and raised blood pressure
B. Raised body temperature and flushed skin
C. Joint pain, body ache listlessness
D. Restlessness & bradycardia

1129. A patient with sever varicose veins of the left leg presents to the clinic. The patient states that three days
ago the right leg became very swollen and the skin on the right half area was very darkly colored. The capillary
refill in the fingers is three seconds and on the right toes is four seconds.
A. Teach the patient self blood monitoring
B. Assess the patient to sided heart failure
C. Encourage the patient to joint an exercise class
ِ‫أوليسِيُجزىِ ُمحسنٌ ِاالِباحسان‬
D. Teach the patient how to apply thigh high anti embolic stocking

1130. When caring for a client who has acute respiratory distress syndrome, a nurse elevates the head of the bed
to 30 degrees. What is the reason for this intervention?
A. Reduce abdominal pressure on the diaphragm.
B. Promote bronchodilation and effective airway clearance
C. Decrease pressure on medullary centre which stimulates breathing
D. Promote retraction of intercostal accessory muscle of respiration.

1131. A nurse need to take the temperature of an adult comatose patient his rectum. How far into the rectum
should the thermometer be inserted reliable temperature?
A. 1.5 cm
B. 2.5 cm
C. 3.5 cm
D. 4.5 cm

1132. A 60 year old man is recovering in a hospital ward three days after hip arthroplasty. He has not had a bowel
movement since the operation and complains of feeling bloated up on examination reveals abdominal
distension and hypoactive bowel the patient rate his pain at a level 7 at rest and 9 with movement pain scale of
1-10. He is taking opioid analgesics. The nurse care plan based on diagnosis altered bowel elimination surgical
intervention as evidence by lack of bowel movement. Which of the following is the most appropriate
intervention?
A. Request an increased opioid dosage
B. Perform passive range of motion exercise
C. Encourage increase fluid intake
D. Provide soft foods based on bland diet

1133. A 40year old client underwent and exploratory laparotomy with anesthesia. An assessment of abdominal 36
hours postoperative showed abdominal distension and an absent of bowel sound. Which complication is most
likely?
A. Paralytic ileus
B. Hemorrhage
C. Rupture colon
D. Intussusception

1134. A patient with carpal tunel syndrome ( CTS) is being prepare surgery . which of the following appropriate to
delegate to a assistant?
A. Assess wrist and hand of patient for discoloration
B. Assist patient with self care activities such as bathing
C. Check digits of the hand for painful tingling sensation
D. Initiate application of a splint for immobilization during the

1135. A nurse I the medical surgical unital is review a plan of care elderly client with chronic obstruction
pulmonary disease ( COPD) limited mobility . the nurse notes that the physical therapist change in the plan of
care to progress ambulation from 50 to 100 times a day. Which action in necessary to ensure that the client
need are?
A. Inform physical therapist of client respiratory status before progressing to ambulation
B. Instruct physical therapist not to proceed with ambulation nurse presence
ِ‫أوليسِيُجزىِ ُمحسنٌ ِاالِباحسان‬
C. Inform physician about physical decision to ambulation
D. Cancel referral to physical therapist

1136. The physician writes the following order. Vancomycine 500mg intravenous injection. The infusion is to be
mixed with 200 ml of 0.9% normal saline and administered through a central. The nurse order the medication
from the pharmacy. Which reviewing the chart, the nurse notes the patient has a penicillin allergy.
A. Withhold the medication and notify the physician
B. Suggest an additional order for antihistamine
C. Dilute in 500 ml prior to administration
D. Administer the medication as ordered

1137. A 55 year old underwent total hip replacement. Two hours post. post-operative an orthopedic unit nurse
notes the patient was lethargic and dizzy
Hb9.8 normal 13-17
What nursing intervention is the most appropriate?
A. Start I V fluids
B. Administer paracetamol 1gm iv
C. Call blood bank and request one unit of packed RBS
D. Encourage ambulation to reduce enhance recovery

1138. While evaluating a child in the Emergency Department, the nurse notices that despite. Feeling warm to
touch, the patient has bluish bail beds and bluish lips. Which of the following should the nurse immediately
assess?
A. Respiration
B. Temperature
C. Pulse
D. Blood pressure

1139. A nurse is caring for a client with syndrome of inappropriate antidiuretic hormone (SIDH). Which of the
following should be a priority intervention for the client?
A. Monitoring hourly intake and output
B. Pressure ulcer prevention strategies
C. Encourage client to eat foods rich in potassium
D. Restricting fluid intake of the client to less than 1000ml per day

1140. A 75 year old man is pale and lethargic . he weights 105kg and is bedridden with urine and stool
incontinence. Which of the following is the appropriate nursing diagnosis?
A. Risk for falls
B. Risk for infection
C. Risk for fluid imbalance
D. Risk for impaired skin integrity

1141. A patient, who had abdominal surgery six days ago, has been ambulating in the halls without much difficulty.
however, on seventh postoperative day, the patient complains of increased pain at incisional site and is walking
bent over. What is the most likely cause ?
A. Intestinal inflammation
B. Pulmonary edema
ِ‫أوليسِيُجزىِ ُمحسنٌ ِاالِباحسان‬
C. Wound infection
D. Deep vein thrombosis

1142. A clinical instructor was explaining to his students about the discharge planning. One of the students asked
about the timing of initiating the discharge planning?
A. When patient asks for discharge
B. On admission to acute care setting
C. After successful recovery from surgery
D. When the treating physician agreed to discharge

1143. A nurse is completing the preoperative assessment of the patient who is scheduled for vein ligation and
stripping. The nurse should explain that which of the following is included in the patient postoperative plan of
care ?
A. Apply cold packs
B. Wearing elasticized stockings
C. Ambulating with axillary crutches.
D. Participating in physical therapy.

1144. A patient was admitted to the emergency room due to abdominal pain, upon assessment, positive psoas
signs was noted. The patient was rushed to the operating Room for stat appendectomy. Before a naso gastric
tube postoperatively?
A. Patient feels hungry
B. Upon patient request
C. Flatus repotted by patient
D. Patient consciousness is regained

1145. During the initial assessment, a registered nurse decided to apical pulse of a patient with coronary artery
disease. Which position is best of the patient to be placed in to measure pulse?
A. Prone position
B. Supine position
C. Position on his /her left side
D. Position on his /her right side

1146. Which of the following interventions is appropriate for the diagnosis of risk for injury of patient that has a
pacemaker ?
A. Offer back rubs to promote relaxation
B. Instruct patient in dorsiflexion exercise of ankles
C. Have patient avoid exposure to magnetic resonance image
D. Observe incision site for the redness purulent drainage warm ,soreness

1147. A patient is brought to the emergency department suffering from inhalation up on assessment, a nurse
observe that the patient aggressive and confused
Which of the following is the most likely explanation?
A. Pain
B. Mania
C. Anxiety
D. Hypoxia
ِ‫أوليسِيُجزىِ ُمحسنٌ ِاالِباحسان‬
1148. A patient has a hip fracture complaining of sever pain. He nurse finds no analgesic was ordered. She calls the
orthopedic doctor by telephone and received a medication order.
What is the most appropriate nursing action to ensure correct medication order taken ?
A. Write down the order and get pharmacy o supply
B. Write down the order and get medication immediately with another registered nurse
C. Get another registered to witness, nurse write down the order and read back to the
doctor
D. Write down the medication and after serving get the doctor to witness

1149. A woman patient with , fever, weight loos, and watery diarrhea is being admitted to health care facility.
During assessment the nurse notices the patient abdomen is concave. additional assessment should proceed in
which order?
A. Palpation, auscultation, and percussion
B. Percussion, palpation, and auscultation
C. Palpation, percussion, and, auscultation
D. Auscultation, percussion, and palpation

1150. A nurse responsible for narcotic medication loses on ampule morphine. She reports the problem to nurse
manager. Which of the following is the first thing she should say according to the SBAR approach?
A. I have lost one ampule of morphine
B. We should ask the nurse about the morphine
C. I think I left the lid of narcotics box open
D. We should open an investigation into problem

1151. A 56 year old present to the emergency department experiencing left sides eye discomfort for the past 3
hours, left eye was blurred while vision in the right eye remained examination showed increased intra-ocular
pressure in the left eye pupil of the left also reacted slowly to light. which is the most likely health problem?
A. Detached retina
B. Macular hole
C. Glaucoma
D. Cataract

1152. A patient is admitted to the Emergency Department with broken nose And no other facial fractures or injuries.
What category of surgery would repair a broken nose fall under?
A. Urgent
B. Elective
C. Emergency
D. Immediate

1153. During nursing assessment, the nurse document the presence of skin breaks, abrasion, blister, or sallow
cater, edema , and infection. According to recognized staging system , what is the classification of this pressure
ulcer?
A. Stage 1
B. Stage II
C. Stage III
D. Stave IV

1154. The nurse has attempted to administer a breath to a patient who was found unconscious on the floor. There
was resistance to air entering the patient air way. Which of the following action should the nurse take next?
ِ‫أوليسِيُجزىِ ُمحسنٌ ِاالِباحسان‬
A. Feel for air movement from the patient nose
B. Perform finger sweep in then patient mouth
C. Hyperextended the patient neck
D. Administer abdominal thrusts
1155. Which one of the following is the reservoir for the pathogens causing Hepatitis C?
A. Blood
B. Bowel
C. Bladder
D. Bronchioles

1156. Which of the following can be considered as a major development in nursing home visits in terms of
information technology ?
A. Medication dispensing
B. Telehealth
C. Patient monitoring
D. Prevention of epidemiological disease
1157. Which of the following contraceptive method offers protection against sexually transmitted infection?
A. Coitus interrupt
B. Intra uterine device
C. Latex condom
D. Oral contraceptive

1158. Which of the following risk can be determined by Alpha fetoprotein analysis screening test ?
A. Neural tube defects
B. Placental insufficiency
C. Hydrous fatalism
D. Intra uterine growth retardation

1159. A 28years old patient in the medical ward was admitted to rule out her recurrent headache for the last year.
On the second day of her admission she complain of sever generalized headache . after identify the intensity
duration and severity the nurse informed physician and administered paracetamol and recorded the details in
the nursing notes after half hour when the nurse asked the patient replied that the pain was relieved and she
felt better what the nursing process step?
A. Nursing problem
B. Implementation
C. Assessment
D. Evaluation

1160. A physician has ordered gavage feeding every 4 hours for A 12-week-old infant with failure to thrive. in order
to know how far toinsert the feeding tube.The nurse should measure the distancefrom which of the following?
The tip of the infant's nose to the ear and then to the xiphoid process of the sternum

1161. Registered nurses must be able to measure blood pressure accurately. What could possibly result in
incorrect high blood pressure reading?
A. Bladder cuff is to wide
B. Cuff is deflated too quickly
C. Cuff is wrapped too loosely
D. Arm is above the level of the heart
ِ‫أوليسِيُجزىِ ُمحسنٌ ِاالِباحسان‬
1162. During meningitis outbreak in one of the hospital the non-immune staff members were given
immunoglobulin in order to prevent them from the infection. What is the type of immunity will be developed by
those staff?
A. Active
B. Long term
C. Natural
D. Passive

1163. A patient was on regular dose haloperidol. The nurse noticed that he start developing side effects in the
form stopped posture with shuffling gait and pill-rolling movement of his hands. Which of the following is most
likely the side effect?
A. Akathisia
B. Acute dystonia
C. Tardive dyskinesia
D. Pseudo parkinsonism
1164. A patient was brought to the emergency room because of a sudden onset of difficulty in breathing. The
result to a stat X ray revealed fluid accumulation in the pleural space. The doctor order for an immediate
thoracentesis. Which of the following sites should be prepared ?
A. Between 5th and 6th
B. Between 6th and 7th
C. Between 7th and 8th
D. Between 8th and 9th

1165. A 60 year old with gout visited the Medical Clinic and advice to increase fluid intake. Which of following is
the benefit of increase fluid?
A. Decrease inflammation
B. Increase calcium absorption
C. Promote the execration of uric acid
D. Provide a cushion for weakened bones

1166. Which of the following pain assessment scale is the most is the most appropriate for seven year-old boy who
twisted his ankle while playing football at school?
A. Numeric
B. Behavioral
C. Wrong- baker faces
D. Simple decretive

1167. During the working phase of the helping relationship, the patient was crying while disclosing his problems to
the nurse. Then in the middle of their conversion, the patient becomes silent. Which of the following should the
nurse do?
A. Ask question
B. Introduce another topic
C. Stay with client and be silent
D. Terminate session immediately

1168. Which of the following patient care plan is the most appropriate for a 37 years old post appendectomy
woman who is at risk of pneumonia ?
A. Restrict fluid intake
ِ‫أوليسِيُجزىِ ُمحسنٌ ِاالِباحسان‬
B. Teach how to use spirometer
C. Encourage ambulation as tolerated
D. Avoid coughing and deep breathing

1169. During the night shift routine rounds, a nurse found that a patient complain of sleep disturbance due to
frequent voiding several times at night. Which of the following is the best condition that describes the patient
complain?
A. Dysuria
B. Polyuria
C. Nocturia
D. Hematuria

1170. A 29-year-old woman is receiving an intravenous infusion of 5% dextrose in 0.45% sodium chloride. the
intravenous line had been started one day prior and now the patient complains of tenderness at the site. On
examination, the site is pink colored, swollen and tender to the touch. Which side effect of intravenous infusion
problem is most likely?
A. Infiltration
B. Phlebitis
C. Catheter sepsis
D. Air embolism

1171. A 25-year-old woman present to the Emergency Room with acute gastritis and signs of moderate
dehydration. Which is the most appropriate nursing diagnosis?
A. Infection
B. Fluid volume deficit
C. Activity intolerance
D. Fluid volume excess

1172. A conflict occurred for a patient with chronic renal failure with regards to the patient's needs, family needs,
the physician's advice, and the hospital polices and rules. What is the first commitment for the nurse who
provides care for the patient
A. Physician
B. Hospital
C. Family
D. Client

1173. A nurse accidentally dropped a medication ampule, informed the charge nurse and complicated an incident
report form. Which of the following best describes the nurse role?
A. Responsiveness
B. Timely decision making
C. Profession accountability
D. Abiding by moral obligation

1174. Patient receiving potassium sparing diuretics should be monitoring. Which of the following?
A. Hypokalemia
B. Hyperkalemia
C. Hypocalcemia
D. Hypercalcemia.
ِ‫أوليسِيُجزىِ ُمحسنٌ ِاالِباحسان‬
1175. Patient diagnosed with vitamin c deficiency. Which of the following pups of foods would the nurse most
likely instruct the patient to add to ?
A. Broccoli, tomatoes, strawberries, potatoes
B. Legumes, egg, oats, meat
C. Yeast, fish , bananas, nuts
D. Milk, liver, rice, beans

1176. Which of the following is the study of interactions are relationship between living organism and their
environment ?
A. Ecology.
B. Pathology
C. Microbiology
D. Parasitology
1177. The nurse who is planning to discharge education should instruct the caregivers that during hyper cyanotic
spell, which position is the most likely to benefits the child?
A. Supine
B. Prone
C. Side lying
D. Knee chest position

1178. A nurse was assigned to facilitate a group therapy for a group of patients with cluster a personality disorders
( odd eccentric group) which of the following should the first nursing intervention ?
A. Avoid self-mutilation
B. Provide adequate nutrition
C. Build a therapeutic relationship
D. Give health education about medication

1179. A 29 modifiable risk factors are those aspects of persons health risk over which she or he ahs no control .
which of the following is considers anon modifiable factors?
A. Genetic
B. Smoking
C. Eating
D. Activity

1180. The nurse is care for a patient who has not had bowel movement in the past few days and is suspected of
having a fecal impaction. Which of the following intervention confirm fecal impaction.
A. Hemoccult blood test.
B. Digital examination
C. Valsalva
D. Squat test

1181. A nurse is reviewing blood chemistry result of a client magnesium 1.7 0.7-1.2 With of the following
symptoms the nurse be monitored?
A. Trousseau sign
B. Depressed respirations
C. Elevated blood pressure
D. Increased tendon reflex

1182. What is the priority nursing assessment before administering methergine for management of postpartum
hemorrhage?
ِ‫أوليسِيُجزىِ ُمحسنٌ ِاالِباحسان‬
A. Blood pressure
B. Uterine atony
C. Amount of lochia
D. Deep tendon reflex

1183. A patient was complaining of depression, muscle weakness and continuous fatigue. which of the following
deficiencies is suspected?
A. Deficiency of vitamin B.
B. Deficiency of vitamin D.
C. Deficiency of vitamin E
D. Deficiency of vitamin K

1184. At the Diabetic Clinic, a doctor wanted to known glucose level patient as a part of diabetes management.
Which of the following tests reflects the average blood glucose period of two to three months?
A. Fasting blood sugar
B. Random blood sugar
C. Glucose tolerance test
D. Glycosylated hemoglobin.

1185. A newly hired was instructed that international guidelines followed in order to properly document the
nursing notes. What is expected from the nurse?
A. Use red ink
B. Write the notes in time sequence and sign each entry
C. Sign the documentation only at the end of the shift
D. Spelling is not important since notes are not in the native

1186. Which of the following statement refers to decision making?


A. Make a choice among alternatives
B. Provide feedback about deficiency in care
C. Increase interpersonal relation

1187. A nurse is providing educational workshop on infection control in the health center. Which of the following is
the body first line of defense against microorganisms ?
A. Cellular response
B. Vascular response
C. Antibody mediated defenses
D. Intact skin and mucous membranes

1188. What occurs during cardiogenic shock and result I adequate tissue perfusion?
A. Increase resistance of arterial
B. Decrease effectiveness of the heart as pump
C. Increased shunting of critical blood flow to heart
D. Decrease capacity of the venous beds

1189. A patient visiting the 10 days after a sinus surgery for a follow up complains of having a bad taste in the
mouth. The nurse smells of foul odor while examining the patient mouth. What does the suspect the patient
may have?
A. Pulmonary decompensation
B. Hemorrhage
C. Aspiration
D. Infection

1190. A 62 year old patient present to the emergency department and complains of muscle cramps nausea
vomiting and fatigue. The outside temp 45 and had been working outside day all day in the sun. he says that his
ِ‫أوليسِيُجزىِ ُمحسنٌ ِاالِباحسان‬
fingers are numb and cannot feel anything with them. BP 106/58 HR 99 RR 22 TEM 37.4 which of the
following electrolyte will be below the normal range?
A. Chromium
B. Magnesium
C. Potassium
D. Bicarbonate

1191. A 53 years old man is diagnosed as HIV positive. He had a history of and off diarrhea and progressive
weakness for past few months. He sigmoidoscopy. Which of the following measure should the healthcare
providers evaluate?
A. Reverse isolation and optimum nutrition
B. Confidentiality and social support system
C. Health maintenance and follow up regimen
D. Family screening and prevent spread of infection
1192. A 55-year-old male has a myocardial infarction requiring admission to the hospital. He is being treated with
an anticoagulant. The healthcare provider would be most concerned about which of the following :
A. Touble sleeping
B. Nausea
C. An elevated creatinine
D. Purpura

1193. A 52 year old woman is scheduled to undergo an abdominal perineal resection in the third day for removal
of a cancer of the rectum. The nurse review the care plan with the patient the patient will receive prophylactic
antibiotics and will be given a mechanical bowel preparation the day before. Which additional preparation
should the patient undertake in this time?
A. Wear pressure stocking
B. Perform leg strengthening exercise.
C. Maintain high-protein low residual diet
D. Take daily ferrous iron tablets

1194. This ECG is showed ?


A. Normal rhythm
B. Supraventricular tachycardia
C. Atrial fibrillation
D. Ventricular fibrillation

1195. A 40 year old man was admitted to a male medical department with pneumonia. A nurse in the department
explained and obtained informed consent for a chest CT scan procedure with contrast. with of the following
ethical principles underpinning informed consent?
A. Autonomy
B. Non maleficence
C. Beneficence
D. Respect

1196. Skeletal traction urinary catheter were discontinued for a patient who was immobilized in traction for sex
weeks. The patient developed a problem with urinary incontinence. Which of the following is the most
appropriate intervention?
A. Scheduled toileting
B. Bladder retraining
C. Promoted voiding
D. Behavioral training
ِ‫أوليسِيُجزىِ ُمحسنٌ ِاالِباحسان‬

1197. A patient presents to the clinic with 3+ edema of the lower extremities, diagnosed neck veins, tachycardia,
bounding pulse, weight gain of 4 kilograms with 7 days, shortness of breath, and wheezing. Fluid intake over the
past 24 hours has been 3700ml output is estimated at 2400ml. which of the following is the most likely nursing
diagnosis?
A. Fluid volume excess
B. Urinary tract infection
C. Hypothyroidism
D. Ketoacidosis

1198. A nurse inserted an intravenous cannula into a patient's right forearm and began to administer 500mg of
erythromycin through a pump delivery system. Following hospital protocol the nurse remains at the bedside for
the first 10minutes to assess the patient's response. After five minutes of the infusion, there is a small
hematoma at the cannula insertion site. The intravenous injection flow a shower than expected when the arm is
elevated. The skin proximal to the catheter site is warm, reddened and painful on palpation. Which of the
following is the most appropriate nursing action?
A. Apply ice pack for twenty minutes at site
B. Discontinue and re-establish a new site
C. Increase infusion of the pump
D. Position right below level of heart

1199. A patient is being prepared for discharge following hip replacement surgery. The nurse is providing him with
discharge education. Which of the following information should be taught to him as an effective pain
management principle?
A. Avoid giving pain medication prior to participating in physical therapy
B. Give a double dose of pain medication if pain is intolerable
C. Give pain medication before pain becomes severe
D. Delay giving pain medication as long as possible

1200. A 57 year old man is admitted to the cardiac unit with palpations, headache, and chest tightness, on
auscultation S3 gallop and a diagnostic murmur can be heard, a doctor orders 2.5mg of verapamil slow IV push.
However, the ventricular rate does not slow down BP 95/62 HR 170 RR 25 TEMP. 36.9 what next action
should the nurse expect?
A. Vagal maneuver
B. Sedation and intubation
C. Another dose of verapamil
D. Synchronized cardio version

1201. This ECG is showed ?


A. Normal rhythm
B. Supraventricular tachycardia
C. Atrial fibrillation
D. Ventricular fibrillation

1202. A nurse is working in a medical unit. She is 65yearold women with cerebrovascular accident home care
visits. The nurse teaches the patients wheelchair safely. Which of the following statements by the
teaching was effective?
A. "The breaks should be used when storing
B. "I will push the wheelchair in from of the
ِ‫أوليسِيُجزىِ ُمحسنٌ ِاالِباحسان‬
C. "I will remove the feet rests from the
D. "When entering the elevator I will pull the that the big wheels en first

1203. A urinalysis is best evaluated for accurate results if the specimen is analyzed within which of the following?
A. 1 hour of collection or refrigerated until analyzed
B. 1 hour of collection or left at room temperature
C. 2 hours of collection
D. 4 hours of collection

1204. Wassermann reaction test For Syphilis

1205. A nurse is assigned to care for a patient with physical immobility due to right knee injury. the nurse is
preparing to write nursing diagnosis. Which of the following is a priority for nursing diagnosis?
A. Pain
B. Hygiene
C. Dehydration
D. Skin integrity

1206. What will happen to the patient if whole blood transfused exceeds four hours in room temperature?
A. Hypokalemia
B. Hyperkalemia
C. Hyponatremia
D. Hypernatremia

1207. A 42-year-old patient is admitted in the Female Medical Ward for sickle cell anemia. She is irritable and
shouting on her helper and the nursing staff. The nurse has politely discussed her concerns and finally asked why
she has been shouting on others. The patient stated that because her husband did not visit her even once during
her hospitalization. Which of the following nursing interventions must have priority?
A. reassure to solve her problem
B. discuss the situation with her husband
C. help patient come up with the solution
D. use probing to gather more information

1208. Toddler is admitted to the hospital with acute laryngitis (group) 50 Respiration 38.7 C Temperature Which of
the following in the is the priority nursing intervention when child carrying for?
A. Decrease stimulation to promotes rest
B. Monitor the toddler's temperature
C. Frequently assess respiratory status
D. Clean the nares with a bulb syringe

1209. A 19 year girl was scheduled for the extraction of her wisdorn teeth under general anesthesia. Her
preoperative anesthesia consultation was done and the consent was signed by her, but she refused to remove
her nail polish and trim her long nails, which was a necessary requirement. Why is it necessary to have
unpolished and trimmed nails as one of the requirements before the general anesthesia?
A. Avoid puncturing the inner side oxygen probe.
B. Prevent chipping or scratches from long
C. Assess for nail color during anesthesia
D. Follow the pre anesthesia protocol

1210. A 52 years old man was working in tall grass when a snake bit him. An ambulance arrived at the scene 20
minutes later. They found the man lying on the ground with cold and clammy skin. He was having difficulty
breathing and the right ankle was swollen. He complained of double vision, feeling weak and itching skin. He
reported that this was his second snakebite. The paramedics prepare to administer intramuscular epinephrine
ِ
‫ِاالِباحسان‬ ٌ‫أوليسِيُجزىِ ُمحسن‬
and place a tourniquet . Blood pressure 86/48 mmHg Pulse rate 130/min Respiratory
rate 28/min Which of the following nursing diagnosis has highest priority ?
A. disturbed sensory perception related to effects of pruritus
B. ineffective breathing pattern related to bronchospasm
C. decrease cardiac output related to systemic vasoconstriction
D. risk for injury related to drug treatment and adverse effects

1211. The nurse is placing a nasogastric tube in a patient for enteral nutrition. Which of the following actions must
the nurse consider prior to give feeding to verify placement?
A. Palpating the abdomen
B. Aspirating and testing the contents
C. Placing and tube in saline to see if bubbles appear
D. Injecting air into the tube and waiting for the patient to burp

1212. A 30 year-old woman suffered an automobile accident and was brought to the hospital by emergency services.
She has no known medical history but exhibits signs of .neurological damage. An assessment shows a Glasgow
Coma Score of 6 Which of the following would be expected ?
A. Decerebrate posturing
B. Withdrawal to pain stimuli
C. Confused verbal response
D. Spontaneous eye openin

1213. A nurse called the doctor while he is at home when she asked him for an order for patient's medication As a
nurse how many hours must the doctor took to write the order in the hospital documentation?
A. 6 hours
B. 24 hour
C. 48 hours
D. 12 hours

1214. A handbook on nursing research emphasises on comprehensive reading and summarising of previous
publications related to the topic Which of the following is the component of this research
A. Literature review
B. Systematic review
C. Discussion of results
D. Referencing

1215. A 25 year old female patient is admitted to the surgical ward for the repair of her umbilical hernia. Her initial
assessment revealed that she belonged to the lower socioeconomic group and she had four sisters and one
brother, all of them were uneducated. She was living in a small house of two bed rooms and one bathroom. The
main source of her family income was through farming and the only bread earner in the family was her father.
What type of information can this be categorized as?
A. Patient's family status
B. Personal information
C. Economic background
D. Demographic information
ِ‫أوليسِيُجزىِ ُمحسنٌ ِاالِباحسان‬
1216. A 63 year old man patient is admitted to the emergency department with complaints of severe chest pain.
The apical pulse is the most appropriate site to obtain the accurate heart beats. Where
should the nurse place the stethoscope to obtain the apical pulse?
A. In the middle of the chest between the nipples
B. Over the apex of the heart between 4th and 5th rib
C. Between 5th and 6th rib slightly above the nipples
D. Between 5th and 6th rib the mid clavicle line below the nipple

1217. A 58 year old woman present to clinic with tiredness and weakness. She is tired throughout the day and
usually sleeps for in the mid afternoon. The last bowel movement was four days has a prescription for
levothyroxine but says she doesn’t remember to take it as prescribed. The doctor arrange for specimen to be
sent to the Lab. Which laboratory test result are most likely?
A. Increase T4 and decrease TSH
B. Increase T4 and TSH unchanged
C. Decrease T4 and TSH increase
D. Unchanged T4 and TSH decrease

1218. A 20 year old woman is hospitalized with a strong and uncontrollable cough and has difficulty breathing
while coughing. A doctor writes an order for the patient to be transported from the medical surgical
department to the radiology department for an xray examination. The nurse prepares to transfer the patient
and considers standard precautions and additional transmission precautions. Who of the following would be
required to wear a mask?
A. Radiology staff
B. Nurse and patient
C. Patient
D. Nurse

1219. A nurse is caring for an elderly bed ridden patient who is dependent on staff most activities of daily living.
There is an order for standard precautions in the patient's chart. The patient's care plan includes feeding,
washing hair with dry shampoo, brushing teeth, and nail care. Which of the activities would require precaution?
A. Brushing teeth
B. Feeding
C. Hair washing
D. Nail care

1220. Staff reports to a charge nurse that they suspect temperature are being incorrectly measured by a certain
machine. Which of the following actions is the best for the charge nurse to take?
A. As those taking vital signs to compare result two different machines
B. Tell staff to obtain a machine another unit and use to take temperature
C. Spend time at shift change to personally assess the accuracy of machines
D. Call maintenance department and as them to pick up machine for repair

1221. Following a lumbar puncture, a patient has several complaints. Which of the following complaints indicate
that patient is experiencing a complaints?
A. I have a headache that gets worse when I sit up
B. Iam having pain in my lower back when I move my legs
C. My throat hurts when I swallow
D. I feel sick to my stomach and I'm going throw up
ِ‫أوليسِيُجزىِ ُمحسنٌ ِاالِباحسان‬

1222. A nurse calls a physician regarding a change in a patient's condition. A physician gives orders over the
telephone for stat arterial blood gases. Which of the following is the best intervention to be implemented?
A. Give order stat to health unit coordinator to input in computer
B. Write down order and read it back to physician
C. Write down order for ABGs immediately
D. Call respiratory therapist stat to draw the ABGs

1223. Which of the following is the most appropriate physical assessment of a patient with blunt abdominal
trauma?
A. Inspect, palpate, auscultate, percuss
B. Inspect, palpate, percuss, auscultate
C. Inspect, percuss, palpate, auscultate
D. Inspect, auscultate, palpate, percuss

1224. A nurse is caring for patient who is being admitted with urinary tract infection. The patient feels cold and
shivering Blood pressure 110/70 mmHg Heart rate
22 /min Temperature 39.7
Which of the following is the best nursing action?
A. Provide a hot drink
B. Cover the patient with light blanket
C. Start the air conditioning system
D. Turn off lights and close curtains

1225. Site of iilestomy :


A. Left lower
B. Right lower
C. Left upper
D. Right upper

1226. A 60year old woman was admitted with chronic pulmonary disease associated with carbon dioxide retention
the patient is on nasal prong 3lpm which of the following is the potential risk this patient if excessive oxygen is
administered?
A. Apnea
B. Hypoxemia .
C. Hypercapnia
D. Pulmonary edema

1227. A 45 years old woman patient in the outpatient department has come with severe throbbing headache at
the crown site since last night. She is a known hypertensive for the past five years but takes her antihypertensive
medicines only when she has headaches and not on daily basis. She took her medicine twice since last night. Her
vital signs show mild elevation in her blood pressure and she is admitted to rule out the cause of her headache
after initial investigations. Which of the following nursing intervention is required at first ?
A. document information that is gathered
B. arrange for investigations as ordered
C. prepare for physical examination
D. promote rest and relaxation

1228. Which of the following is the main role of the charge nurse?
A. Patient are reassessed at appropriate intervals
B. Provide enough equipment and supplies
C. Assess patient regularly for pain
D. Coordinate patient care.
ِ‫أوليسِيُجزىِ ُمحسنٌ ِاالِباحسان‬

1229. Charge nurse hears a nurse speaking to patients with disrespect. which of the following is the best action for
the charge nurse?
A. Apologize to patient
B. Tell nurse to be kind
C. Take over the care
D. Speak privately with nurse

1230. Which of the following patients is most appropriate to care for first?
A. Patient with a central venous catheter inserted 4 hours ago
B. Paralyzed patient who has had continued tube feeding
C. Adolescent patient who had appendectomy 1 day ago
D. Gastrectomy patient who came from postoperative care unit 1 hour ago

1231. Which of the following nursing intervention assists in the prevention of pressure ulcers following abdominal
surgeries?
A. Frequent surgery site every two hours
B. Frequent moisturizing of skin
C. Use side rails as restrains
D. Encourage ambulation

1232. The nurse is preparing the patient for hospital for discharge following above-the-knee amputation with rigid
dressing over the residual limb. Fourteen days following surgery, the patient is successfully fitted with Prosthetic
limb and begins physical therapy Which of the following resting position is most beneficial? o
A. Sitting with legs crossed
B. Abduction of residual limb
C. Knee flexion when sitting in chair
D. Knee extension when in bed

1233. A client has just returned to a nursing unit after an above-knee amputation of the right leg. The nurse should
place the client i which position?
A. Prone
B. Reverse Trendelenburg's
C. Supine, with the amputated limb flat on the bed
D. Supine, with the amputated limp supported with pillows

1234. A patient was admitted to burns unit 12 hours ago with deep partial-thickness burns. Which of the following
would be expected to be seen?
A. Increased body temperature
B. Increase hematocrit levels
C. Increase blood pressure
D. Increase urine output

1235. Most common cause for acute renal failure ?


A. pyelonephritis
B. Tubular destruction
C. Urenary tract obstruction
D. Dehydration
ِ‫أوليسِيُجزىِ ُمحسنٌ ِاالِباحسان‬
1236. Health education about accident in school, role of who?
A. Nurse
B. Doctor
C. Social worker
D. Administrator

1237. patient changed his mind after informed research onset. What principle is linked to this patient right?
A. Justice
B. Competence
C. Autonomy
D. Beneficence

1238. A patient treated an insect bite with traditional alternative therapy of row garlic juice caused strong skin
reaction. The patient agreed not use it again after the nurse explained its harmful effects Which of the following
is the best expression of the nurse's role?
A. patient's advocate
B. Resource person
C. Care planning
D. Caregiver

1239. A 40 year-old client is admitted to the hospital with a 'SCI with no function below the level of injury and with
Foley's catheter in place. He is complaining of a severe throbbing headache that suddenly started one day ago.
Assessment of the patient reveals she has diaphoresis, and flushing of the face and neck. Blood Pressure 168/94
mmHg Heart Rate 48 /min Respiratory Rate 30 /min Temperature 39° C
What action should the nurse take first?
A. Adjust the temperature in the patient's room
B. Check the foley tubing for kinks or obstruction
C. Notify the physician about the change in status
D. Administer the ordered acetaminophen (Tylenol) 1Spinal cord injury

1240. After the pericardiocentesis, the doctor inserted a polyethylene catheter left it in the pericardial sac. The
patient asked the nurse about the response of the catheter. What should be the nurse's response?
A. Monitor consistency of drainage
B. Prevent movement of pericardial sac
C. Prevent recurrence of cardiac tamponed
D. Prevent increase in venous and blood pressure

1241. A 53-year-old was referred to the Emergency Department by the primary healthcare center for hyperglycemia.
The nurse interviews him and finds out that he has been recently diagnosed with diabetes. He says that he does
not understand why he needs insulin (see lab result) Test Glucose, fasting Result 12.2 Normal Values 3.5-6.5
mmol/L
Which of the following learning needs is indicated for this patient?
A. Insulin Alternatives
B. Complications of diabetes
C. Disease process of diabetes
D. Lifestyle and dietary changes

1242. A patient is scheduled for surgical placement of a fecal ostomy. The nurse instructs the patient what the usual
consistency of the feces at this is a fluid. Based on the illustration. Which of the following areas is the most likely
surgical placement of this fecal ostomy ?
A. Two ( right lower)
B. Three
C. Four
D. Five
ِ‫أوليسِيُجزىِ ُمحسنٌ ِاالِباحسان‬
1243. Which patient is a contraindicated for enema
A. Gluocoma
B. hypertensive
C. renal failure
D. liver disease

1244. A 40-year-old woman presents to the clinic with cramping abdominal pain that ) comes and goes every 5 to
10 minutes. She is unable to pass gas or stool. The abdomen appears greatly distended. On auscultation bowel
sounds are quiet and hypoactive. On percussion, there is resonance. On palpation, the abdomen examination is
performed and the rectum is found to be empty. is tender, without "guarding". A rectal Which of the following
health problems would be most likely?
A. Diverticulitis
B. Obstruction
C. Appendicitis
D. Crohn's disease

1245. A 69 year-old obese woman with past medical history of peptic ulcer disease underwent an urgent
Laparoscopic loop ileostomy with intra-operative colonic lavage without complications. On discharge plan, the
nurse explained several instructions. Which of the following symptom the nurse should instruct the client to report
immediately?
A. Temperature of 37.6° C
B. Passage of liquid stool in the stoma
C. Occasional presence of undigested food
D. Absence of drainage from the ileostomy for 6 or more hours

1246. A 70 year-old woman is admitted to the Cardiac Care Unit with new onset atrial fibrillation and is receiving
intravenous diltiazem and heparin. What is the most likely nursing diagnosis?
A. High risk for infection
B. High risk for impaired gas exchange
C. High risk for decreased cardiac output
D. High risk for disturbed sensory perception

1247. The primary nurse in the Cardiac Care Unit provided care for a patient with myocardial infraction with
chest pain. Which activity is related to nursing planning?
A. Administer morphine as prescribed
B. Assess the level of pain management
C. Clearly state the nursing diagnosis as pain
D. Determine a set of nursing intervention outcomes for pain.
1248. A 21-year-old woman patient was admitted in the Psychiatric Ward for her panic anxiety attacks. She was
mostly quiet and unsocial, but her assigned nurse gradually developed a trusted relationship with her. In one of
the interactions with the nurse, she asked about how she could improve her behaviour with her mother as
regardless of anyone's fault, she yells and even hits her mother. What should be the main emphasis for health
education of this patient?
A. Social interaction
B. Family's counselling
C. Role and relationship
D. Coping-stress tolerance

1249. A nurse is discharging a patient after hospitalization due to myocarditis. Which of the following statements
should be included in discharge teaching?
A. There is usually some residual heart enlargement
B. May resume previous activities as before hospitalization
C. Avoid immunizations against infectious disease
ِ‫أوليسِيُجزىِ ُمحسنٌ ِاالِباحسان‬
D. Rapidly beating heart is a common side effect of the illness and is not dangerous

1250. While undergoing a soapsuds enema, the client complains of abdominal cramping. The nurse should:
A. Immediately stop the infusion
B. Lower the height of the enema container
C. Advance the enema tubing 2 to 3 inches
D. Clamp the tubing

1251. A 40 year-old woman present with painful leg cramps and muscle twitching, which wakens her at night. Her
menstrual cycles are irregular and cause painful abdominal cramping. The hair, skin and nails appear dry and
brittle (see lab results) Test Result Normal Values Calcium 2.09 2. 15-2.62mmollL Phosphate, inorganic 1.69 0.82-
1.51mmollL Magnesium 0.55 0.7-1.0mmollL What type of lifelong diet would be most beneficial for this patient?
A. Increased magnesium and vitamin C
B. Increased calcium and vitamin D
C. Increased dairy and iron
D. High protein and high calorie

1252. A patient comes to the Emergency Department after falling out The nurse notices that the patient's left leg is
externally rotate shorter than the right leg. The patient cannot move the left complains of severe pain. What is
the most likely cause of these symptoms?
A. A Dislocated patella
B. Fractured left hip
C. Fractured left tibia
D. Dislocated left fibula

1253. CAUSES of spontaneous abortion……… congenital anomalies

1254. Which of the following conditions would be most appropriately cared for by a telemetry unit nurse?
A. New onset of atrial fibrillation

1255. While a nurse was having a conversation with a patient about his daily Iife routine, she noticed that he moved
closer to her body and made her feel tense. Which of the following is the most appropriate response?
A. Yell and send him to his room
B. Change the topic of conversation
C. Keep silent and step back
D. Appear uncomfortable and limit behaviour

1256. A head nurse of a coronary care unit delegated staff scheduling to a senior nurse in that unit. Which of the
following steps must the head nurse implement before delegating tasks?
A. Negotiate with the senior
B. Take signature of senior
C. Explain task to senior
D. Make sure hospital policies for delegating tasks

1257. Which of the following describes functional nursing?


A. Task and activity oriented care.
B. Involves a group of nurses working together to identify, plan, implement, and evaluate
patient centered care
C. Ratio of patient care
D. Emphasizes on continuity of care responsibility
ِ‫أوليسِيُجزىِ ُمحسنٌ ِاالِباحسان‬
1258. Which of the following patients is most appropriate to care for first?
A. Patient with a central venous catheter inserted 4 hours ago
B. Paralyzed patient who has had continued tube feeding
C. Adolescent patient who had appendectomy 1 day ago
D. Gastrectomy patient who came from postoperative care unit 1 hour ago

1259. A patient is to receive a prescribed blood transfusion using a subclavian catheter. Catheter was insert=ed
about 30 minutes ago. Which of the following is the most appropriate step before starting blood transfusion?
A. Check patency of catheter by flushing with normal saline solution
B. Position patient with the head of the bed elevated 30 degrees
C. Review result of chest radiograph completed 15 minutes ago
D. Obtain patient's most recent complete blood count result.

1260. A patient diagnosed with hypothyroidism. Which of the following periods will the patient need to take
thyroid hormone replacement therapy before reassessment by his physician?
A. About two months
B. One year
C. His entire life
D. Unit his laboratory result normal

1261. A 40 year old man was admitted to a male medical department with pneumonia. A nurse in the department
explained and obtained informed consent for a chest CT scan procedure with contrast. with of the following
ethical principles underpinning informed consent?
A. Autonomy
B. Non maleficence
C. Beneficence
D. Respect

1262. Skeletal traction urinary catheter were discontinued for a patient who was immobilized in traction for sex
weeks. The patient developed a problem with urinary incontinence. Which of the following is the most
appropriate intervention?
A. Scheduled toileting
B. Bladder retraining
C. Promoted voiding
D. Behavioral training

1263. A patient presents to the clinic with 3+ edema of the lower extremities, diagnosed neck veins, tachycardia,
bounding pulse, weight gain of 4 kilograms with 7 days, shortness of breath, and wheezing. Fluid intake over
the past 24 hours has been 3700ml output is estimated at 2400ml. which of the following is the most likely
nursing diagnosis?
A. Fluid volume excess
B. Urinary tract infection
C. Hypothyroidism
D. Ketoacidosis

1264. A 45 year old patient has a colostomy bag attached as a result of benign mass removal, while conducting the
discharge teaching session, the nurse assessed the patient's wife understanding of the teaching. Which of the
following steps of nursing process is exercised here?
ِ‫أوليسِيُجزىِ ُمحسنٌ ِاالِباحسان‬
A. Implementation
B. Planning
C. Assessment
D. Evaluation

1265. A nurse inserted an intravenous cannula into a patient's right forearm and began to administer 500mg of
erythromycin through a pump delivery system. Following hospital protocol the nurse remains at the bedside for
the first 10minutes to assess the patient's response. After five minutes of the infusion, there is a small
hematoma at the cannula insertion site. The intravenous injection flow a shower than expected when the arm
is elevated. The skin proximal to the catheter site is warm, reddened and painful on palpation. Which of the
following is the most appropriate nursing action?
A. Apply ice pack for twenty minutes at site
B. Discontinue and re-establish a new site
C. Increase infusion of the pump
D. Position right below level of heart

1266. A patient is being prepared for discharge following hip replacement surgery. The nurse is providing him with
discharge education. Which of the following information should be taught to him as an effective pain
management principle?
A. Avoid giving pain medication prior to participating in physical therapy
B. Give a double dose of pain medication if pain is intolerable
C. Give pain medication before pain becomes severe
D. Delay giving pain medication as long as possible

1267. The nurse is caring for a client who had sustained a head injury vehicle accident. In the follow-up visit the
client appears disoriented and is unable too recall recent events. When asking details the client responds by
talking about the weather. Which of the following is most likely additional clinical findings?
A. Dryness of the face
B. Increase in heart rate
C. Constriction of pupils
D. Decrease in blood pressure

1268. What the important considerations that the nurse must checked after the placenta is delivered?
A. Checked if the cord is long enough for baby
B. Check if the umbilical cord has 3 blood vessels
C. Check if the cord has a meaty portion and a shiny portion.
D. Check if the placenta is complete including the membranes

1269. A 35 year old patient was admitted to a medical ward with confirmed agnosies f meningococcal infection.
Which of the following infection control preventions the nurse should implement?
A. Droplet precaution
B. Contact precaution
C. Airborne precaution
D. Standard precaution

1270. The nurse is placing a nasogastric tube in a patient for enteral nutrition. Which of the following actions must
the nurse consider prior to give feeding to verify placement?
A. Palpating the abdomen
ِ‫أوليسِيُجزىِ ُمحسنٌ ِاالِباحسان‬
B. Aspirating and testing the contents
C. Placing and tube in saline to see if bubbles appear
D. Injecting air into the tube and waiting for the patient to burp

1271. A patient with an allergy to cats arrives at the emergency department with symptoms of pruritus, and
difficulty in swallowing after visiting some who has three cats. Addition to confirming airway patency, which of
the following is it most appropriate to prepare the patient for?
A. Intravenous line insertion
B. Intravenous glucocorticoid
C. Subcutaneous epinephrine
D. Application of ice throat

1272. A 25 year old patient is admitted for observation in the surgical unit with ad concussion after being involved
in a road traffic accident. A nurse performs a routine neurological assessment and notes a decrease in the
patient’s level of consciousness. BP 17/110 HR 52 RR 11 TEM 36.1 SAT 93%
Which of the following is the most likely cause?
A. Hypovolemia
B. Hypothermia
C. Pulmonary embolism
D. Increased intracranial pressure

1273. An 80 year-old man has Alzheimer's disease and increasingly has frequent periods of dementia during
which he doesn't remember family members names, and is not oriented to person, place or time. The health
care provider speaks with the man's son about becoming the medical power of attorney for his father who is no
longer able to do this for himself. Which two elements of informed consent does the father lack?
A. Autonomy and fidelity
B. Confidentiality and justice
C. Competence and comprehension
D. Voluntariness and veracity

1274. The home care nurse is providing wound care for a patient. The nurse evaluates the wound and notes the
presence of granulation tissue in the wound bed. This observation represents which phase of wound healing?
A. Maturation
B. Inflammation
C. Proliferation
D. Finalization

1275. A 45 year-old immigrant from Southeast Asia is administered a Mantoux test. He has no significant health
history and shows no symptoms of active cough, night sweats, fatigue or weight loss. 72 hours after the
subcutaneous injection there is a wheal of 6 mm diameter at the site. A sputum sample and blood sample are
collected for analysis. X-ray: Shows clear lung fields with an absence of infiltrates bilaterally. Which of the
following should be suspected?
A. Active tuberculosis
B. BCG1 vaccination
C. HIV infection
D. Reactivation
ِ‫أوليسِيُجزىِ ُمحسنٌ ِاالِباحسان‬
1276. A 30 year-old married female has dilatation and curettage as a therapeutic abortion. A nurse was preparing for
discharge instructions. What should the nurse include in the discharge instructions?
A. Take high protein diet
B. Use tampons during swimming
C. Avoid sexual intercourse for two weeks
D. Continue on bed rest for two weeks at home

1277. A community health nurse was evaluating DOT program for a pulmonary tuberculosis patient. What is the aim
of DOT program?
A. Assess family support
B. Evaluate nutritional status
C. Monitor home environment
D. Prevent antibiotic resistance "

1278. Raynaud disease

1279. Weber test :how is the Weber test done…… By placing a 512 vibrating tuning fork on the patient forehead
(midline), where you can hear a buzzing noise.

1280. The nurse is performing a physical examination to client with hearing difficulties . the nurse activated the
tuning fork and placed it on top of the client head what test did the nurse perform?
A. Whisper test
B. Rinne test
C. Audiometer test
D. Weber test

1281. The nurse is performing a physical examination to client with hearing difficulties . the nurse activated the
tuning fork and placed it on top of the client head what test did the nurse perform? A. Whisper test B. Rinne
test C. Audiometer test D. Weber test
site of weber test is ?
A. Occipital bone
B. Bilateral bone
C. Mastoid bone
D. Middle of forehead

1282. A 62 year old man present to the clinic with complains of hearing loss in the left ear the hearing loss has
been progressive over the past year. A semi-transparent, a pale grey color and clear of cerumen. A weber test is
performed and the sound lateralized to the right ear. Which type of hearing loss is suspected?
A. Left-sided conductive
B. Right-sided conductive
C. Left-sided sensorineural
D. Right-sided sensorineural
ِ‫أوليسِيُجزىِ ُمحسنٌ ِاالِباحسان‬

1283. At 6:00 PM while admitting a woman for a scheduled repeat C section1, a patient tells a nurse that she drank
a cup of coffee at 4:00OM because she wanted to avoid getting a headache. Which of the following actions
should the take first?
A. Inform anesthesia care provider
B. Ensure preoperative lab results are available
C. Start prescribed IV with lactated ringers
D. Contact patient's obstetrician

1284. Staff reports to a charge nurse that they suspect temperature are being incorrectly measured by a certain
machine. Which of the following actions is the best for the charge nurse to take?
A. As those taking vital signs to compare result two different machines
B. Tell staff to obtain a machine another unit and use to take temperature
C. Spend time at shift change to personally assess the accuracy of machines
D. Call maintenance department and as them to pick up machine for repair

1285. Following a lumbar puncture, a patient has several complaints. Which of the following complaints indicate
that patient is experiencing a complaints?
A. I have a headache that gets worse when I sit up
B. Iam having pain in my lower back when I move my legs
C. My throat hurts when I swallow
D. I feel sick to my stomach and I'm going throw up

1286. A nurse calls a physician regarding a change in a patient's condition. A physician gives orders over the
telephone for stat arterial blood gases. Which of the following is the best intervention to be implemented?
A. Give order stat to health unit coordinator to input in computer
B. Write down order and read it back to physician
C. Write down order for ABGs immediately
D. Call respiratory therapist stat to draw the ABGs

1287. Which of the following is the most appropriate physical assessment of a patient with blunt abdominal
trauma?
A. Inspect, palpate, auscultate, percuss
B. Inspect, palpate, percuss, auscultate
C. Inspect, percuss, palpate, auscultate
D. Inspect, auscultate, palpate, percuss

1288. A nurse caring for a patient with liver cirrhosis observes the following laboratory result
NA 130 134-146 K 3.0 3.5-5.1 chloride 98 97-108 Ammonia 32.1 9-33
Which of the following is the most likely plan based on ammonia level?
A. Encourage high protein intake
B. Restrict high vitamin C foods
C. Restrict high protein intake
D. Encourage high vitamin C food

1289. A patient who is being scheduled for open reduction internal fixation is interested in knowing the details of
the surgery and insisting on the nurse to explain to explain the procedure. Which of the following nursing
intervention is the most appropriate?
ِ‫أوليسِيُجزىِ ُمحسنٌ ِاالِباحسان‬
A. Assess anxiety level
B. Assure he will be better after surgery
C. Explain surgery
D. Give patient some sedative medication

1290. A 45 year old man is admitted to the neurosurgery ward for the surgical elevation of depressed skull
fracture. He has episodic severe headache with seizure and is unable to concentrate. Which of the following
initial nursing problem needs more attention?
A. Disturbed coping and anger spells
B. Risk of injury to seizur
C. Disturbed communication and irritability
D. Pain management and comfort measure

1291. At 6:00 PM while admitting a woman for a scheduled repeat C section1, a patient tells a nurse that she drank
a cup of coffee at 4:00OM because she wanted to avoid getting a headache. Which of the following actions
should the take first?
A. Inform anesthesia care provider
B. Ensure preoperative lab results are available
C. Start prescribed IV with lactated ringers
D. Contact patient's obstetrician

1292. A mother of a patient who is on antipsychotic drug asked a doctor about some more detail about the drug.
The doctor told the mother that her son is taking an atypical antipsychotic drug. Which of the following is an
examples of this type of drug?
A. Thioridazine
B. Clozapine
C. Chlorpromazine
D. Haloperidol

1293. A cardiac care unit nurse decides to gather relevant data about a myocardial infraction patient who has
breathing difficulty and sever bradycardia. Which of the following would be the most relevant and suitable
database?
A. Focused
B. Emergency
C. Complete
D. Follow-up

1294. A nurse measures blood pressure of a patient admitted to the Male Medical Ward for 3 days. Which of the
following is the best method to ensure that the nurse's reading is reliable and accurate?
A. Check patient's blood pressure every shift
B. Measure patient's temperature
C. Document result in chart
D. Measure patient's heart rate

1295. On orientation day, a nurse educator said to a new nurse" our hospital provides the highest level of
specialized healthcare in an integrated educational and research setting" which of the following aspects of the
hospital's strategic plan is indicated by this statement?
A. Vision
ِ‫أوليسِيُجزىِ ُمحسنٌ ِاالِباحسان‬
B. Policy
C. Values
D. Mission

1296. A head nurse of an intensive care unit wrote a report about the needs of her department and gave it to the
nursing supervisor. Which of the following types of communication is this?
A. Upward
B. Horizontal
C. Diagonal
D. Downward

1297. A head nurse of a coronary care unit delegated staff scheduling to a senior nurse in that unit. Which of the
following steps must the head nurse implement before delegating tasks?
A. Negotiate with the senior
B. Take signature of senior
C. Explain task to senior
D. Make sure hospital policies for delegating tasks

1298. Which of the following describes functional nursing?


A. Task and activity oriented care.
B. Involves a group of nurses working together to identify, plan, implement, and evaluate
patient centered care
C. Ratio of patient care
D. Emphasizes on continuity of care responsibility

1299. Which of the following patients is most appropriate to care for first?
A. Patient with a central venous catheter inserted 4 hours ago
B. Paralyzed patient who has had continued tube feeding
C. Adolescent patient who had appendectomy 1 day ago
D. Gastrectomy patient who came from postoperative care unit 1 hour ago

1300. A patient is to receive a prescribed blood transfusion using a subclavian catheter. Catheter was insert=ed
about 30 minutes ago. Which of the following is the most appropriate step before starting blood transfusion?
A. Check patency of catheter by flushing with normal saline solution
B. Position patient with the head of the bed elevated 30 degrees
C. Review result of chest radiograph completed 15 minutes ago
D. Obtain patient's most recent complete blood count result.

1301. A patient diagnosed with hypothyroidism. Which of the following periods will the patient need to take
thyroid hormone replacement therapy before reassessment by his physician?
A. About two months
B. One year
C. His entire life
D. Unit his laboratory result normal

1302. A 40 year old man was admitted to a male medical department with pneumonia. A nurse in the department
explained and obtained informed consent for a chest CT scan procedure with contrast. with of the following
ethical principles underpinning informed consent?
ِ‫أوليسِيُجزىِ ُمحسنٌ ِاالِباحسان‬
A. Autonomy
B. Non maleficence
C. Beneficence
D. Respect

1303. Skeletal traction urinary catheter were discontinued for a patient who was immobilized in traction for sex
weeks. The patient developed a problem with urinary incontinence. Which of the following is the most
appropriate intervention?
A. Scheduled toileting
B. Bladder retraining
C. Promoted voiding
D. Behavioral training

1304. A patient presents to the clinic with 3+ edema of the lower extremities, diagnosed neck veins, tachycardia,
bounding pulse, weight gain of 4 kilograms with 7 days, shortness of breath, and wheezing. Fluid intake over the
past 24 hours has been 3700ml output is estimated at 2400ml. which of the following is the most likely nursing
diagnosis?
A. Fluid volume excess
B. Urinary tract infection
C. Hypothyroidism
D. Ketoacidosis

1305. A 45 year old patient has a colostomy bag attached as a result of benign mass removal, while conducting the
discharge teaching session, the nurse assessed the patient's wife understanding of the teaching. Which of the
following steps of nursing process is exercised here?
A. Implementation
B. Planning
C. Assessment
D. Evaluation

1306. A nurse inserted an intravenous cannula into a patient's right forearm and began to administer 500mg of
erythromycin through a pump delivery system. Following hospital protocol the nurse remains at the bedside for
the first 10minutes to assess the patient's response. After five minutes of the infusion, there is a small
hematoma at the cannula insertion site. The intravenous injection flow a shower than expected when the arm is
elevated. The skin proximal to the catheter site is warm, reddened and painful on palpation. Which of the
following is the most appropriate nursing action?
A. Apply ice pack for twenty minutes at site
B. Discontinue and re-establish a new site
C. Increase infusion of the pump
D. Position right below level of heart

1307. A patient is being prepared for discharge following hip replacement surgery. The nurse is providing him with
discharge education. Which of the following information should be taught to him as an effective pain
management principle?
A. Avoid giving pain medication prior to participating in physical therapy
B. Give a double dose of pain medication if pain is intolerable
C. Give pain medication before pain becomes severe
D. Delay giving pain medication as long as possible
ِ‫أوليسِيُجزىِ ُمحسنٌ ِاالِباحسان‬
1308. most common cause of death in Saudi kingdom is cardiac causes and traffic accident

1309. sinusitis avoid travel with Airplanes

1310. Saudi Arabia neonatal screening for hypothyroidism

1311. Cervical dilation in the active phase of the first stage 4 -7 cm

1312. Transformational Leader ship definition leader that causes change I individual and social system enhance
motivation morale and performance of followers

1313. Rheumatic Fever medication is Penicillin

1314. Influenza in HONG kONG and killed millions of people consider as Epidemic disease

1315. Tuberculosis is caused by Mycobacterium

1316. Which of the following is not considered as an obstetrical emergency ?


A. Ectopic pregnancy
B. Placenta previa
C. Placenta anterior
D. Eclampsia

1317. Doctor order 0.125mg available is 0.25mg capsule how many capsule the nurse will give to the patient
Half capsule

1318. Patient with burn right forearm , face and Both leg anterior WT is 62 kg how to calculate fluid requirement
for the patient according to parkland formula ?
Formula = 4 x Weight x burned area …… 4 x 62x 27 ( face 45.+ both anterior leg 18+ forearm 4.5) =6696

1319. CAUSES of spontaneous abortion……… congenital anomalies

1320. Which of the following conditions would be most appropriately cared for by a telemetry unit nurse?
A. New onset of atrial fibrillation

1321. Basal skull fracture signs................... Racoon eyes

1322. VITAIN K dose 0.5mg for preterm ------1 mg for full term

1323. What is the example for primary prevention?


A. Given education on HIV to high school students

1324. Doctor order to give 1000ml over four hour. What is the rate ?
A. 25drop/min
B. 50 drop/min
C. 250ml/hour
D. 500ml/hour

1325. Nurse plan community program to these who experienced a disease and assess the recovery or rehabilitation
or disability . which level of prevention?
A. Primary
B. Secondary
C. Tertiary
1326. IQ is mild retardation characteristic.. can learn some of daily life actives but still need help.
ِ‫أوليسِيُجزىِ ُمحسنٌ ِاالِباحسان‬

1327. IQ 45 considered MODERATE mental retardation

1328. Which patient the nurse should see first?


A. Patient with potassium level 7.5
B. Anemia client with pain level

1329. A nurse obtain a urine dipstick analysis sample from a 35 year-old woman who reports having burning
sensation with urination and a sense of urgency and frequency. She had been diagnosed with the condition six
months previously and was prescribed a course of antibiotics.
Urinalysis Result Normal Range
Colour Dark yellow Straw coloured
Odour Abnormal Almost nothing
Appearance Turbid Clear
Leukocyte esterase Positive Negative
Nitrities Positive Negative
Which type of pharmacological is most likely?
A. Anti-viral
B. Anti-fungal
C. Anti-bacterial
D. Anti-parasitic

1330. This mask is used for?


A. Airborne

1331. Haemorrhoidectomy intraoperative position ? left anterolateral position

1332. HIV patient will be risk for which of life threating disease ?
A. Pneumonia
B. Tuberculosis

1333. A couple has a son and a daughter and both moved out and got family on their own, which type of family
structure?
A. Extended
B.Blended

1334. Site of weber test?


1. Middle of the forehead

1335. Site of the Rinne Test?


1. Mastoid bone

1336. A female patient diagnosed with cancer she intended to spend a part of her money for the poor people in case
of her cure. this action by the patient is considered?
A. Denial
B.Acceptance
C.Bargaining
ِ‫أوليسِيُجزىِ ُمحسنٌ ِاالِباحسان‬
D. Anger

1337. SIDH ………….nursing management is …………..Fluid restriction

1338. Sharp containers for needles and sharps

1339. Which is consider as normal blood pressure?


A. 90/60
B. 111/72
C.136/82

1340. Patient blood pressure 142/88 what is the pulse pressure ?


Pulse pressure = systolic – diastolic 142 – 88 = 54

1341. When caring for a 3-day-old neonate who is receiving phototherapy to treat jaundice, the nurse in charge
would expect to do which of the following:
A. Turn the neonate every 6 hours
B. Encourage the mother to discontinue breast-feeding
C. Notify the physician if the skin becomes bronze in colour
D. Check the vital signs every 2 to 4 hours

1342. patient presented with high fever, headache, vomiting and neck stiffness for the past 3 days, which of the
following is the first diagnostic intervention for this patient:
A. Urine and stool analysis
B. Lumber puncture with CSF aspiration ……………. Rationale meningitis
C. Complete blood count
D. Chest and abdomen x-ray

1343. A newborn APGAR score at 1 and 5 minutes is 5 and 10, half an hour later the baby became bluish in color with
heart rate of 140/m, your first action would be:
A. Estimate the score again
B. Shower the baby with warm water
C. Give oxygen immediately
D. Ignore the finding because it is normal

1344. A 68-years-old woman diagnosed with thrombocytopenia due to acute lymphocytic leukemia is admitted to
the hospital. The nurse should assign the patient to a:
A. To a private room so she will not infect other patients and health care workers.
B. To a private room so she will not be infected by other patients and health care workers
C. To a semiprivate room so she will have stimulation during her hospitalization.
D. To a semiprivate room so she will have the opportunity to express her feelings about her illness.

1345. A patient complains of severe pain which he stated to be 9/10, the physician ordered morphine 50 mg IV every
4 hours, the last dose was given 2 hours ago, what is the best action his caring nurse would take:
A. Give another dose of morphine
B. Inform the doctor to change the order
C. Distract the patient by TV, radio or games for 2 hours
D. Ignore the patient completely

1346. Which of the following is the best method Report an issue Report an issue to decrease confusion and
irritability for asthmatic patient:
A. Give antibiotic
ِ‫أوليسِيُجزىِ ُمحسنٌ ِاالِباحسان‬
B. Give sedative
C. Give vasodilator
D. Give oxygen

1347. On arrival, she had a severely decreased level of consciousness. Her breathing is shallow and irregular. The
skin is cool, dry and pale. There is generalized non- pitting edema of all extremities and face. Blood pressure
70/40 mmhg Heart rate 60/min Respiratory rate 12/ min Body temperature 35.5 C tympanic What is the most
appropriate method to rewarm this patient?
A. Warmed intravenous isotonic fluids
B. Place the patient is a warm bath
C. Apply heat packs to the head and neck
D. Cover the patient with hospital blankets

1348. During the evaluation at a community clinic, the patient completes the medical history. Which of the following
is NOT a risk factor for an acute myocardial infarction?
A. Coronary artery disease
B. Smoking
C. Hemophilia
D. Hyperlipidemia

1349. Adult patient complains of diarrhea, vomiting, abdomen cramp and pain within the past 2 weeks. The patient
reported that the pain increases when he eats and relieves when he passes stool. Which of the following may be
the cause:
A. Appendicitis
B. Crohnâs disease
C. Ectopic pregnant
D. Cholecystitis

1350. The district nurse visits a 30 year-old woman at home following the delivery of her second child, a full term
girl. Following the delivery of her first child, she had Report an issue Report an issue developed a breast
infection and stopped breastfeeding because of the pain. She asks the nurse how she can best prevent it with
this infant. What is the most appropriate response?
A. Provide feedings on demand
B. Apply vitamin E cream to the nipples
C. Request a prophylactic antibiotic
D. Apply heat to the breasts after feeding

1351. Measles isolation period 5 days after rashes appears

1352. Rubella ( geminal measles ) isolation period 7 days after rashes appears

1353. Different between Measles and Rubella is Spot Fever

1354. Dr order to give 1000 IU of heparin and the available is 5000 IU per 5 ml how many ml the nurse will give?
A. 1ml
B. 2ml
C. 3ml
D. 4ml

1355. Rheumatic fever test is


A. Blood culture
B. Antisreptolysin test
1356. As identified by DR Elizabeth Kubler , which stage of dying is characterized by the transition from ‘ NO’ not me
to “ yes ,me but…..”
ِ‫أوليسِيُجزىِ ُمحسنٌ ِاالِباحسان‬
A. Anger
B. Depression
C. Acceptance
D. Bargaining

1357. The hospice nurse visits a client who is dying of ovarian cancer. During the visit, the client says, "If I can just
live long enough to attend my daughter's graduation, I'll be ready to die." Which of the following phases of
coping is this client experiencing?
A. Anger
B. Denial
C. Bargaining
D. Depression

1358. A female patient diagnosed with cancer she intended to spend a part of her money for the poor people in case
of her cure. this action by the patient is considered?
A. Denial
B. Acceptance
C. Bargaining
D. Anger

1359. The said that the head staff gave all of them the LOW evaluation because she did not have time to review their
of work? What is the behaviour of the head nurse in this situation?
A. Halo Effect
B. Horn Effect

1360. Adult CPR ?


A. 30:2 100 at 5 cm
B. 15:2 120 at 5 cm
C. 15:2 120 at 5 cm
D. 30:2 100 at 4 cm

1361. Braxton Hicks contractions ( false Labor ) In false labor, contractions are irregular and do not produce dilation,
effacement, or descent ‫مهم جدا اعراض الطلق الكاذب‬

1362. Influenza in HONG kONG and killed millions of people consider as


A. Epidemic
B. Pandemic

1363. Otitis media surgery is


A. Myringotomy
B. Tympanoplasty

1364. A13-weeks-pregnant,multigravida women is anxious, and apprehensive she has five children and is not
willing to continue with this pregnancy she is requesting the midwife to about the fetus, she is underweight,
malnutrition and is over worked, BMI 17kg/m2,what intervention is desired immediately ?
A. Admission and intravenous line management for induction.
B. Family planning and birth control measures
C. Dietary management and supplements.
D. Support , reassurance and counselling.

1365. During the research process, when should a hypothesis be developed by the researcher?
A. Before any statistical analysis
B. After a research design is determined
C. Before development of the research question
ِ‫أوليسِيُجزىِ ُمحسنٌ ِاالِباحسان‬
D. After development of the research question

1366. ER witnessing a victim escaping from a house fire, a passing nurses es to aid the individual. He has sustained
burn to face and neck, oke inhalation, and is having slight difficulty in breathing. Which of the following is most
appropriate to perform while waiting for emergency medical personnel?
A. Place in supine position and give rescue breathing
B. Place in a comfortable position and monitor for patency of the airway
C. Keep the patient in standing position and support while leaning on the wall
D. Place a wet towel over the face of the patient and placed in supine position

1367. Hemorrhagic fever disease caused by viruses ( arenarivdae , filoviridae , and flaviviridae)
1368. Pap smear test should every ?
A. 1 year
B. 3 years
C. 5 years
D. 6 years

1369. 20. An older adult client with a history of blood clots is in the emergency room with suspected deep vein
thrombosis (DVT) of the left leg. The nurse starts IV heparin as ordered. Which of the following is LEAST likely to
be included in the care plan?
A. Ambulation as tolerated
B. Warm, moist soaks applied to the affected area
C. Analgesics as ordered
D. Anti-embolism stockings

1370. Breast self examination should done monthly ..... after period

1371. Mamogram shoud be done yearly after age of 40 years


1372. Most common cause of death in saudi arbia is cardic disease and accident

1373. A 50 year-old woman post myocardial infraction was admitted in the medical ward . A nurse was assigned to
care for the patient. Blood pressure 140/80 Heart rate 120 Respiratory rate 22 Temperature 37.5 O2 sat
95%
A. Arrhythmias
B. Endocarditis
C. Cardiac failure
D. Cardiogenic shock
1374. Signs of Shock :
ِ‫أوليسِيُجزىِ ُمحسنٌ ِاالِباحسان‬
A. Increase BP
B. Decrease Heart rate

1375. Uterus level at 16 week : is half way between symphysis pubis and
umbilicus

1376. Uterus at 36 weeks at the xiphoid process

1377. The nurse is using BATHE technique to assess a client who recently lost
his friend at vehicle accident which interview question is most appropriate?
A. How are you handling the situation

You might also like